Combined remembered Qs 2020

Decembery RQS – NBDE II

December: • trismus in which space The hallmark of a masticatory space infection is trismus or infection in anterior compartment of ​ ​ lateral pharyngeal space results in trismus. If these infections are unchecked, can spread to various ​ facial spaces of the head and neck and lead to serious complications such as cervical cellulitis or mediastinitis • propanolol interacts with- LA or epi to cause bradycardia If a patient on a nonselective beta-blocker receives a systemic dose of epinephrine, however, the ​ beta-blocker prevents the vasodilation, leaving unopposed alpha vasoconstriction. The resulting hypertensive reaction can be large, with systolic pressure well over 200 mm Hg. ​ • (I actually surprisingly didn't get a lot of pharmacology on both days which was kind of disappointing because I prepared the most for it) • recent studies cannot prove perio due to- smoking or vit deficit • trephination is? (definition) surgical creation of a fistula by puncturing the soft tissue and bone overlying the root apex to ​ provide drainage of infectious materials. Also called apicostomy. Surgical creation of a drainage tract ​ ​ with a bur or sharp instrument in the soft tissue or bone overlying a tooth root apex. • osteotomy/tectomy (definition)

• alveoloplasty performed immediate or after extractions

● xylitol best form? gum/spray/patch Xylitol is widely used as a sugar substitute and in "sugar-free" chewing , mints, ​ ​ ​ ​ ​ ​ ​ ​ and other candies. ​ ​ • calculations sensitvity specify 2 qs

Combined remembered Qs 2020

• Specificity calculator to evaluate the chances of a person being affected with diseases, calculated based on the present health conditions. Negative cases are classified as true negatives (healthy people correctly identified as healthy) whereas false negative (sick people incorrectly identified as healthy). Specificity = True Negative / (False Positive + True Negative) x 100 ​ Sensitivity (true positive rate or probability of detection) is a statistical method that correctly ​ identifies a person with a disease. Sensitivity = True Positive / (True Positive + False Negative) x 100 ​

• ohl which papilla ​ ​ • cementoblastoma pic

• calcified carotid? xray pic

Combined remembered Qs 2020

• boggy gingiva diffuse ewings or leukemia young pt pic. pt had bright red spongy gums leukemia • best anti seizure in kids? diazepam post trauma seizure • which prep first in class 3? which filled first?prep large cavity first ,fill the small cavity ​ first. • pic with 4 dots over gums of centrals I put sinus • aggressive perio bacteria action LAP is localised to first molar or incisor interproximal attachment loss, whereas GAP is the interproximal attachment loss affecting at least three permanent teeth other than incisors and first molar. 65-75% of bacteria are Gram-negative bacilli ​ rapid loss of periodontal attachment associated with Actinobacillus actinomycetemcomitans, Capnocytophaga spp and Porphyromonas gingivalis The inflammatory exudate in the gingival tissues and gingival crevicular fluid is mostly polymorph neutrophils but also includes B cells and plasma cells. ​ ​ ​ ​ ​ The plasma cells produce specific antibodies in response to the periodontal pathogens, IgG ​ increased amounts IL-1α and IL-1β which cause osteoclastic bone resorption ​ ​ ​ ​ The main distinction between the localized and generalized form of AgP lies in the number of teeth affected. GAP brings about attachment loss involving more than 30% of sites on teeth;[1] effectively ​ ​ being at least three permanent teeth other than the first molars or incisors.[16] ​ • best way to differentiate lesions cyst granuloma- biopsy/histollogical evaluation • incisional bipsy excisional biopsy measurements. questions were like, best to use which for diagnosis Incisional biopsy is often used in lesions larger than 1 cm in size, as opposed to excisional biopsy, which is often used in lesions less than 1 cm. • first pass metabolism absorbed in intestine or enterohepatic circulation ​ ​ First pass effect: After a drug is swallowed, it is absorbed by the digestive system and enters the portal ​ ​ ​ ​ ​ circulation. The absorbed drug is carried through the portal vein into the liver. The liver is responsible for ​ ​ ​ metabolizing many drugs. ​

Combined remembered Qs 2020

• day 2 • asa classifications. I had probably 3 questions on this current

ASA I Normal healthy patient non smoking, no or minimal alcohol use

ASA II mild systemic disease smoker, social alcohol drinker, pregnancy, obesity, (30

ASA III severe systemic disease one or more moderate to severe diseases. poorly controlled DM or HTN, COPD, morbid obesity (BMI>40), active hepatitis, alcohol dependence or abuse, implanted pacemaker, history of MI (> 3 mos), TIA, CVA

ASA IV severe systemic disease that is recent MI (< 3 mos), CVA, TIA, or CAD/stent, ongoing constant threat to life cardiac ischemia or severe valve dysfunction, severe reduction of ejection fraction, sepsis, DIC, ARD or ESRD not undergoing regularly scheduled dialysis

• qs about teeth not even in arch so always check ALLLL the pics and don't get confused. • worst perio prognosis? furcation/mobility/7mm pocket ​ ​ • levothyroxine moa? Levothyroxine is a synthetic form of thyroxine (T4), an endogenous hormone secreted by the thyroid gland, which is converted to its active metabolite, L-triiodothyronine (T3). T4 and T3 bind to thyroid receptor proteins in the cell nucleus and caubse metabolic effects through the control of DNA transcription and protein synthesis.

Combined remembered Qs 2020

December 6&7 RQ’s:

1. Best cement for temporary crowns- -Polycarboxylate temporary luting cements present low postoperative sensitivity, adequate retention, and ​ ​ easy cleanup. Examples of this category are: Cling2 (CLINICIAN'S CHOICE) and Hy-Bond (Shofu Dental). ​ ​ -ZOE temporary luting cements are commonly used because of their sedative effect on sensitive teeth. It is well ​ ​ documented that eugenol is able to penetrate and diffuse throughout the dentin and can affect the bond strengths provided by resin materials used for definitive restorations. It presents an excellent antibacterial effect. -ZONE temporary luting cements are designed to replace eugenol with various types of carboxylic acids that do ​ ​ not interfere with definitive cementation. They have the characteristics of being compatible with resin provisional materials, are compatible with permanent resin cements, and show greater retention compared to ZOE cements but have no sedative effect on the pulp. -Resin temporary luting cements present high strength, excellent retention, better aesthetics, and easy cleanup. However, these temporary cements have a higher incidence of microleakage, discoloration, and odor associated with their use.

2. Pt came with diastema between two centrals and one of the central is rotated initial treatment for this patient for correcting his diastema (derotating his centrals)

3. 3 questions on efficacy and potency by asking which one of the above drug is showing more efficacy (I remembered efficacy as taller,potency as faster onset)

Combined remembered Qs 2020

4. Pt with neuromas on his skin,half swollen which disease- Neurofibromatosis ​

5. Pt with micrognathia,glossoptosis,cleft -Pierre Robinson syndrome ​

Combined remembered Qs 2020

6. Embrassure rest,I bar used on which side- mesial ​

7. GIC does show fluoride release and one more advantage of GIC is - ionic bond with dentin ​ and enamel

Combined remembered Qs 2020

8. East-west cryers elevator used for which tooth- mandibular molar roots Cryers: #27 Left, #28 Right 9. Thick viscous material to fluid this is know as- Thixotropic ( periodontal ligament has it ) ​ ​ ​

10. Width of attached Gingiva- options are tricky like mucogingival groove to vestibular Phoenix,free mucosa to alveolar mucosa The width of the attached gingiva is defined as the distance between the and the projection on the external surface of the bottom of the or periodontal pocket 11. To decrease penumbra- decrease object film distance

Combined remembered Qs 2020

The size of this penumbra can be reduced by using a smaller object film distance (OFD), a greater ​ ​ ​ ​ focal film distance (FFD) and a smaller focal spot size. The larger the focal spot, the larger the size of the penumbra and the lower the image resolution. With a smaller focal spot size, a smaller OFD and a ​ ​ larger FFD the image is magnified to a lesser extent and the resultant image is of greater accuracy.

12. Two implant distance-3mm ​ 13. Implant to tooth distance-1.5mm ​ 14. Bioharzard waste regulation by- EPA ​ 15. Abused child: local authorities o social services Child Protective Services: social servce stafif obtains facts from the person making the referral 16. Old person abuse-no DHHS ( doctor in human health services) Adult Protective Services (APS) agency 17. Antibiotic prophylaxis needed for- Artificial Aortic valve pt

18. Antibiotic for penicillin allergy pt- Axzzithromycin 500mg ​

Combined remembered Qs 2020

20. Post operative sensitivity after perio surgery decreased by doing-plaque control 21. Percentage of fluoridation water in USA- 74% ​ 22. Mucous retention cyst of lower minor due to- trauma ​ 23. Hyperplastic tuberosity removed by which method- wedging technique ( distal wedge) 24. Tricyclic antidepressants mainly act on which receptor for depression- serotonin receptor

25. Crown fits on die perfectly but not on pt tooth cause- while trimming the die technician trimmed the finish 26. Taurodontism-morphodifferentiation ​ Failure in morphodifferentiation results in size and shape abnormalities, such as peg lateral incisors and macrodontia 27. After applying benzocaine on mucosa pt became bluish and some other feathers I forgot – methemoglobinemia ​ 28. single one of aspirin action on platelet aggregation how long [time] 7-10 days

30. Cleft pt of 15years have underdeveloped maxillary even after surgeries what might be the cause:

Combined remembered Qs 2020

Cleft palate repair. These procedures can affect the vertical, anteroposterior, and transverse development of ​ the maxilla and alveolar processes. Periosteal stripping at the time of surgery and the resulting fibrosis are the ​ ​ most likely reasons for this response. 31. embrasure rest and I bar clasp work on which side: MESIAL repeat question #6 above

32. pt with erupted 2 laterals palatally what might be the cause: ??? ​ ​ 33. -this is pale and told you that she is feeling unwell and faint what’ve is the cause: vasovagal syncope

34. Retrognatic mandible which ostectomy- sagittal (Bilateral Sagittal Split Osteotomy ) In patients for whom only mandibular deficiencies need to be corrected surgically, bilateral sagittal split osteotomy (BSSO) is the most common procedure. Intra-oral distraction osteogenesis during orthodontic treatment as a solution for a Class II has been proposed as an alternative to BSSO.

35. Neutrophils <1,000-no surgery should be done on this patient ​ ​ 36. Pt with good maintenance visit now showing pocket depth>5mm- need skillful ​ periodontist to remove the calculus with curettes one more option was ultrasonic tip cant be ​ that effective in removing the calculus below 5 37. Guided imaginary: Guided imagery is an extremely, effective, non-painful, inexpensive and easy mind-body technique people can practice in advance of dental appointments for best results and can also listen to in the waiting room and during dental procedures.

38. Crown fits on die perfectly but not on pt tooth cause- while trimming the die technician trimmed the finish line repeat question #25 above ​ 39. C factor- bound to unbound surfaces

Combined remembered Qs 2020

40. PMMA as a temporary crown- exothermic reaction Tensile strength decreases with increased water absorption.[27] Its coefficient of thermal expansion is ​ ​ ​ ​ relatively high at (5–10)×10−5 °C−1. 41. Marijuana effects except- increases appetite

The short-term effects of marijuana or cannabinoid use include:

● increased heart rate ● low blood pressure, orthostatic hypotension ● muscle relaxation ● slowed digestion ● dizziness ● distorted perception (sights, sounds, time, touch) ● difficulty in thinking, memory, and problem solving ● loss of coordination and motor skills ● agitation, anxiety, confusion, panic, paranoia ● increased appetite ● dry mouth, dry eyes

42. Main component of biological width-options junctional epthelium, 43. Which is not act on cell wall- azithromycin Azithromycin is a bacteriostatic drug acts by inhibiting protein synthesis. It binds reversibly to 50S ​ ribosomal subunits of sensitive microorganism. Azithromycin interferes with transpeptidation and translocation ​ thus there is inhibition of protein synthesis and hence inhibition of cell growth.

44. Metastasis to mouth from all except- brain 45. Access opening of mandibular molar round bur directed towards-no option of pulp chamber dear options Towards furcation,towards mesiobuccal,towards mesiolingual,towards distal orifice

Combined remembered Qs 2020

46. Sailolith most common which duct- Wharton’s duct ​ Sialoliths are most common within the body of the or Wharton's duct (80% to 90%). 47. Vestibuloplasty which flap- supraperiosteal flap 48. Single dose of aspirin effect on platelet aggression duration- I thought it mainly act on trombaxane A2 so it will be 10hours repeated question #28 ​ the effects of aspirin last for the duration of the life of the platelet [10 days] 49. Before surgery warfirin should be stopped till- 3 days (1-3 days) Warfarin therapy should be stopped five days before major surgery and restarted 12 to 24 hours postoperatively. 50. Change in taste sensation which drug- losartan(metallic taste) ​ Losartan is angiotensin II receptor antagonists.

51. Strength of zirconia is due to- Phase transformation ​ The high fracture toughness exhibited by many of zirconia ceramics is attributed to the constraint of the tetragonal-to-monoclinic phase transformation and its release during crack propagation.

52. Position of teeth while talking-teeth doesn’t touch each other 53. Pt who takes 30packets of cigarettes per year which stage he is in- pre-contemplation ​ 54. Chroma- Saturation, Hue- Wavelength ​ ​ ​ 55. Natural characteristics of enamel at its Incisal edge- options that I was in between was opalescence

Translucent enamel displays the characteristic of opalescence. Opalescence causes tooth enamel to reflect ​ ​ blue light back to the observer. Enamel has a primary mineral makeup of hydroxyapatite, which is crystalline calcium phosphate. Hydroxyapatite crystals align in organized, tightly packed masses to form prismatic enamel rods.

Combined remembered Qs 2020

56. Xylitol most effective- chewing gums ​ repeated question

57. Pictures of Lymphoepithelial cyst

58. “ recurrent herpes

59. “ sinus retention cyst

Combined remembered Qs 2020

60. “AOT X-ray On radiographs, the adenomatoid odontogenic tumor presents as a radiolucency (dark area) around an unerupted tooth extending past the cementoenamel junction.

61. Ear ring X-ray

62. Mandibular tori X-ray

63. Mediastinum space infection spreads from- ​ 64. Guided bone graft best in - 3 wall infrabony defect ​ 65. Kids heart rate- 110 (70 to 110 beats per minute) ​ ​

66. Dr said the pt to get change his amalgam filling with composite which ethics- Veracity 67. Antibiotic used for- ANUG (other options herpengina,lichen planus, aphtus ulcer ​ ​

Combined remembered Qs 2020

Treatment of ANUG is by debridement (although pain may prevent this) and antibiotics (usually metronidazole) in the acute phase, and improving oral hygiene to prevent recurrence. herpangina is a viral infection

68. Gardner syndrome : multiple osteomas, cotton wool, supernumerary, café au late 69. Micrognathia,cleft palate and glossoptosis- Pierre Robinson Syndrome repeated 70. Traumatic Neuroma

In the oral cavity, the most commonly affected sites are the lip, tongue, and mental nerve area. Pressure on the suspected area usually provokes pain.

71. Adrenocorticosteroids used for antiinflamatory and analgesic 72. IAN didn’t work which accessory nerve- Mylohyoid 73. 16 year old Pt says anaesthesia will go away very quickly which LA will you give – Benzocaine 74. 1cartridge no anaesthesia what to do before second dose- wait for 5 more min 75. Digoxin work by +cardiac inotropic affect increases the force of contraction on cardiac ​ ​ muscle.

Combined remembered Qs 2020

76. Nitroglycerin work by- vascular smooth muscle relaxation 77. Morphin work like- enkephalins 78. Pear shape bur- 245, 330 ​

79. Nitrous oxide toxicity symptoms- nausea,tingling of fingers

Signs and symptoms of nitrous oxide-induced neurotoxicity include: numbness and tingling of the distal extremities and impairment of vibration sensation, light touch and pinprick, proprioception, and gait.

80. Microabrassion question 81. Y line of Ennis X-ray- (nasal fossa&maxillary sinus) nasal cavity (straight radiopaque line) and border of the maxillary sinus (curved radiopaque line) 82. Restoration covering the cusp- Resistance form 83. Picture of compound odontoma ● complex odontoma: irregular calcified lesions with no distinct tooth components ​ ● compound odontoma: identifiable tooth components, collection of small teeth, average age of 14–18 ​ years. They are slightly more common in females and more common in the maxilla, especially the anterior maxilla ● Odontoma may occasionally grow large, resulting in bone expansion

Combined remembered Qs 2020

● Generally, odontomas have been associated with trauma during primary dentition as well as with inflammatory and infectious processes, hereditary anomalies (Gardner syndrome, Hermann's syndrome), odontoblastic hyperactivity and alterations in the genetic components responsible for controlling dental development

84. Stafne bone cyst clinical features

Stafne’s bone cavity is an uncommon bony defect occurred especially only at the lingual cortex of mandible. Clinically, patients with SBD are usually asymptomatic. Ovoid or round radiolucency can be observed near the angle of mandible, usually located between the inferior alveolar nerve and the lower margin of mandible.

85. For small class 1 cavity which cement do you prefer- composite 86. Modified wickman flap- 4questions

MODIFIED WIDMAN FLAP ● The aim of the modified Widman flap surgery is healing and reattachment of periodontal pockets with minimum loss of periodontal tissues during and after surgery reduction of probing pocket depth by shrinkage individually occurs. ● No surgical pocket elimination and apical displacement of the flap. ● The MWF is indicated for the treatment of all types of periodontitis and provides excellent result with probing depths up to ca. 6 mm. ● Advantages --> 1. Root cleaning with direct vision 2. Protective of tissues, reparative 3. Healing by primary intention 4. Lack of pain or complications postoperatively. ● The flap surgery should not be initiated until one or two months after completion of the hygienic phase of the periodontal therapy. ● The 1st incision "internal reverse bevel incision" should be made with a knife that can be directed parallel to the long axis of the tooth. ● 2nd incision is made around the neck of each tooth from the bottom of the pocket to the alveolar crest. Vertical gingival releasing incision usually is not needed. A full thickness flap is elevated for only 1-2 mm from the alveolar crest as needed for access to the root surfaces and the interproximal one.

Combined remembered Qs 2020

● 3rd incision is made with a narrow interproximal knife. The buccal and/or lingual flaps are deflected by a periosteal elevator on top of the alveolar crest to dissect free the collar of gingival tissues, which is been separated from the buccal and lingual gingival flaps and the teeth. The separated collar of gingival tissue is then removed with curettes. 87. Gingivectomy contraindicated in- if the pocket is below the crest of bone ● Access to osseous structure is critical or ● Gingival attachment is inadequate (minimal) or absent, ● Bottom of the pocket is apical to the mucogingival junction, ● Esthetic considerations, anterior region of maxilla

88. Peutz- Jegher Syndrome

mutation of tumor suppressor gene stk11

89. Apexification,Pulpectomy questions- 3 questions on them 90. Irreversible pulpitis questions 91. Dentigerous cyst- cyst with epithelial lining filled with clear fluid attached at CEJ 92. Gorlin Goth Syndrome which cyst- OKC 93. Attrition question 94. 3 years ago pt had trauma to the mandible radiographically large radiolucency. What is your next step in treating this patient - check for vitality and do RCT for non vital teeth 95. Chronic alcoholic abuse 4 molars extraction which analgesic- options Ibuprofen,acetaminophen,hydrocodone,acetaminophen + hydrocodone

Ibuprofen- Use with caution in patients with hepatic impairment; patients with advanced hepatic disease are at ​ an increased risk of GI bleeding with NSAIDs. Acetaminophen- Avoiding use in patients with advanced chronic liver disease or cirrhosis who are actively drinking alcohol, malnourished, not eating, or receiving a concomitant interacting medication. Hydrocodone - Use with caution in patients with hepatic impairment; dose adjustment may be needed.

96. Premolar class 5 cavity which filling- GIC 97. Adolescent Xerostomia due to- 98. Catastrophizing- It is a common cognitive distortion that makes us anxious, depressed and unmotivated. It's basically expecting the worst of current situation or of future situation.

Combined remembered Qs 2020

99. Fetal alcoholic syndrome c/f except- prominent &sharpe nose 100. Peripheral odontogenic fibroma-2times 101. Marginal ridge width in occlusal rest- 1.5mm 102. Malignant melanoma common on- palate,gingiva 103. Picture of blue Nevus 104. PSA artery on X-ray identification 105. H2 receptor blocker MOA- deceases gastric production 106. Pt has allergic reaction,hypotension,Bronchoconstriction first line of treatment for this patient- Epinephrine 107. Osteogenesis Imperfecta related to- Dentinogenesis imperfecta 108. Neck trauma pt for airway- lift head back 109. Primary stress baring areas of mandible- buccal shelf & alveolar ridge Note; (Primary in maxilla is alveolar ridge n secondary is palatal rugae mandible it is buccal shelf and secondary alveolar ridge)

110. Mnodulesonths baby presents with midpalatine whitish on examination they are not getting rubbed off with pressure what are they- Gingival cyst of palate 111. Deepest part of occlusal rest- fossa 112. Benzodiazepines antagonist- Flumazenil ​ 113. Opiods antagonist- Naloxone ​ 114. Ruler used in ceph for- magnification 115. High copper and low copper amalgam identification by seeing the pictures 116. Osteoporosis X-ray 117. ASA 4 question 118. LA calculation easy one 119. Collimation- lead 120. Infection from lower 2nd premolar goes into which space- sublingual space 121. evaluation of 2 groups A and B with 2 drugs for same period, what type of study- clinical ​ trial Cross sectional study ​ 122. A question on Paraphrasing- Tell the patient what you think you understand 123. Acid etch does all except- improve esthetics 124. Sterilization monitoring by- biological agents(spores of bacillus steothermophillus) 125. Pt has nocturnal pain- irreversible pulpitis 126. Medically compromised child- Chlorhexidine mouth wash 127. Hyoid bone - they asked like what you see beside C4 vertebrae

1. Fungiform papilla swelling. Sloughing of the tongue, red and swollen. Fever. → Scarlet fever.

Combined remembered Qs 2020

2. Picture of red patch on tongue, and the question says that there are same red patches on palate, too → Syphilis.

3. X ray showed multiple radiolucencies in skull. P’t has bone pain. Multiple myeloma

4. 2 questions on eagle syndrome. P’t has pain when turning his head and swallowing.

5. In which condition does p’t get thinner hair? →Hypothyroidism.

6. Hypercementosis in which disease? →Paget’s

7. Antidote for opioids. →Naloxone.

8. Antidote for BZDs. →Flumanezil

9. Just cemented an onlay, 1 hr later p’t returns complaining that every time he bites, there is sharp shooting pain. →Hyperocclusione

10. Children with HIV are likely to get what kind of infection? →Bacterial

11. Why shade selection before RD application? →dessication makes tooth look whiter

12. What receptors do opioids act on. →Enkephalin (which is another na)me for opioid receptor).

13. Mechanism of BZDs. →GABA receptor agonist.

14. Posture of p’t just after he got an asthma attack and treated with B2 agonists? →Upright, and a little bit crouched.

15. Adult can make decision for old dude under what circumstances. →Power of attorney?

16. A dentists reports to insurance that he just did a 2 surface restoration. However, the insurance only paid for 1 surface restoration. →Downcoding.

17. 2 questions on mechanisms of antitumor drugs. Forgot the details, they all looked the same to me.

18. Higher concentration in gingival crevice? →Tetracyclines. ​

19. Cannot be used in p’ts with decreased plasma esterases? →Articaine (has an ester side chain).

Combined remembered Qs 2020

20. What kind of caries is increasing? →Root caries (due to more elderly people).

21. Which of the following root caries is active? →The one with soft lesion and plaque covered.

22. P’t just got trauma and broke his central incisor, horizontal root fracture in middle third, what should you do? →Splint and follow, if necrosis RCT.

23. What will you not do in a through and through furcation defect? →GTR. Other options were tunnel, hemisection and extraction.

24. Highest chance of 2 canals in which mandibular teeth (Options: lateral, canine, 1pm, 2pm). →lateral

25. Perforation of maxillary central incisors during RCT opening? →Mesial

26. You give p’t a IAN block. His tongue is numb but not his lip. What do you do? →Another IAN.

27. IV sedation adverse effect? →Hypoxia

28. P’t taking NTGs 2-3 times daily →ASA IV

29. What is the advantage of brushing your tongue? →Get rid of malodor.

30. Highest concentration of NO? 70% NO/30% O2

31. A doctor uses a new technique for diagnosis. As a result, lots of people with the disease are classified as not having the disease. The test has low ___? →Sensitivity

32. Pano of a pterygomaxillary fissure.

33. X ray of a p’t with enlarged pulp occluso-apically. →Taurodontism.

34. Why can’t you use tetracyclines along with penicillins? →Inhibit bacterial growth.

35. The century-old debate about the dentist who gets pissed off when working but keeps his cool when working, but as soon as the patient leaves the assistant drops something and he yells at her. I still don’t know what the correct answer to that question is btw. There seem to be two different opinions in the group.

36. H2 receptor blocker MOA→ldeceases gastric production ​ ​

37. A question about pierre-robin syndrome (micrognathia,cleft palate and glossoptosis)

Combined remembered Qs 2020

38. Aspirin acts on? trombaxane A2

Rqs 1.Dissensitizing ingredient percent: potassium nitrate 3-5 %

2.Doses for antibiotic prophy for prosthetic heart valve, allergic to ampicilin : clindomycin 600mg

3.Cleft lip what week in utero: 6 to 9 week

4.Aspirin adult max dose: 4000 mg

5.Px with HIV what can you sedate them? Midazolam

6.Amalgam ingredient to decrease corrosion?Chromium

7.Restoration on cusp indicated for? A) resistance b) retention

8.What spore hard to sterilize: bacillus thermophillus

9.Cyclic neutropenia: CBC

10.LA doesnt work on what kind o perio- abscess

11.Tooth most susceptible to coronal caries-mandibular first molar

12.Invisalign edge importance? Hue, chroma translucent value

13.What holds sealants: mechanical, deep fisures

1sd14.Extrusion of posterior teeth causes? Deep bite, incisor , incisor extrusion ​ ​

15.Probing numbers is an example of?? Nominal ordinal ​

16.Most esthetic pontic?modified ridge /ovate

17Least likely to produce gingival hyperplasia? Nephidine, propanolol, diazepam, phenitoin ​

18.Spectrum of penicillin? Narrow, broad, ​ ​

19.Child with bad hygiene what ortho tx? Removible, fixed, partial removible ​ ​ ​

Combined remembered Qs 2020

20.Kinetic frug question that doesnt matter about dilution? First order single compartment ​ ​ second compartment 2 order

21.Overnightpain: naproxen

22.Epithelium for implants attach: hemidesmosomes

23.Contraindication to implant placement: adolescent ​

24.Nerves from infraorbital- MSA /ASA

25.What nerves moves upper eyelid? The oculomotor nerve (CNIII) innervates the main upper eyelid retractor, the levator ​ ​ palpebrae superiorus, via its superior branch.

26.What will you not see on 3year old teeth? Pellicle, calculus, plaque ​ ​

27.Wich IL causes bone loss? ILL 1

Interleukin-1 (IL-1), a cytokine best known for its ability to stimulate lymphocyte proliferation, has recently been shown to stimulate bone resorption and modulate bone formation in vivo. Yale researchers learned that interleukin-1 not only causes a loss of bone but has ​ ​ been shown to increase the synthesis of collagen, a connective tissue protein, which can cause scar tissue in the abdomen. Symptoms Linked to Interleukin-1 28.Corrosion resistance: chromium

Corrosion resistance of various steel types increases with increase in chromium ​ content. 29.Sibilant position:

Words with the sibilant sound (hissing sounds) are pronounced correctly with the ​ incisal edges of maxillary and mandibular almost touching. These sounds are usually ​ produced between rest and the occluding position.

30.Dont need radio for? Hutchinsons teeth

31.Chronic vs suppurative(acute) : percusion

32.Endo perforation on maxillary central on what area? Mesial

33.Retention pin per cusp??

Combined remembered Qs 2020

34.Composite photo initiator: camphoroquinone

35Varices in tongue: hypertension ` A varicosity is a condition indicating an enlarged and tortuous vein. Previously described lingual ​ ​ ​ ​ varices referred to sublingual varices on the ventral surface of the tongue or floor of mouth. This represents the first reported case of dorsal tongue base varices in a patient with portal hypertension. ​ ​

36.Desquamative gingivitis causes: lichen planus

37.Epulis fissuratum resembles : fibroma

38.Absolute contraindication for endo: leukemia, HIV, rheumatoid artritis ​ ​

39.Tb: painless ulcers

40.MRI: radio waves

41.Shortness of breath, pedal edema: CHF

42.Side effect for nitroglycerin?

Adverse Reactions >10%: Central nervous system: Headache (patch, ointment: 50% to 64%; sublingual powder, lingual ​ ​ spray: >2%)

1% to 10%:

Cardiovascular: Hypotension (≤4%), syncope (≤4%), peripheral edema (lingual spray: ≤2%) ​ ​ ​ ​ ​ ​ Central nervous system: Dizziness (>2% to 6%), paresthesia (>2%) ​ ​ Gastrointestinal: Abdominal pain (lingual spray: ≤2%)

Neuromuscular & skeletal: Weakness (all sublingual forms: ≤2%)

43.O in PICO ( population, intervention, comparison, outcome) ​ ​

44.Ruler on lateral cephalogram: magnification

01/03/2020

Know endo pulpal diagnosis •Perio diagnosis

Combined remembered Qs 2020

•Hardest 3rd impacted molar to extract- vertical impacted •Pulpotomy vs pulpectomy for peds •Apical scar •Apexogensis, apexification indcations •Sinus tract drainage- chronic apical abscess ​ •Perio abscess vs endo abscess (wide lateral defect, percussion) •Endo file separation= best prognosis for vital tooth without par oYou separate an endo file 3mm from the apex and obturate above it... which case will show the best prognosis? a. vital pulp w/ no periapical lesion (yes) •Tooth avlusion splinting-7-10 DAYS •Most common permanent tooth missing - 3rd molar, then the mandibular 2nd premolar, then ​ the maxillary lateral incisor.

•Den in dens tooth? maxillary lateral incisor ​ •Replacement resorption and ortho- ANKYLOSIS •Straight line access easier in short teeth •Red complex bacteria = Porphyromonas gingivalis, Treponema denticola, and Tannerella ​ forsythia. •Some indirect restoration questions •Forceps for extraction permanent max PM-150 •Burs for smoothing out preps? - More flutes and shallow •Implants= 3mm between implants, 2mm from vital structures, high torque low speed ​ •: You did a prep with high speed + diamond bur and tooth is sensitive, what is it about bur and handpiece that it caused sensitivity? A) Desiccation B) Traumatized dentin C) Heat ​ 1.What would cause displacement of odontoblastic processes? Thermal Desiccation Mechanical Chemical •Whats the indication for # of pins to put when you are placing an amalgam (idk) •Indications for onlay- I put not enough dentinal structure under cusps •Cement veneers and see brown discoloration on margins? I said resin or microleakage depending on q •Inow indications for composite (not good for big posterior fillings) •Why GI is good for class 5 •**know HUE, VALUE, CHROMA (chroma=saturation) , choose hue first… know this well •Most rention of crown? Axial taper (came up 3x) •Which of the following do you not do in cementation of a porcelain crown: etch enamel with hydrofluoric acid •What could the reason be if you see opaque white porcelain in the incisal 1/3 facial of the PFM crown: Inadequate reduction of the inciso facial part of the tooth •Most common complication of internal bleaching: cervical root resorption (this wasn’t an answer choice on mine)

Combined remembered Qs 2020

• Pontic for FPD should rest on the soft tissue without blancing •What pontic can you put on posterior FPD that will change esethcis and phonetics and cant be put on anteriors? (I put hygenic b/c idk) •Strength of soldered connected of FPD is due to increasing height •No occlusion q •A LOT of q facebow registration •Question about thyroid storms… •If pt with CD is talking and denture keeps “popping” off what is going on ? I put overextended borders •Pt feels like lip is sticking out with CD (I put adjust labial flange) •Light intiator for composites= camphoroquionine •QUESTION: What happens when you take an impression & lip immediately swells? Angioedema (allergy reaction) •Imbibition and syneresis affect which one the most a. Reversible hydrocolloid •Question about which is the best for indirect restorations and impression material (asked about charactiersics of the material but not the names of the material) •Best interocclusal record for MIP? (I put just cusp tips with some light showing ) •Fl- replace hydroxyl •Osteomas- gardners •Asked about cleiocraial and other ones that cause supernumary •Downs syndrome- lower affinity for caries •dI and dysplasia •how to treat internal resorption •pagets association with cancer (osteosarcoma) treated with bisphos •a lot of osteoradionecrosis (u can figure it out… not hard) >65 high risk of ORN •pyogenic granuloma and pregnancy •neurofibromatosis- lische nodules, etc •cysts not seen on radiograph •pemphigoid – basement membrane •histoplasmosis orally looks like tb •pleomorphic adenoma- most common salivary gland t •Kaposi sarcoma •Congential epulis •A good antifungal that can be taken orally and have systemic effect. Idko fluconazole ​ •Actini cheliltis= loss of vdo •Symptoms of acute herpetic gingivitis •Warty and cauliflower? Hpv v •Oliodontia? Ectodberm

23rd rqs AOT Xerostomia pilocarpine Erythema migrans (description)

Combined remembered Qs 2020

Gingival hyperplasia drugs Resceptors (Epi not epi) Multiple myeloma first symptom (bone pain) Dementia: short term memory loss Drugs mechanism of action Protective stebilization where to use

Jan 24th -If you think the drug causes serious problem, should you repor to? FDA, CDA ​ -Which muscle closes the tongue? Genioglossus, hyoglossal .. -Common fungal infection? Histoplasmosis, candidiasis, blastomatosis -Lesion at the basement membrane? Pemphigoid, pemphigoid, erosion lichen planus -Reduction of the crown should look like? Flat and round, follow occlusal morphology -As you get older, people lose which teeth first due to periodontitis? Max molar, Man molar -bulimia? Caucasian women, Caucasian men, black men, black women -Heparin? PTT, PT ​ ​ -Naloxone: completive antagonist? -Provisional for veneer? Bis-cryl, methyl methacrylc, composite, shell ​ ​ -got a patient who currently prescribed penicillin V for pain. He has a heart condition and need prophylaxis before procedure begin. which prophylaxis should you use? Amoxicillin 2g or clindamycin 600mg -A tooth was root canal treated and there was a small radiolucency under the apex. After 4 years it is still there. What do you think it is? Apical scar, apical cyst, forgot other choices -Which experimental design would you use to study gastric bypass and nutrition? Cohord, ​ ​ randomized clinical trial, observational -primary stability of implant osteo integration is? Patient health status, primary stability of the ​ implant -you give an IA nerve block and the needle inserted medially from the pterygo raphe. Which two muscles involved? medial pteryroid, superior constricture, buccinator ​ -children with congenital heart defect seem emotionally and physically dependent because? limit physical abilities..., parents overly protective, hospitalized frequently… ​ ​ -Coronal caries effect least on which tooth? Max molar, mand molar, max central, man ​ central -Coronal cares affect most on which tooth? Max molar, mand molar, max premolar, man premolar -Patient has a deep carious, non-restorable #23 and the dentist recommended extraction. No medical history. The patient’s spouse told the dentist that the patient has frequent urination, thirst, hunger and lost a lot of weight recently. What is the recommendation for the treatment?

Combined remembered Qs 2020

Extract #23 with local anesthesia,refer to physician refer to the periodontist, Refer to physician -How many % of adults have herpes simplex virus? 20-30%, 50-80%, 100% -calcific metamorphosis ? internal root resorption, external root resorption, something else -sterilize plastic instruments using? glutaraldehyde, phenol, alcohol.. -a pt has deep pocket depth, which procedure would preserve the gingiva? split palatal, apical positioned flap, lateral position flap, gingivectomy -contraindication of root canal therapy? HIV, leukemia, recent MI -Which facial space does infection of Lugwig NOT travel to? Submental, Sub mandibular, sublingual, retropharyngeal ​ ​ -maxillary artery is from which branch? internal carotid, external cortical artery -Development of cleft lip? 2-3weeks, 6-9weeks, 12-15 weeks, 18-21 weeks ​ ​ - You observed the a child was very upset going to the dentist due to a past experience at another medical office. What it is? Association, modeling.. You observed the a child was very upset going to the dentist due to a past experience at another medical office. What it is? Association, modeling.. -Amantadine is an? antivirus, antifungal… ​ ​ -optimal periodontal maintenance is in ? 1 month, 3 month, 6, 12 months ​ ​ -characteristics of body dysmorphic disorder -common sign of mandibular fracture? trismus, parasthesia -symptoms of thyroid storm versus anaphylaxic shocks - Thyroid storm-high temperature, fast heart rate, nausea, vomiting, confusion, perspiration, diarrhea.Anaphylaxis-low BP, difficulty breathing, fast heart rate

More RQS *What is component in algintae that react with calcium n give the desored working time. Trisodium phosphate. ​ ***what is effect of post in post n core: Ferrule effect Retention of core ( i chose) Other choices dont remeber.

* what is the factor that prvent the fracture of tooth with post n core: A. Reaking tooth structure ( i chose this) B. Post length C. Post width

* clinic closed for 2 weeks what is best course of action. Leave without pay

Combined remembered Qs 2020

Termination Non patient care dental practice Part time job * what is behavior shapping * child had multiple vist to clinic, after multiple visit what is cause of objective fear: Previous experiences ( i chose this) F4om parents

***what antibioic for mycoplasma Bacitracin Penicillin Clarithromycin Dont remebr other choice

***simple diagnosis for asymptomatic apical periodontitis, irreversible pulpitis. ** pic of pt had untreated caties n cavitation with small swelling on gum ,asked what was it? Parulis

* what is parulis associated with ?

** intial dose of epinephrine in anaphylactic shock: 0.3 mg 0.003 mg 30 mg * i had an opg ,pt hd a lesion in maxillary sinus had no pain. What was dagnosis? Chronic maxillary sinusitis

Antral Rention pseudocyst??

* yellow painless hard bony swelling in floor of mouth. Diangosis Calcified lymphiepithelaul cyst? Siaalolith ** * all muscles hav e attachment at mandible ramus excpet: Lateral pterygoid Medial pteryhoid Massater These were the only options, i choose leteral

* other name for eosinophilic granuloma: langerhan cell histiocytosis * what is the sructure at rooth of maxillary tooth: Maxillr sinus Zygoma ( i chose this)

Combined remembered Qs 2020

* what causes steep mandibular plane angle? A. Increase vertical grwoth patten with increase lower facial height( ans) B. Deep with incread3 lower facial height * when we do molar uprighting what occurs: Lack of Mesial movement of molar Distal movement. ***while access preparation if u move the bur mesial or dial perforation of what tooth occur? Maxillary first premolar( i chose this but dont know) Mand molar * flossing doesnt not clean the proximal surface of which tooth? MesialMaxillary first premolar Distal of max lateral incisor Dont remeber other options ** Dont remember other option * pt was had depression n hypertension which antidpresant with propanol will cause severe imcrese in blood pressure: Imipramine Fluoxetine

* which of following describe extent of periodontitis: Chronic Localized

* gingival enalrgement occur in all of following excpet: Gingivostomatitis Desquamtive gingivitis Leukema Pregmacy

* 50 pound pt, haow many cartridge of lidocaine 2 4 6 8 **opiod odesnt cause Somnolence Periperal pain inhibition Contipation Dont remeber other choices * pt recently diagnosed with alzheimer what will u do: A. Start **what treatment for tmj?

Combined remembered Qs 2020

Splintnonly * common site of salivary gland tumor? Palate minor salivary gland Parotid * hyperventilation symptoms Bradypnea Dizziness n lightheadedness * down syndrome all except: Delayed erution Maxroglossia Epicanthal fld Short strature * what is symptom of MI: A. Pain , ECG abnormalities, elevated serum enzymes B. Pain, frictional rub, elevated CPK ** CI for endodontic treatment: A. Diabetes B.uncontrolled hypertension. **what is wheezing A. on ausculation an exhaltion sound is heard that pops open n close B. Occur due to flow of ir in narrow bronchi.

RQ-17/18

1. Hypoglycemia signs except- 2. Shock sign except- 3. Pic of Healthy pt having -tongue lesion also on palate - candidiasis 4. For saliva control if pt given Anaesthesia- glycopyrrolate (in options) 5. Ace inhibitors - moa 6. Beta blockers- moa in angina kkkk 7. Propanalol + epi- which receptors work 8. Vasoconstriction by which receptors 9. Pics- fibroma vs neuroma,geographic tongue, median rhomboid glossitis 10. Digoxin+ diuretic 11. Hypertension stages

Combined remembered Qs 2020

12. Pt is havi ng all crowns n implants - she is confused hw to take oral hygiene-ask her to watch vedio on oral hygiene, give chlorhexidine mouth wash ​ 13. Dual bond 14. biting - how to correct it - grind the lower buccal 15. Beading on major connector -to avoid food impaction and retention 16. Tongue tie - pt with denture what problem she will face - dislodgement of denture 17. A lot of patient management 18. popular licensure question 19. wine 2 glasses rrrquestion 20. doctor angry with pt-but put it on assistant 21. 2 ethics - when pt ask for extract all teeth to give CD but he is having good teeth - 22. Smoker pt what not to give for smoking cessation - smokeless tobacco 23. Many Endo vs perio diagnosis (should be so clear about this) 24. Diagnosis suppurative periodontitis 25. To diagnose class 1,2,3 not from pics of molars but from lat ceph 梁路♀ but they do ​ ​​ particular ask about angle classification without molar pictures 26. Transillumination 27. How to change hue 28. Histamine cause except 29. If pt very anxious - what to give a day before when he is coming in for treatment 30. Pt is taking so many medication- TCA,smoking cessation-what will affect oral hygiene - xerostomia 31. Pt is interested in smoking cessation- how will u help except -some motivational interview n weird options 32. 16 yr old pt- lost per 1st molar - how to replace -Rpd, Fpd, implant ​ ​ 33. Fluoride supplements at what age start 34. Pregnancy pain killer safe acetaminophen ​ 35. Turner hypoplasia 36. Randomized clinical trial - to reduce bias 37. Aspirin toxicity 38. EDTA - chelating agent ​ 39. Some RQ from strawberry file,elmastro file, I don’t remember question but I have seen them in some files.

Combined remembered Qs 2020

40. Be clear with your basics you gonna need them in exam no memorizing the RQ,options r different n complicated so try to find the correct ans n read that topic in detail 41. Day -2 - pt with so many different medications even you haven’t heard some of them but they gonna ask u questions related to the main ones 42. Pharma- mostly moa of drugs n some basic interactions 43. On day 2 try to be as conservative in treatment as possible but in some question they gonna ask you best treatment - diastema t/t- crown, veneer,composite 44. Some pt management even in day 2 45. Pt ask not to tell my family about cancer 46. Basic signs n symptoms-Asthma, hypertension, hypoglycemia, shock , penicillin allergy 47. Pt is hypoglycemia, 1st thing to check if he is unconscious 48. Asthma pt - what’s important -has to get asthma inhaler 49. What the pt can have if she is using beclomethasone Inhaler - candidiasis

DISHA’S – 22 AND 23 JANUARY

1. Combination syndrome features - tricky words.. I almost thought nothing matches 2. Mucocele - due to trauma 3. Mucous retention cyst - due to plugs 4. Exostosis and Enostosis 5. Subman infection spreads to mediastinum 6. Congenital epulis - there was some description and only this matched partly .. again one of the question where I thought nothing matches. 7. Diff between primary herpetic stomatitis and recurrent herpes .. there were pictures to identify 8. LA calculation but units in cc ( know it.. cc is equal to ml) .. but the calculation was very weird .. I had to choose LA amount and epi amount.. nothing in LA amount matched but in epi it did so I just chose that option This table really helped for the epi calculation. 9. When does teeth start showing tetracycline stains ? Prenatal, at birth , 3 years , 6 years

Combined remembered Qs 2020

1. Burs used for different finish lines. Chamber. - round end tapered 2. Description of a dark blue lesion - cavernous hemangioma or amalgam tattoo 3. Location of pier abutment 4. Minimum clearance for occlusal rest 5. Difference between crystalline and amorphous .. their melting points atomic range 6. Description of a lesion (except question) - I chose verruca vulgaris out of herpes, pemphigus, and pemphigoid

7. Which pulp horn is exposed first of primary man molar ? Mesial of 1, distal of 1, mesial of ​ ​ 2, distal of 2 8. Emergency drug for bronchospasm and something else.. I think we discussed epi on the group 9. Stannous floured mouthwash for medically compromised patients 10.10% chor varnish best use 11.2 - 3 questions on endocarditis prophy - needed in aortic valve replacement and the dosage and all 12.Pictures and description on granular cell tumor, lymphangioma 13.Autoimmune dieases most common in which demographics 14.Targetpoid lesions 15.Topical agent for Perio disease 16.Rheumatoid arthritis - oral manifestations

17.Pierre robin syndrome 18.Taurodontism - which developmental stage 19.Cafe au lait spots

Combined remembered Qs 2020

20.Mccune albright syndrome 21.Weird radiograph.. none of the options matched again 22.Cetirizine side effects 23.ALARA principle application on a patient based radio question 24.Keneddys classification identification on a cast 25.Peripheral cemento osseos dysplasia 26.Peripheral ossifying fibroma 27.Peripheral giant cell granuloma 28.Propanolol alters taste sensation 29.Radiograph with PSA seen in max sinus 30.Alzeimers medications MOA 31.Warfarin therapy stop before 2 days

32.Metabolic disorder except question 33.Perio prognosis is better for pulp necrosis leading to Perio or Perio disease leading to pulp necrosis 34.Hematoma vs hemangioma - which one can be blanched .. more on such red lesions 35.Effective only against anerobes - metronidazole 36.Contents of feldspathic .. 37.lots of questions on components of dental materials 38.Ingredients in chemicals used 39.Lots of questions on their chemical reactions .. I honestly don’t remember because I didn’t understand the questions .. too long questions and options 40.Cement used in porcelain veneer resin 41.Radiograph showing radiopaque wavy lines - resin 42.Question on thyroid therapy - ans was radioactive iodine

43.Question on ectodermal dysplasia but the option given was ectodermal hyperplasia .. so I got confused 44.Treatment of OKC but it wasn’t asked directly,.. I had to find from the description that it was OKC and then choose a treatment 45.Odontodysplasia 46.Endo questions on root perforation, pain on biting and

Combined remembered Qs 2020

root fractures 47.Loss of attachment in a given case due to vertical root fracture 48. Day 2 has typical questions on calculating missing number of teeth in a mixed dentition pano 49.Lots of prostho cases.. 50.Picture of inflamed minor salivary glands due to smoking 51.Precomtemplation question on smoking but very twisted words

52.Controlled substances ACT contains which drugs (except question) 53.Drug addict person - pain relief medication 54. Loss of attachment definition 55.Indirect pulp therapy definition 56.Pharmac questions on day one were very confusing. It was about a lot of drug reactions.. pharmacokinetics and pharmacodynamics.. I did mental dental vides, tufts pharma, el maestro,, all the other files and possible sources including DD .. but they didn’t have any of these concepts.. lot of detailed question on microsomal and non microsomal enzymes ,, their oxidation, conjugation and all 57.Day 2 pharmacy was easy and straight forward 58. Anatomy questions from part 1 59. Host modulation - doxycycline 60. PMMA reaction basics

61. lOts of questions on the basic actions for alpha and beta receptor . Know it well Jan 26

1. Jan rq: very similar to this picture. Which sinus is not shown clearly? ethmoid ​Which ​ facial profile? Straight, convex, concave

Combined remembered Qs 2020

2. Jan rq: similar to the picture, the large lesion is in the middle of the central incisor, not ​ near the cervical. Abrasion, acidic chemical (erosion is not in the choices ​ ​ 3. got a patient who currently prescribed penicillin V for pain. He has a heart condition and need prophylaxis before procedure begin. which prop hylaxis should you use? Amoxicillin 2g or clindamycin 600mg, erythromycin

4. How many cartridge of LA 2% lidocaine u can give to child 45 lbs ​1. 1 ​2. 3 ​3. 6 ​4. ​ ​ ​ ​ 9 ​Ans 2 ( 3 cartridge ) ​

5. Patient has a deep carious, non-restorable #23 and the dentist recommended extraction. No medical history. The patient’s spouse told the dentist that the patient has frequent urination, thirst, hunger and lost a lot of weight recently. What is the recommendation for the treatment? Extract #23 with local anesthesia, refer to the periodontist, refer to the physician

6. You give an IA nerve block and the needle inserted medially from the pterygo raphe. Which two muscles involved? medial pteryroid, superior constricture, buccinators

7. How many % of adults have herpes simplex virus? 20-30%, 50-80%, 100%

8. sterilize plastic instruments using? glutaraldehyde, phenol, alcohol...

9. Amantadine is an? antivirus

10. Coronal caries effect least on which tooth? Max molar, mand molar, max central, man central 11. Coronal cares affect most on which tooth? Max molar, mand molar, max premolar, man premolar

12. children with congenital heart defect seem emotionally and physically dependent because? limit physical abilities..., parents overly protective, hospitalized frequently...

Combined remembered Qs 2020

13. A tooth was root canal treated and there was a small radiolucency under the apex. After 4 years it is still there. What do you think it is? Apical scar, apical cyst, forgot other choices

14. Which experimental design would you use to study gastric bypass and nutrition? Cohord, randomized clinical trial, observational

15. calcific metamorphosis ? internal root resorption, external root resorption, something else

16. Which facial space does infection of Lugwig NOT travel to? Submental, Sub mandibular, sublingual, retropharyngeal

17. optimal periodontal maintenance is in ? 1 month, 3 month, 6, 12 months

18. a pt has deep pocket depth, which procedure would preserve the gingiva? split palatal, apical positioned flap, lateral position flap, gingivectomy

19. contraindication of root canal therapy? HIV, leukemia, recent MI

20. You observed the a child was very upset going to the dentist due to a past experience at another medical office. What it is? Association, modeling..

21. primary stability of implant osteo integration is? Patient health status, primary stability of the implant

22. maxillary artery is from which branch? internal carotid, external cortical artery

23. common sign of mandibular fracture? trismus, parasthesia

24. Development of cleft lip? 2-3weeks, 6-9weeks, 12-15 weeks, 18-21 weeks

25. characteristics of body dysmorphic disorder?

26. symptoms of thyroid storm versus anaphylaxic shocks

Combined remembered Qs 2020

according to fb: Thyroid storm-high temperature, fast heart rate, nausea, vomiting, confusion, perspiration, diarrhea Anaphylaxis-low BP, difficulty breathing, fast heart rate

27. If you think the drug causes serious problem, should report to? FDA, CDA

28. Which muscle closes the tongue? Genioglossus, hypoglossal ...

29. Common fungal infection? Histoplasmosis, candidiasis, blastomatosis

30. Lesion at the basement membrane? Pemphigoid, pemphigoid, erosion lichen planus

31. Reduction of the crown should look like? Flat and round, follow occlusal morphology

32. As you get older, people lose which teeth first due to periodontitis? Max molar, Man molar

33. Bulimia? Caucasian women, Caucasian men, black men, black women

34. Heparin? PTT, PT

35. Naloxone: completive antagonist? Morphine?

36. Provisional for veneer? Bis-cryl, methyl methacrylc, composite, Shell

FEBRUARY RQ

1. epiphyseal plate? Synchondrosis

2. Implant to implant? 3 mm

3. Dentin binding and 2 canals?

Combined remembered Qs 2020

Fusion Germination (only has 1 canal)

4. C.I. For RcT? Vertical roots fracture

5. CEJ apical to crest? 2 mm If they talk about emergence profile it should be 2-4 mm

6. Biological width 2mm CTand JE

7. Xray denture not fitting? Pageants ds

8. Tooth brush abrasions

9. tooth loading?

Abfraction

10.Gingival hemorrhages and skin lesion? Leukemia

11.Pt Gagging?

Inc VDO *( causes of gagging : increase of vdo - causes inadequate post extention ​ and food to be trapped, bulkiness of denture Hh palatal seal ( increase deph of posterior palatal seal cause unsealing of the denture) Buccal flange

12.Difficult to maintain space in early loss ? 2 molar* ​ ​ 1 molar

13.Sensitivity in test

Combined remembered Qs 2020

14.Dose epi 1.7 Ml 1:5000? 0.036 mg or 36 mcg

15.Drug run report to FDA ​

16.Impacted tooth maxillary Canine

17. Alv. Osteitis most common? Mand. 3molar

18.Initiation of caries? Strep mutants

19. Progression of caries? Lactobacllus

20.Frequent urine in 3 trimester? Fetus pressing on bladder

21.Asthma? Albuterol or Surbetol (Ventolin) opening passage for asthma 22.Morphine antagonist? Naloxene 23.Benzodiazepine. Antagonist: Flumezenil ​ 24.Tongue blue lesion? Blue mass on the tongue indicates the presence of : Lymphangioma Hemangioma Neurofibroma Pyogenic Granuloma Chronic Inflammatory gingival hyperplasia (epulis) Squamous cell carcinoma

25.Papilloma pic

26. Vercuous carcinoma pic

Combined remembered Qs 2020

27. Pseudomemb colitis? Clindamycin due to over proliferation of Clostridium Difficile Bacteria ​

28. Tetracycline not with? Penicillin for example ( don’t mix bactericidal with bacteriostatic) Make the list of cidal and static that we cant mix CAMP FV Celphalo Aminoglycosides Metro Penicillin Fluoroquin vancomycin

29.Lisinopril ? Lisinorol - angioten inhibitor used for hypertention (Ace – angiotensin converting enzyme inhibitors)

ACE inhibitors, “inhibit” the conversion of inactive Angiotensin I→Angiotensin II (a vasoconstrictor). ​STIMULATE THE RELEASE OF ALDOSTERON ​

30.Chronic periodontitis?

Black male

31.Oral cancer poor prognosis? Black male

32.Antibiotic and surgical therapy? ANUG, ( just do debriment) LAP * 33.Implant most retentive in? Anterior mandible* ( bone more dense ) rule : more in Mand than max , more in ​ anterior than post

Combined remembered Qs 2020

Posterior mandible

34.Skin nodules and mouth pigmentation? Neurofibromatosis

35.Common in Gardner and pouts Jeghers? Polyps

36.Mucus retention on ? Trauma* Mucus plug (choose this one if trauma is not there) Stone

37.Class 2 chances in permanent? Distal step in primary* (remember: the teps always stays the same or move a little mesially) ​ ​

Mesial step

38. ? Class 3

39.Nitroglycerin ? Angina

40.Not required in angina pt? Antibiotic prophylaxis ​

41.Attachment loss? CEJ to base of pocket ​

The periodontal examination includes probing pocket depth (distance from the to the base of the pocket) and clinical attachment level (distance from the CEJ to the base of the pocket).

42.Sturgeon Weber? Port wine ​

Combined remembered Qs 2020

43.Root mand 3 molar lost into? Sub mandibular ​

44.Herpes gingivostomatitis? 2 years ​

45.Non malifience? Referring and knowledge updating

46.Splint implant 4 47.Amalgam pin

48..m , 7Horizontal root fracture ?

Splint and observe ( 3 PA varying vertical position) If coronal (rigid splint - 3 months) If ½ root (flexible – 3 weeks) If apical (flexible 2 weeks) Chance of necrosis 25 %

49.3 year old central incisor intrusion? Radiograph and antibiotics* ( just x -ray, if not close the permanent, leave it there, no need ​ for antibiotic) Exo

50.Hypodontia a defect on which stage?

Iniciation – anodontia or supranumerary Bud Stage – Hyper or Hypodontia* Cap Stage – Dens in dente, gemination, fusion and tubercules Bell stage – dentinogenesis and amelogenesis imperfecta, macro and microdonty Appositional _ enamel dysplasia, concrescence, enamel pearl

51.2 cm white patch on sub mandibular duct? Incisional biopsy (more than 1 cm , make incisional) the only white patch that u make ​ cytology or brush biopsy is pseudo membranousn

Combined remembered Qs 2020

52.0.5 cm? Excisional biopsy ​

53.best test for pdl inflammation: Percussion

54.Buccal Roots max molar more mesial to palatal, position of tube Mesial * Distal is the answer Inferior Superior

55.Cells on tension side Osteoblasts Clasts * (seen on the compression site) Cyte PMN

56.Kid after active ortho treatment, retention by (history of poor oral hygiene) Fixed Removable * ​ Other 2 options

57.Lesion not on tongue

Peripheral granuloma * – periferial giant cell granuloma? – limited to alveolar ridge ​ ​ anterior to first molar Ectopic thyreoid 2 more that I don’t remember

58.Osteoporosis Radiographically thin trabecular * less osteoid calcification other 2 options

59.Palatial root more mesial to buccal root, porition of tube

Combined remembered Qs 2020

Mesial * slob rule Distal is the answer Superior Inferior

60.Not a method to check sterilization Chemical ( for instruments that cant be heat sterile) Mechanical Biological (most common used) Electrical *

61.DKA Hypoglycaemia Hyperglycaemia Hyperglycemia * - diabetes ketoacidosis ​ 1 other options

62.Periapical cementoblastoma Is a relatively rare benign neoplasm of the of the teeth. It is derived from ectomesenchyme of odontogenic origin. Less than 0.69-8% of all tumors of the teeth Osseous dysplasia

63.Gingivectomy indications Suprabony indicated only in keratinized area

64.Carcinoma in situ.

Erythroplakia Leukoplakia white sponge nevus lichen planus

C in S: A group of abnormal cells that are found only in the place where they first formed in the body (see left panel). These abnormail cells may become cancer and spread to nearby normail tissue (see right panel)Lateral Periodontal Cyst

Combined remembered Qs 2020

65.Safer drug in HIV Erythromycin Amoxicilin Ibuprofen *

66.Class 3, 4, 5 composite bevel 15 30 45 * 60

67.Swelling of lip Mucus retentioncysts Minos SG something Sialolith

68.Swelling of lit after anaesthesia injection Angioedema * Anaphylaxia other 2 options

69.Nitrous Oxide And epinephrine antagonize by ​ Physiologic * competitive 2 other options

70.Lots of questions on finishing lines Axio gingival – Class 1 - 2 Axio pulpal – class 5 Grooves – proximal

71.Hemorrhagic shock first sign Tachycardia * Hypotention other options

Combined remembered Qs 2020

72.Epinephrine works on Alpha 1 * Alpha 2 Beta 1 Beta 2

73.3 questions about DKA diabetes ketoacidosis Lets search for everything about this

Tino Feb 7,8-2020 RQs (216 Qs)

1.QUESTION: Case: Patient with tooth that has sensitivity that lingers with thermal test, sinus tract, and positive to ​ ​ percussion, what does the patient have? Irreversible pulpitis with acute periapical abscess (other choices were Irreversible pulpitis with no acute ​ ​ periapical abscess, and 2 other choices with reversible pulpitis in them).

2.QUESTION: Chronic periradicular abscess indicates: necrotic Pulp ​ ​ Percussion- presence of inflammation in PDL or not Palpation- spread of inflammation to from PDL or nothing EPT- Pulp vitality, responsiveness (necrosis or not) Thermal test (hot & cold)- pulp vitality. Hot (irrev), cold (rev)

3 QUESTION: Which is incorrect? Do EPT for traumatic tooth ​ ​ 4 QUESTION: When heat is applied to the tooth, lingering pain for several minutes indicates: irreversible pulpitis ​ ​ 5 QUESTION: What is diagnosis for lingering pain to cold and sensitivity to percussion? Irreversible pulpitis & acute ​ ​ periapical abscess - Usually periodontal abscess is sensitive to percussion, irreversible is usually positive to percussion

6 QUESTION: What is test to diagnose chronic periradicular periodontitis? Percussion ​ 7 QUESTION: EPT does NOT indicate health of the Pulp, only vitality ​ ​ 8 QUESTION: How do you differentiate between an endo/perio lesion? EPT ​ ​

Combined remembered Qs 2020

9 QUESTION: Chronic endodontic lesion has what type of bacteria? Anaerobes ​ ​ 10 QUESTION: What is initial treatment of a combination perio and endo lesion? Perform endo with RCT first, then perio ​ ​ ​ Sc/RP

11 QUESTION: What treatment is required with a tooth that has a draining sinus tract has been treated via RCT? No ​ further treatment

12 QUESTION: A patient has a non-vital tooth & a fistula that is draining around the gingival sulcus. What kind of abcess is ​ it? endo and perio at same time perio and then endo only endo only perio v ,

13 QUESTION: There usually is no lesion apparent radiographically in acute apical periodontitis. However, histologically ​ bone destruction has been noted. a. Both statements are true b. Both statements are false. c. First statement is true, second is false. d. First statement is false, second is true.

14 QUESTION: Least reliable test on primary teeth - Electric pulp test ​ ​ - On primary teeth you don’t want to use EPT b/c thin enamel creates false results & after trauma, you don't want to use ​ electronic pulp tester.

15 QUESTION: 7 yr old boy has vital pulp exposure of 1st perm max molar. What do you do for treatment? Pulpotomy ​ ​ ​ ​ 16 QUESTION: A 7-year-old patient fractured the right central incisor 3 hours ago. A clinical examination reveals a 2-mm ​ exposure of a "bleeding pulp." The treatment-of-choice is A. pulpectomy and apexification. B. pulpotomy with calcium hydroxide. ​ C. direct pulp cap with calcium hydroxide. D. one-appointment root canal treatment.

17 QUESTION: Why would you do a pulpotomy in a mandibular first molar of a 7-year-old? To continue physiologic root ​ development (apexogenesis)

Splint the tooth is for pt comfort - Avulsion: 7-10 days non-rigid/flexibile splint, antibiotics ​ - Horizontal root fractures: Rigid splint, 3 months - Extrusion: 2-3 weeks splint

18 QUESTION: Reason for failure of replantation of avulsed tooth: external resorption or internal resorption ​ ​ ​ ​ 19 QUESTION: Which is more damaging to the PDL? Extrusion, intrusion, lateral luxation, avulsión ​ ​ ​

Combined remembered Qs 2020

20 QUESTION: Luxated tooth, negative EPT, why? disruption of nerves to tooth ​ ​ 21 QUESTION: Sodium hypochlorite is used for everything except? Chelation ​ ​ - Bleach is not a chelating agent ​ 22 QUESTION: Which is a chelator/chelating agent for endo? EDTA, sodium hypochlorite, etc. ​ ​ ​ - EDTA is chelator, removes SMEAR LAYER and inorganic material. ​ - NaOCl (sodium hypochlorite) only dissolves organic material, only disinfects & is most common irrigant. ​ 23 QUESTION: Treatment for internal resorption: RCT ​ ​ 24 QUESTI ON: When a tooth is ankylosed, what type of resorption? replacement resorption ​ ​ 26 QUESTION: Incomplete removal of bacteria, pulp debris, and dentinal shavings is commonly caused by failure to ​ irrigate thoroughly. Another reason is failure to: A. use broaches. B. use a chelating agent. C. obtain a straight line access. D. use Gates-Glidden burs.

27 QUESTION: You separate an endo file 3mm from the apex and obturate above it... which case will show the best ​ prognosis? a. vital pulp w/ no periapical lesion (yes) b. vital pulp wI periapical lesion c. necrotic pulp wI no periapical lesion d. necrotic pulp wI periapical lesión

28 QUESTION: Pt has crown cemented 2 weeks ago & is sensitive to pressure and cold, why? Occlusal trauma ​ ​ 29 QUESTION: Crack tooth syndrome is most likely found? Mandibular Molars ​ ​ 30 QUESTION: Cracked tooth with no pulpal involvement, what is the treatment? Endo, extracoronal restoration, ​ ​ occlusion reduction, amalgam with adhesive

31 QUESTION: pH that enamel starts to demineralize – pH = 5.5 (critical pH of developing cavities) ​ ​ ​ 32 QUESTION: Most Cariogenic? Sucrose ​ ​ ​ - S.mutans adheres to the biofilm on the tooth by converting sucrose into an extremely adhesive substance called dextran ​

33 QUESTION: Where does caries start? Apical to proximal contact. ​ ​ 34 QUESTION: 40 y pt w/ all 32 teeth. No cavities. Has stain & catch in pit of molar. what do you do? ​ a. Watch & observe ​ b. sealant ​ c. composite ​ 35 QUESTION: Main difference and advantage of using GMT instead of Enamel hatchet? ​ a. bi-angled cutting surface b. angle of the blade c. push/pull action instead of

Combined remembered Qs 2020

36 QUESTION: What would cause displacement of odontoblastic processes? ​ Thermal Dessication Mechanical Chemical

37 QUESTION: What is the most important thing for retention? Surface área ​ ​ 38 QUESTION: Most lab complain that the tooth is under reduced. ​ ​ 39 Not Sure answer: QUESTION: What causes the most retention of crown? Axial taper, surface area, surface roughness, ​ ​ ​ retention grooves

40 QUESTION: Which of the following do you not do in cementation of a porcelain crown: etch enamel with hydrofluoric ​ ​ acid

41 QUESTION: What could the reason be if you see opaque white porcelain in the incisal 1/3 facial of the PFM crown: ​ Inadequate reduction of the inciso facial part of the tooth

42 QUESTION: What is worse outcome of nonvital bleaching (internal bleach for endo)? internal root resorption ​ ​ /CERVICAL RESORPTION.

43 QUESTION: How long after vital tooth bleaching can you bond resin to it? 24 hours, 3 days, 1 week ​ ​ 44 QUESTION: Pontic of 3-unit FPD should rest gently on the soft tissue & should not blanch tissues. ​ ​ ​ ​ 45 QUESTION: Strength of abutment connection to pontic which is more important? occlusogingival width ​ ​ 46 QUESTION: Which represents position on the spectral wavelength? Hue ​ ​ 47 QUESTION: What kind of occlusion if in right lateral movement all posterior teeth are not in occlusion: canine ​ ​ ​ ​ guidance

48 QUESTION: Best imaging for TMD (soft tissue, disc & condyle of TMJ): MRI ​ ​ 49 QUESTION: What causes TMJ ankyloses? Trauma, Rheumatoid arthritis ​ ​ ​ 50 QUESTION: Trismus includes what muscle? Medial pterygoid ​ ​ 51 QUESTION: What type of bond is composite on tooth structure? ​ a. chemical bond b. mechanical bond (micromechanical) c. organic coupling d. adhesión

52 QUESTION: What do you place on a 75 y/o patient with ~ 8 class V carious lesions? GI ​ ​ ​ ​ 53 QUESTION: 65 y/o pt shows several new caries in molars and premolars class V, what material would you use: ​ a) amalgam b) composites) c) glass ionomer ​ 54 QUESTION: Patient’s chief complaint is #8 and #9 don’t look right. Picture shows nothing is wrong with #9. #8 has extra ​ enamel at the incisaldistal

Combined remembered Qs 2020

aspect. What do you do? Shave the inciso-distal aspect of #8. ​ ​ 55 QUESTION: Photo initiator of resin composite? Camphoroquinone ​ ​ 56 QUESTION: Etch removes the smear layer & exposed collagen fibers to form hybrid layer with resin ​ 57 QUESTION: Which indicated for high caries risk or multiple class Vs? Glass Ionomer ​ ​ 58 QUESTION: What happens when you take an impression & lip immediately swells? Angioedema (allergy reaction) ​ ​ ​ 59 QUESTION: Which is not recommended for final FPD cast impression? ​ • irreversible hydrocolloid • reversible hydrocolloid • PVS • Polyether

60 QUESTION: Most rigid impression material: Polyether ​ ​ 61 QUESTION: What ion gets replaced in hydroxyapatite by fluoride? Hydroxyl ​ ​ 62 QUESTION: What’s the concentration of acidulated phosphate fluoride is used in the dental office? 1.23% ​ ​ 63 QUESTION: Which space is not involved/associated with Ludwig's angina? ​ Sublingual Submandibular Retropharyngeal Submental ​ 64 QUESTION: Turner’s tooth is caused by: trauma or local infection ​ ​ 65 QUESTION: HIV patient with oropharyngeal candidiasis, what would you prescribe? Fluconazole ​ ​ 66 QUESTION: Healthy 36-year-old, red patch on palate, redness in middle of tongue: ​ - Kaposi sarcoma - Syphilis - Median rhomboid glossitis à Candidiasis ​ ​ ​ - Gonorrhea

67 QUESTION: 85% of people have herpes ​ ​ - 65-90% worldwide; 80-85% USA

68 QUESTION: Patient gets recurrent herpetic lesions very often with gingivostomatitis. What should be done? (herpetic ​ gingivostomatitis) Acyclovir ( recurrent is by Acyclovir , you give acyclovir 3 days before the symptoms and palliative is during the symptoms) Palliative tx Systemic antibiotics Steroids

69 QUESTION: Lesion in lip with cauliflower shape: Papilloma ​ ​ 70 QUESTION: Which one resembles Epilus Fissuratum – Fibroma (both share trauma as etiology) ​ ​ ​ 71 QUESTION: If you have leukoplakia for biopsy, do you incise or excise for biopsy? Incision ​ ​

Combined remembered Qs 2020

- incise multiple areas w incisional biopsy ​ 72 QUESTION: Chewing Betel nut can lead to à SCC, xerostomia, gingival recession ​ ​ ​ ​ 73 QUESTION: Warthin tumor is most common in what gland? Parotid (don’t get mixed up with Wharton’s duct) ​ ​ ​ 74 QUESTION: Which one can lead to ameloblastoma? Dentigerous Cyst ​ ​ 75 QUESTION: Radiographic picture: upside down molar with lucency around crown, what is it? ​ Dentigerous cyst

76 QUESTION: Amelogenesis imperfecta is autosomal dominant. ​ ​ ​ Dentiogenesis Imperfecta’’’’: Crowns are short & bulbous, narrow roots, obliterated Pulp ​ 77 QUESTION: X-ray: Dentiogenesis Imperfecta – obliterated pulp chamber ​ 78 QUESTION: All of the following are differential diagnosis for Dentinogensis imperfecta except? ​ ectodermal dysplasia amelogenesis imperfectat enamel dysplasia dentinal dysplasia enamel hypoplasia (AI)

79 QUESTION: Radiographs of a patient's teeth reveal that the crowns are bulbous; the pulps, obliterated; and the roots, ​ shortened. These findings are associated with which of the following? Porphyria Pierre Robin syndrome Amelogenesis imperfecta Osteogenesis imperfecta Erythroblastosis fetalis

80 QUESTION: 12 y/o boy’s X-ray shows roots are short & open apex. Sister also has same condition. What condition is ​ this? DI - autosomal dominant AI - autosomal recessive Dentin dysplasia – autosomal dominant

81 QUESTION: Ectodermal dysplasia: which of the following is correct? It is X-linked, not autosomal dominant ​ ​ 82 QUESTION: Characteristic of Ectodermal Dysplasia is? Oligodontia (some missing teeth, > 6 teeth, not all teeth) and ​ ​ ​ hypohidrotic (reduced sweating) or anhidrosis (lack of sweating)

83 QUESTION: Not a bone cyst? Nasolabial cyst b/c it occurs outside of bone & is a soft-tissue cyst ​ ​ ​ 84 QUESTION: Most common location for mucocele? Lower lip ​ ​ ​ 85 QUESTION: Sialoliths are most common in what gland? Submandibular gland & duct ​ ​ Parulis = localized collection of pus in gingival soft tissue. Pus is produced as a result of necrosis of non-vital pulp ​ tissue or occlusion of a deep periodontal pocket.

Combined remembered Qs 2020

86 QUESTION: Neurofibromatosis clinical presentations: Cafe au lait, lisch nodules of the iris ​ 87 QUESTION: Dens in dente are most commonly seen in maxillary lateral incisor. ​ ​ 88 QUESTION: Sjogren’s syndrome: destruction of salivary and tear ductsà dry mouth ​ 89 QUESTION: The mucosa of the is the usual intraoral site for which of the following conditions? ​ - Mucocele - Sialolithiasis - Minor aphthous ulcer - Major aphthous ulcer - Necrotizing sialometaplasia

90 QUESTION: What is primary source of radiation to the operator when taking x-rays? ​ radiation left in the air scatter from the patient scatter from the walls leakage from the x-ray head

91 QUESTION: MRI uses what electromagnetic wave? RADIOWAVES ​ ​ 92 QUESTION: What does collimation do? reduces x-ray beam size/diameter & volume of irradiated tissue, reduces area ​ ​ of exposure - usually with circle diameter of 2.75 in

93 QUESTION: Collimation control of size & shape of x-ray beam ​ ​ 94 QUESTION: X-ray tube target metal is made out of: tungsten (target = tungsten/filter = aluminum) ​ ​ ​ 95 QUESTION: Overlap on bitewings due to horizontal angulation ​ ​ 96 QUESTIO N: What does it look like on a pano when your patient moves during the pano? A vertical blur line vs ​ horizontal defect.

97 QUESTION: If you have lesion of maxillary sinus, what kind of radiograph do you take? Waters ​ ​

98 QUESTION: If change from 8 mm cone to 16 mm, how much exposure time do you need to increase by? 2, 4, 6, 8 ​ ​ ​ - Remember that going from an 8 mm to 16 mm cone means the cone/target is LONGER. This is the PID (target to film distance). If the PID is increased there is LESS magnification. If the PID is shorter there is MORE magnification. Also density (darker x-ray) increases when kA, mA and exposure are increased.

99 QUESTION: What is most radio-resistant cell: Muscle ​ 100 QUESTION: What is the mechanism of action of bisphosphonates? Inhibit osteoclasts ​ ​ 101 QUESTION: Pt taking bisphosphonates for 1 yr. IV, highest risk during dental tx? Osteonecrosis ​ ​

Combined remembered Qs 2020

102 QUESTION: Pt has sickle cell anemia & has a thrombolytic crisis, what could precipitate this? ​ a. Nitrous oxide / oxygen use b. Cold c. Trauma d. Infection - Sickle cell anemia is seen exclusively in black patients. Periods of unusual stress or of O2 deficiency (hypoxia) can ​ ​ precipitate a sickle cell crisis.

103 QUESTION: Pt is taking warfarin (Coumadin), what test do you run prior to extraction or surgery: INR (= 2.0-3.0) ​ ​ ​ 104 QUESTION: Pt taking ginseng. Which med should be avoided? ​ • Penicillin • Aspirin • Digitoxin

104 QUESTION The drug contraindicated in pt taking gingko biloba: HEPARIN ​ ​ 105 QUESTION: Proposed modes of action for the oral antidiabetic agents include each of the following EXCEPT one. ​ Which one is the EXCEPTION? A. Blockade of glucagon release from pancreas B. Block?ade of catecholamine release from adrenal medulla C. Stimulation of insulin release from pancreatic beta cells D. Action as direct receptor agonists for the insulin receptor E. Increase affinity of tissues for utilization of available plasma glucosa

106 QUESTION: Patient with orthopnea (shortness of breath-dyspnea-while lying flat), dyspnea, pedal edema ​ a. Emphysema b. Pulmonary edema c. COPD d. Congestive heart failure

107 QUESTION: Pt has history of cardiovascular disease and now, pt is taking aspirin. Pt needs ext. What should dentist ​ do? • Med consult with physician • Normal extraction • Stop aspirin 3 days before and 2 days after surgery

108 QUESTION: Side effect of nitroglycerin: orthostatic hypotension and headache ​ ​ 109 QUESTION: Epi and Nitroglycerine: antagonist ​ ​ 110 QUESTION: Child makes a wheezing sound before injection? Asthma (induced by stress) ​ ​ ​ 111 QUESTION: Which of the following drugs is can trigger asthma? ​ a) narcotic analgesic b) NSAID ​ c) corticosteroid d) sympatolytic amine.

112 QUESTION: Patient begins to wheeze, what do you not do? ​ o Beta-2 blocker inhaler ​

Combined remembered Qs 2020

o sit pt up & make them more comfortable ​ o corticosteroid inhaler ​ o Give oxygen ​ 113 QUESTION: Pregnant in supine position, what gets too much pressure? ​ Fetus Placenta Inferior Vena Cava Superior Vena Cava

114 QUESTION: Most common dental complication/emergency in office? Syncope ​ ​ 115 QUESTION: After receiving one cartridge of a local anesthetic, a healthy adult patient became unconscious in the ​ dental chair. The occurrence of a brief convulsion is A. pathognomonic of grand mal epilepsy. B. consistent with a diagnosis of syncope. C. usually caused by the epinephrine in the local anesthetic. D. pathognomonic of intravascular injection of a local anesthetic.

Benzodiazepines: ​ - MOA – modulate activity of inhibitory NT (GABA) receptor

Grand mal seizure: Phenytoin (Dilantin) Status epilepticus: Valium (diazepam)

116 QUESTION: Which of these is indicated for grand mal seizure? DILANTIN (phenytoin) ​ ​ ​ 117 QUESTION: Most common seizure in children – Febrile seizure ​ ​ 118 QUESTION: Which of the following drugs, when administered intravenously, is LEAST likely to produce respiratory ​ depression? A. Fentanyl B. Diazepam ​ C. Thiopental D. Meperidine E. Pentobarbital

119 QUESTION: What’s the action of the Benzodiazepines? Facilitates GABA receptor binding by Increasing the ​ ​ ​ frequency of chloride channel opening.

120 QUESTION: Which drug best reverses the effect of benzodiazepines? Flumazenil ​ ​ ​ - Flumazenil: Benzodiazepine antagonist b/c competitive GABA receptor.

Anti-Histamines: competitive histamine receptor blockers ​ 121 QUESTION: How does antihistamines work? Competitive inhibition of histamine receptors ​ ​ ​ ​ 122 QUESTION: Which of these opioid analgesics is associated with a serious life threatening drug interaction when ​ administered with an MAO inhibitor? Meperidine (Pethidine, Demerol) ​

Combined remembered Qs 2020

morphine fentanyl propoxyphene codeine - Can cause life-threatening hyperpyrexia reactions (fever)

123 QUESTION: Opioid usage shows all except: xerostomia, chronic cough, diarrhea, miosis, constipation ​ ​ ​ ​ 124 QUESTION: Which of the following symptoms is the most distinct characteristic of morphine poisoning? ​ ​ A. Comatose sleep B. Pin-point pupils (miosis) ​ C. Depressed respiration D. Deep, rapid respiration E. Widely dilated, non-responsive pupils

125 QUESTION: Naloxone: use for Opioid overdose. ​ ​ ​ - Naloxone blocks or reverses the effects of opioid medication, including extreme drowsiness, slowed breathing, or loss of consciousness

126 QUESTION: Tylenol - can cause hepatotoxicity ​ ​ ​ 127 QUESTION: Pt has hepatic dysfunction; which pain medication can prescribe? ​ ​ a. Oxycodone b. naproxen c. acetaminophen

128 QUESTION: Which of the following does not have anti-inflammatory action? Acetaminophen ​ ​ ​ 129 QUESTION: NSAIDs – mech of action of suppressing platelets – inactivate cyclooxygenase à decreased prostaglandin ​ ​ ​ ​ synthesis

130 QUESTION: After one effective dose of aspirin, how long must you wait before there is not effect on bleeding time? 1 ​ ​ ​ week

131 QUESTION: Aspirin works on which pathway for pain? Cyclo-ox pathway ​ ​ ​ 132 QUESTION: Biopsy - indicated when treatment doesn’t work after 14-20 days ​ ​ - about 2 weeks—any red or white lesion that doesn’t resolve itself in two weeks

133 QUESTION: Oral candidiasis biopsy of choice is: ​ a. incisional biopsy b. excisional biopsy c. brush biopsy (collects the cells for cytological smear) d. cytologic smear ​ 134 QUESTION: White lesion is 2x3x2 cm, what type of biopsy? ​ excisional biopsy incisional biopsy smear

135 QUESTION: What kind of bacteria is under implants? At the apex of root canal? Gram (-) rods and filaments ​ ​ anaerobic

136 QUESTION: What is the least important factor when evaluating for implant? ​

Combined remembered Qs 2020

concavity of mandible bone density distance to mandibular cancel bone width

137 QUESTION: Minimum distance between adjacent implants? 3 mm ​ 138 QUESTION: How much space between implant and tooth? Answers were 1.5 mm, 2, 3.5 3, ​ ​ ​ 139 QUESTION: In anterior maxilla, for a 4mm diameter implant, how far apical to the CEJ of adjacent tooth for optimal ​ emergence profile? 1 mm above cej of adj tooth 1 mm below cej of adj tooth 2-4 mm below cej of adj tooth

140 QUESTION: Implants osteointergrate best in? posterior mandible ​ 141 QUESTION: What causes the greatest incidence of implant failure? ​ Smoking Osteoporosis with HTN Hypotension Allergy to antibiotics

142 QUESTION: Epithelial attachment for implant? ​ • Hemidesmosome* (epithelial attachment to tooth structure and implant are the same) ​ ​ • fibronectin

143 QUESTION: What speed and torque for implant is used? High Torque, slow speed ​ ​ 144 QUESTION: How to clean implant- prophy cup, plastic scalers, not stainless steel! ​ 145 QUESTION: You are considering the placement of an upper and lower implant-retained complete denture. How many ​ implants will you place in the anterior region? a. maxillary one and mandibular one b. maxillary two and mandibular two c. maxillary four and mandibular two d. maxillary four and mandibular six - If implant supported complete denture, add 2 more screws to each.

146 QUESTION: Which tooth is least likely to be missing? ​ ​ ​ Canine

Most commonly missing teeth are the 3rd molars, 2nd premolars and upper lateral incisors ​ ​ ​ ​ 147 QUESTION: Extraction of #30, which way do you section? Buccal- lingual ​ ​ 148 QUESTION: Where are you most likely to damage a nerve in vertical release of flap? lingual, Wharton’s duct and the ​ ​ ​ - void vertical incisions in lingual and palatal

149 QUESTION: When doing flap surgery on mandible, what structure do you watch for? mental nerve, mentalis ​ ​ ​ attachment

Combined remembered Qs 2020

150 QUESTION: What number forceps to use when extracting mand premolars: 151 A ​ ​ 151 QUESTION: Treatment of alveolar osteitis: placement of a palliative medicament/dressing. DON’T USE antibiotic. ​ ​ 152 QUESTION: Lefort I fracture are associated with? ​ nasoethmoidal air cell frontal sinus maxillary sinus mastoid air cell

153 QUESTION: A patient experiences numbness of the left upper lip, cheek, and the left side of the nose following a ​ fracture of his midface. This symptom follows a fracture through the A. nasal bone. B. zygomatic arch. C. maxillary sinus. D. infraorbital rim. ​ 154 QUESTION: Most common complication of sagittal osteotomy: IAN, loss of sensitivity ​ ​ ​ Red Complex – group of bacteria grouped together based on their association w/ periodontal disease ​ - red complex = P. gingivalis, Tannerella forsythia, treponema denticola - BOP & deep pockets Orange Complex - fusobacterium, prevotella, campylobacter ​ ​ - Precedes red complex, plaque formation & maturation

155 QUESTION: What is the 1st step in bacterial plaque formation on a tooth? Pellicle formation (glycoproteins, enzymes, ​ ​ ​ ​ ​ proteins, phosphoproteins). - 2nd step is adhesion and attachment of bacteria ​ ​ - 3rd step is colonialization and plaque maturation ​ ​ 156 QUESTION: Which one is not a periodontal risk factor? Smoking, oral hygiene, malnutrition, diabetic mellitus ​ ​ ​ ​ 157 QUESTION: What cytokine responsible for osteoclasts? IL-1, IL-8, IL-5, IL-3 ​ ​ ​ 158 QUESTION: If you have 1 mm recession and can probe 3 mm, how much attachment loss is there? 4mm. CAL= PPD + ​ ​ recession

159 QUESTION: Best brushing technique to clean periodontal pockets: ​ A. Charters B. Sulcular (another name for modified Bass) ​ C. Whitman’s

160 QUESTION: The role of chlorohexidine is cause: Substantivity (anti-plaque) ​ 161 QUESTION: Best time for supportive periodontal therapy? 1, 3, 6, 9, months post s/rp ​ ​ ​ 162 QUESTION: Initial tx for Localized aggressive periodontitis Sc/RP Antibiotics Sc/RP and Antibiotics

Combined remembered Qs 2020

Antibiotics for 1 week and then Sc/RP

163 QUESTION: First step in initiation treatment of HIV necrotizing ulcerative gingivitis? debridement and antibacterial ​ ​ rinse, antibiotics ​ ​ Gingivectomy

164 QUESTION: What’s the #1 cause of medication ​ induced gingival hyperplasia? Anti-convulsant meds Dilantin (30% of all drug induced) ​ ​ ​ 165 QUESTION: Indications for gingivectomy – hyperplastic gingiva & suprabony pockets ​ ​ 166 QUESTION: Gingivectomy is contraindicated with? Minimum attached gingiva ​ ​ ​ 167 QUESTION: How does a gingivectomy heal? Secondary intention ​ ​ 168 QUESTION: Decalcified freeze dried bone allograft: has bone morphogenetic proteins (BMP). BMP is a protein ​ ​ complex responsible for initiating osteoinduction (ability of molecules contained in the graft to convert neighboring cells into osteoblasts)

169 QUESTION: What is indicated for the tx of unilateral posterior cross bite? from lingual of max mol to Buccal ​ ​ of mand molar - A single too th cross bite can be adjusted by placing cross elastics from maxillary lingual to mandibular buccal.

170 QUESTION: Calcification of premolar tooth at birth? NO ( premolar = starts at 2years), 1st molars = birth ​ ​ 171 QUESTION: In relation to their parent drug, conjugated metabolites are what? more ionized in plasma (more water ​ ​ ​ soluble)

172 QUESTION: What best describes biotransformation? Increase in polarity, more ionized and more water soluble ​ ​ - Whatever helps its excretion – polar and more water soluble

173 QUESTION: Epinephrine = physiological antagonist of histamine & nitroglycerin ​ ​ ​ - Doesn’t act on same mechanism (epi = α vasoconstriction vs nitro = smooth muscle dilatator) but opposing action - Same mechanism = competive antagonist; physiological antagonist = competing physicological effects

174 QUESTION: What is bioavailability of a drug? amount of drug that is available in blood/plasma ​ ​ ​ ​ 175 QUESTION: Two different drugs with same dosages, bind to the same receptor, and cause same intrinsic affect. ​ ​ However, they have different affinities for the receptor. In which aspect these 2 drugs are similar? a. ED50 b. LD50 c. Potency d. Efficacy ​ - Efficacy bc they can both produce the same maximal response if enough is given

176 QUESTION: Main sign of dementia: ​ ​ a. confusion b. short term memory loss c. long term memory loss - short term memory loss = first main sign. Long term loss occurs later. ​

Combined remembered Qs 2020

177 QUESTION: What catecholamine do tricyclic antidepressants affect? Dopamine, serotonin, acetylcholine ​ ​ ​ ​ 178 QUESTION: Know drugs used for conscious sedation à SSRIs/BDZ Diazepam and Prozac (fluoexitine ​ ​ ​ 179 QUESTION: Why is nitrous oxide used on children? Alleviate anxiety ​ ​ 180 QUESTION: When is nitrous contraindicated? Asthma or COPD ​ ​ 181 QUESTION: What is not on cocaine overdose? pinpoint pupil ​ ​ - Cocaine OD—mydriasis - Opiate OD—pinpoint pupil

182 QUESTION: Indication for antibiotic prophylaxis: Prosthetic valve ​ ​ 183 QUESTION: (Patient’s medical tab say he is allergic to Amoxicillin), He needs to be premediated, what do you ​ prescribe? Clindamycin, 600mg 1hr ​

184 QUESTION: Pt w/ total knee replacement but was taking Amoxicillin for a while; how do you premeditate? NO (or ​ ​ ​ MED CONSULT)

185 QUESTION: Which med kills only anaerobic and parasites: metronidazole ​

186 QUESTION: Systemic antifungal: Fluconazole (diflucan) ​ ​ ​

187 QUESTION: Purpose of Major Connector ​ Stability and Rigidity Stability and Retention Retention and Rigidity Rigidity and Esthetics

188 QUESTION: Main purpose of buccal flange of Mx denture? Stability ​ ​

189 QUESTION: Which of the following best explains why the dentist should provide a postpalatal seal in a complete ​ maxillary denture? The seal will compensate for: A. errors in fabrication. B. tissue displacement. C. polymerization and cooling shrinkage. D. deformation of the impression material.

190 QUESTION: Asked about what sound will determine VDO? S sound. This will bring teeth slightly together with 1-1.5 ​ ​ ​ mm separation. This is the “closest speaking space”

191 QUESTION: When you find VDO & the max tuberosity touches retromolar pad, what should you do? ​ • Make metal extension on mand RPD • Surgery on max tuberosity • Surgery on retromolar pad • Open VDO

Combined remembered Qs 2020

192 QUESTION: Patient feels fullness of upper lip after delivery of complete denture: Overextended labial flange ​ ​

193 QUESTION: Pt wearing a complete dentures & is cheek biting: posterior teeth set up with no horizontal overlap. ​ ​ 194 QUESTION: Open mouth while maxillary border molding - Coronoid process will block buccal extensión ​ ​ 195 QUESTION: Posterior buccal extention of a mandibular complete denture is limited by: ​ Masseter muscle

196 QUESTION: Mand CD interfere with what muscle in lingual side? Mylohyoid. ​ ​ 197 QUESTION: You would relieve a mandibular denture in the area of the buccal frenum to allow which muscle to ​ function properly? Orbicularis ​ Oris

198 QUESTION: What is the best way to treat a tooth supported lower denture? Use metal copings to cover teeth ​ ​ 199 QUESTION: When does cleft lip and palate develop? 6-9 weeks in útero ​ ​ 200 QUESTION: What is true of patients with Down Syndrome/trisomy 21? Lower incidence of dental caries ​ ​ 201 QUESTION: What resembles epiphyseal plate? Synchondrosis ​ ​ 202 QUESTION: Sphenooccipital closure, what kind of tissue fills it in? Cartilage ​ ​ 203 QUESTION: Which of these undergo suture closure latest? ​ • sphenoethmoidal • Sphenoccipital ​ • Intrasphenoid • Intraoccipital

204 QUESTION: Cleidocrainal dysplasis – supernumerary teeth & problems with eruption ​ 205 QUESTION: Which will give you very narrow facial structures and delayed eruption of permanent teeth? ​ • cleidocranial syndrome • downs syndrome

206 QUESTION: LA with epinephrine contraindicated in? Uncontrolled Diabetes, hypothyroidism, hyperthyroidism ​ ​ 207 QUESTION: Tell patient that he needs to take of amalgam fillings bc they are not good for his health (hazardous): not ​ ​ practicing veracity (truthfulness)

208 QUESTION: Pt presents with amalgam restorations in good shape and the dentist suggest to change them for ​ composites due to systemic toxicity of the amalgam. What ethic principle is the dentist is violating? Veracity ​ 209 QUESTION: When should patient sign informed consent forms for surgery? AFTER there has been a discussion w/ ​ ​ the dentist about the Surgery

210 QUESTION: Definition of rapport? mutual openness / harmonious relationship ​ ​

Combined remembered Qs 2020

- Rapport = mutual sense of trust and openness between individuals that, if neglected, compromises communication

Paraphrasing: repeating, in one’s own words, what someone has said. This serves to confirm one’s understanding, validate a patient’s feelings, convey interest in the patient’s experience (thereby building rapport), and highlight important points.

211 QUESTION: Pt. says, “I do not have time to quit smoking.” What stage is s/he in? ​ A: Precontemplation, contemplation, action, denial ​ ​ 212 QUESTION: How to reduce stress & dental anxiety? Tell-show-do ​ ​ 213 QUESTION: Autistic kids have what characteristic? Repetitive behavior ​ ​ 214 QUESTION: if you find problems with a medical conditions occurring with a certain drug, who do you contact? OSHA, ​ FDA, EPA ​

215 QUESTION: If there is an adverse reaction to a medication in the office, who do you notify? a) FDA b) CDC c) HIPPA d) ​ ​ ​ ​ OSHA e) EPA.

216 QUESTION: What is it called when a dentist charges several procedures instead of one? ​ a. upcoding b. downcoding c. unbundling d. bundling

Feb 10- 11

-child with bruise on his belly Tell assistant to call child abuse Or call after appointment

-Ear lobe on panoramic

-Pterygomaxillary fissure on pano

-Kid felt from bicycle has Headache & nausea what u Do ?

-what's the most important when choose the tooth shade ? I made value

Info: Hue is the color Value darkness & lightness Chroma saturation of color

-Also i had the question about who is responsible for hygienist mistake!?

Combined remembered Qs 2020

All 3 dentists and hyginest

-Like wt kind of force applied to a tooth with vertical bone loss: A/Heavy w great moment B/Heavy w light moment C/Light w great moment D/Light w light moment? I choose this but not sure

-translation movement, the force must be applied through :

A- center of resistance B- Center of rotation C-Root D-Crow

-What is component in algintae that react with calcium n give the desored working time.

Trisodium phosphate.

-Most of hypertension pt show little complaince of the medication why Because It cause : A-Baldness B-Or sexual dysfunction

-How to handle asthmatic patient Position & medicine & precautions

-ADHD pt 9 years dental management With previous bad experience: A-Discus with pt B-Discus with parent C-Don't discus while extraction of molar Fractured pieces of bone Wht to do?

A/Remove all fractured pieces B/remove all except pieces attched to periosteum

-Lingual inclines of baccal cusp of mand molars: working Non working

-3 Questions on the sequence of treatment

Combined remembered Qs 2020

Memorize PROSP

-Most recurrent infection with HIV: bacterial fungal

_what's the eye dysfunction called if someone has eye problem in which If he is looking at moving object And both eyes moves but at the midline one eyes stop moving and the other keep going?? ?All option end with gmus and only recall nystagmus ؟A/ I can't remember the options

When the pt all time nagging and dr got angry and shout on assistant when she dropped the instrument what is that?

.patietn is scheduled to extract multiple teeth, best time to do alvioloplasty?

-Same day of the extraction

If vertical dimension of rest is 66, what is VDO?

- 63(should be 2-3 mm less) what is not important in the first visit for patient needs cleaning?

-update medical history

-X-rays

-periodontal charting

-Diagnostic casts?

Kenddy class III main retention?

-direct clasp

-indirect clasp

-rests requirement for major connector?

Combined remembered Qs 2020

-Support

-retention

-stabilization Rigidity and stability injury to which nerve/artery could lead to blindness?

Temporal main sign of hypoxia ?

-Tachycardia

-blue finger tips after you give a block of LA to a kid he feels agitated, main reason?

-injection in the artery main reason when doing CPR, inflation ? compressing with too much force? main reason for failed CPR? obstruction air way

The space difference between primary canine, first & second molar and the succedaneous teeth ? freeway space primate spacing leeway space

-pathology, pt with pain in neck and when swallow and when turn neck to lateral side A- eagle syndrome

-condyle inclination is used for? Protrusive record

Combined remembered Qs 2020

-which X-ray technique shows the floor of the maxillary sinus imposing on the roots of maxillary?

-bisected angle technique periapica - DETERMINE TONGE in pan - DETERMINE Airway in pan

-What not cause nephrotoxcicity

- 2 Q like if dentist Extract all teeth and do complete denture under which code of ethic

-Central incisors looks wide to shape it what to do : A- put 2 Lines in middle of crown B

-know the exact definition of granuloma and absces and cyst and which one of them contain ​ ​ macrophage and lymphocyte ? ​

-Unrestorable tooth in pt take Biophosphante what to do: Extract Rct

-day 2 I had 2 children ortho cases 2 cases on complete denture 3 caseson RPD Few patho Many Pt management on both days

Over all I like day 2 than day 1 and exam is durable just focus in high yield fact they like pt management then prostho and operative then pharma and lastly period patholog

Kamalijot RQ

Carcinoma in situ- erythroplakia

White sponge nevus Lichen planus

Safer drug in hiv- Erythromycin Amoxicillin Ibuprofen

Class 3,4,5 composite bevel 45

Not a method to check sterilization-

Combined remembered Qs 2020

Electrical

Buccal roots max molar more mesial to palatal, position of tube- mesial distal inferior superior

Cells on tension side- osteoblasts pMn Kid after active ortho TX, Retention by- (history of poor oral hygiene) Fixed removable Other 2 options

Perio disease more in- Black male

Max sinus seems more close to max teeth in- Periapica

Best way to control caries- Flouride varnish

In IV for post op pain relief we Ketorolac

DKA- Hypoglycaemia Hyperglycaemia 2 other options

So many OP ques Periapical cementoblastoma Ossesous dysplasia Cemento*

Gingivectomy indicaitons

Lesion not on tongue- Peripheral granuloma Ectopic thyroid 2 more i dont remembr Lots of ques on finish lines, Axio gingival axio pulpal, grooves-blah blah I hope you got what i mean

Hemorrhagic shock first sign- tachycardia hypotension other options

Epinephrin works on- Aloha 1 Alpha 2 Beta 1 Beta 2 Answer: All

DKA- 2,3 ques on this topic

Ethics basic questions

All staff once in yr should go for checking of- TB

Osteoporosis radiographically- Thin trabecular Less osteoid calcification Other 2 options

Combined remembered Qs 2020

Palatial root more mesial to buccal root, position of tube- Mesial Distal Superior Inferior

Swelling of lip Mucus retentions cysts- Minor SG something Sialolith

Swelling of lip after anesthesia injection- Angiodema

Nitrous oxide and epinephrin antagonise by- Physiologic Competitive Other 2 options

NBDE PREPARE RQ nbde part 2 rqs 1.Pic(o) o in pico?? Outcome. 2.Multiple myeloma initially what sign do v see?? Bone pain !! 3.cold lingering,cold non lingering. 4.blue nails wer ?? Like in asthma or sarcoidosis or wer?? 5.many questions on cross sectional study. 6.super numerary teeth n multiple osteomas .wat do u investigate?? In options intestinal polyposis is der.its a twisted question I think for gardener’s syndrome. 7. Divergent pupal to gingival is the answer dnt remember question dnt knw if this is correct ans also. 8.many questions from operative n perio flaps especially modified widman flap incision like apical to mucogingival? It goes like dat. wer do u give n y do u give??regarding pocket elimination or y ?? Very little prostho n very little pharma. 9.many mango almost all n Rita. 10.radiograph radiolucent area wid radioopaque foci. 11.3 yr kid 16 kg hw much lidocaine?? 12.tooth n tissue born appliance. 13.10 MA exposure 1 gy .wat is the exposure for 0.5 gy if the density is same for both.i don’t remember. 14.t or f question vertical incision on mid root area....and vertical incision to cover roots or to cover recession it goes like dat.. 15.dentigerous cyst radiograph.

Combined remembered Qs 2020

16.cellulitis Wat do u see??neutropenia,lymphocytisis are options. ​ ​ 17.folic acid something in which cancer drug??sulfonamide block folic acid synthesis by competing with PABA . 18.bur with many flutes. 19.which microorganism in periodontitis without necrosis dint rem question .options were like filamentous rods,spirichetes ​ ​ 20.plaque hypothesis all are related to plaque hypo except

1. Options I forgot Day2. New drug names are given like Rouvastatin,bupiropreone I think it causes (xerostomia) check it ,lisinopril,ameloperene I don’t remember.,cevimelin,chantix,nifedipine these were mentioned in patient history question is like which causes xerostomia?options were rouvastatin+bupiropion ,lisinopril+rouva n so on. 2.18 yr old case with narrow arch n supraerupted canines a question is while performing extractions he loses consciousness heart rate 60 n bp is also very less like 70/80 or something wats the cause hyperventilation or vaso vagal syncope n other options. 3.extraction to a patient which options are correct 1.deep anesthesia is more affective by intravenous injection 2.Nitrous oxide inhalation causes amnesia 3.oral sedation has better titration .i don’t remember. 4.periapical cemento osseous dysplasia radiograph. 5.hyoid bone radiograph. 6.radiolucency between mandibular central incisors no signs or symptoms.answer is menta fossa

MARCH ● Kv=onset ​ ● Supernumerary=initiation ​ ● Angina/MI how to differentiate duratin ● Increase caries= root ​

Combined remembered Qs 2020

● Differentiate root caries= soft ​ ​ ● Recent studies prove periodontal disease related with=oral cancer (??)cvs ● Lingual inclination of lower buccal with buccal inclination of upper lingual when?=non-working ● When we split procedures make it multiple is this unbundling? ​ ​ ​ ● when the insurance decide to pay for crown not for endo is this bundling ​ ​ ● Positive Nikoliski sign but there was no pemphigus or pemphigoid...all weird options i picked hyper sensitivity reaction TRUE. Steven Johnson has + nikolsky ​ ​ ● Pt on anticoagulants with no more details...u want to do simple implant an ok INR would be 2.5...3.5 (3.5 is minimum for surgery) ​ ​ ● Pt is alcoholic and u want do surgery. What to check? Cbc..pt time. Something likes ​ ​ fat. Or glucose level ● Diabetic in day of procedure. Liquids with half dose of insulin? ● Best prognosis? 3 walls ​ ● Best furcation II? GTR ​ ● Healing after perio flap surgery? I picked long junctional epi..othe option was by ​ ​ formation of new bone, ligaments and ct ● Root amputation best prognosis: coronally and the roots are diverge...coronally and ​ ​ ​ the roots are converge...after my brain got freeze and start not differentiating between diverge and converge i ended up with convergep ● Pt came 3 days after ext with 6mm antrum communication....i chosed cose with gold ​ ​ foil stent ...there was flaps ​ Decks: Sinus communication abx amoxicillin ​ Opening 2-6mm figure 8 suture ​ Opening more than 7Closed with flap procedure ​ ​

● first 24 hours free gingival graft get from host nutrients from underlying connective ​ tissue. ● attached gingiva thickest in lateral maxilla thinnest lower first premolar T ​ ● most congenital missing was 2premolar lower ​ ​ ● lateral lower most to have 2 canals? i choosed T ​ ​ ● Study between two groups’ boys and girls? Chi square ​ ● Null hypothesis question? Group A is equal to B (There is no difference is null hypo) ● New invention drug experiments? Random clinical try ● most associated with vital tooth i picked middle palatal cyst ● Man molars get space by? i picked apposition of ant ramous . Resorption of ant border ​

Combined remembered Qs 2020

● Surgical guidance for implant for what? I picked to lead for location of implant. Others ​ ​ where to know the length of the implant ● U put palatal splint why? I picked to avoid contamination with fluid...other was to ​ ​ ​ protect bone and ct from trauma, to prevent hematoma and support the flap. ● Tooth gets its color from? Enamel, dentin, cementum? i picked dentin ​ ​ ● u make bulk PFM why? bec the opaque dentin layer was thick so u fix the color? i ​ ​ picked this other was the dentin porcelain layer was thin ● minimal accepted crown root ratio 1:1 true this is minimum its in decks ​ ​ ● best crown for posteriorzirconium ​ ​ ​ ● phenothiazine =extrapyramidal ​ ● Why u prefer zirconium in posterior? long longevity ​ ● Anterior cross-bite in child with central only what to do? Refer to ortho or fix it ​ immediately? i picked immediately ​ ● pt with gold inlay come after 3 hours with shooting pain why? galvanic shock or hyperocclusion....i don’t remember which one i ended up with but i think galvanic (DEPENDS IF THEY SAY IF DIFFERENT METAL IS OCCLUDING) ● 45 yrs with bilateral post cross bite how to fix? i picked surgery ​ ● ortho tooth with vertical bone resorption what to expect...i chosed less force...less time ● ? i picked to restrict the x-ray beam ● What is illegal for dentist? i picked to prescribe schedule 2 for back pain not wine, not ​ ​ sexual relation not advertise with price ● not ADA code ...i picked Credential needs to be a dentist (licensure) ● what is not in concent...i picked price ​ ● distance when to discuss price with pat i pick 3-6 feet...there was 8-11 feets ​ ​ ● pt have pain in one side of face this pain worse when moving his face what is most probably? i picked unilateral thrombosis...other was sinus but the said when move the head not lay down or bent ● Collimination restricts the size and shape of xray beam reducing patient exposure ● Can mump be bilateral? YES ​ ● pt come with swelling upper lip happen bc yesterday had treatment why? i pick ​ haemangioma ( probably angioedema) ● Tooth with lingual anatomy cause calculus to stick? i pick upper central or upper ​ lateral can be if in options if central it’s spcingulum if lateral it’s gongivoocclusal groove ● Why not to take color while put rubber dam? i pick rubber dam make tooth look lighter ​ ● How to diagnose vertical roots fracture? 1-ray 2- visual 3- use the probe and look for ​ ​ pocket… X-ray is the only way to diagnosis for any root fracture radiolucent halo seen. ● DMF...Filled tooth is with most? White ​ ● u broke the ridge what to do? i pick u put lingual bar and make sure its stable

Combined remembered Qs 2020

● What protection for root caries? Home stannous fluorideor home neutral sodium ​ ​ ​ fluoride? Decks say stannous is best for root caries ● What fluoride does? I put decrease enamel solubility ​ ● How saliva help in reminerlization/demineralization? Options like saliva has calcium, other saliva has ions, I picked the option says saliva is there 24/7 and it help in demineralization/re-mineralization ● What adhesive do? All options about removing smear layer i put it don’t remove ​ ● All improve crown retention except? I put decrease convergence of preparation. Other option was shoulder All around ● Elastic for posterior molar? Put on lingual upper and buccal lower ​ ● Upper central intruded 6mm what to do? I put leave it ​ ● Sealant close fissures by? I put micromechanical ​ ● Sealant....stop caries if there is small in deep fissure or fossae i put true ​ ● If animal were feeding through stomach feed what happen? Caries reduce ​ ● Type to allergic II reaction? I closed RH blood transfer from mother to baby ​ ● In asthma attack? Put pt on upright position ​ ● What first thing to do after u gives shock to pt with cardiac arrest? I put check pulse ● ...i pick junction of buccinator and superior constrictor ​ ● Ian block only tongue numb no lips? Re do the block incisive block ​ ​ ● U give 1 carpule of lido to kid and he start being super active and agitated why? I pick intravenous administration. Others were allergic to lido..allergic to epi ​ ● What to make sure to do in pedo anesthesia? I pick aspirate and administer slowly ​ ● What make intracranial anesthesia work? Pressure anesthesia ​ ● Dentist was nice and professional with pt then after appt he become mad on his assistant for dropping instruments? I pick dissociation ( Displacement )

You had a very difficult pediatric patient who was crying a lot. You remain calm and professional the whole time. When the patient leaves, your assistant accidentally drops an instrument on the floor and you get extremely angry at her and yell at her. This type of defense mechanism is known as what? Dissociation Displacement Desensitization

Combined remembered Qs 2020

● Dentist dont do treatment on pt..didn’t contact him...i put neglecting ● Xerostomia dont do..i pick teeth attrition All of the following cause xerostomia except? a. caries, b. candidiasis c. dental attrition

● Refuge pt don’t speak English has severe pain and she believes bad spirits behind this. What is the most concern of the dentist will be? Language? Consent form from her daughter? Her believe? Or removing the pain? I picked removing the pain ● Refuge pt dont speak English but her daughter does. What best to do? Ask daughter to translate? Ask office manager to translate? Call healthcare interpreting phone ​ service? I picked the phone service since they should be professional and know the ​ terminology ● What is not obligated by HIPPA? I pick dentist send bi-annual report to DHSS ​ ● What annual screening mandated for all healthcare workers? I pit TB test ​ ● What bacteria in pocket? I picked spirochetes??? ● What primary disinfectant should do? I picked kill tb (inject according to Trump) ​ ​ ● X-ray of head when u see it its typical picture of moon surface with two or three empty circles. Border is fine? I picked osteoporosis ● Osteoradio necrosis pt with pain what to do? Endo and crenectomy ​ ● Pt with red bleeding gingiva and with something in the extremities ...no picture just description i picked acute leukemia. I don’t remember if the age was young but mostly ​ ​ it was young ● Less hair? Hypothyroidism (it suppose to be thin hair if less ectodermal) ​ ● They asked me about the eyes are not in the same direction what is this? No diplopia ​ in options STRABISMUS ​ ● How to see vertical resporption and diagnose heavy caries for endo ? Options where cbct, occlusal, pano and mesial shift x ray? I picked mesial shift ● What used to diagnose perio bone level? I pick pano…options where bitewing, ​ ​ periodical’s, cbct ● Eagle syndrome..pano with calcified stylo ● Mixed dentition help dentist to? I pick know the dental age ​ ​ ● One big central Mesiodistally with two pulp chambers is an example of? Fusion ​ ● Diastema closure is an example of? External something and i put intraoral something ​ (the something is like splint. Or approached don’t remember the word)- BONDED BRACKET WITH INTER TOOTH TRACTION ● Posterior landmark of upper denture landmark is? I put palatal vault

Combined remembered Qs 2020

● Most reason to replace anterior composite? I put color change ​ ● Major connector? Rigidity and stability ​ ● Premolar is moral and under it is a white radiopaque big circle? I pick idiopathic ​ osteosclerosis ● Cyst around canine? AOT ​ ● Implant platform is? 2-4 apical to adjacent gingival margin tooth ...all other options where apical ● Implant 3-4 from adjacent cej ● Cavernous sinus thrombosis? Swelling in canine fossa region ​ ● 3rd molar disappeared? Take x ray to locate options where open from canine fossa ​ ​ and do the procedure ● MTA stimulate? INDUCES HARD TISSUE FORMATION ​ ​ ● Neuroparaxia...dint say temporary or motor. Just pick the opn say cut in axon but not in neurostium ● What big nerve effect on nerve order? I said no change other were increase, decrease inhibit ● Drug A is more effective than B what means? I pick drug A can cause stronger effect higher potency ● Diagram of three drugs asking which one is more effective I just pick the drug with the high line in the diagram ● Color stability=TEGDMA( causes low color stability ) only UDEMA make color stable ​ ​ ● Pt class three skeletal what decrease with age? ANB ​ ​ ​ ● Impression not for fixed prosthesis=irreversible hydrocolloid ​ ● Effect polymerization=eugenol ​ ​ ● Cleft palate does not cause? Change in teeth shape ​ ​ ● You saw a pt a year ago he was taking nitroglycerine for angina once per week when ​ he do exercise. Today you saw him with a permanent patch of nitroglycerine that he take daily 3-4 times…what is his ASA I pick 4 ​ ● Dry mouth=anticholinergic ​ ● Overhanged fillingwedge was the problem ​ ​ ● 3 months baby=no Fluoride supplement ​ ● At birth teeth calcification= the only option with permanent first molar said: permanent ​ first molar and all primary teeth ● Ectodermal displace= oligodontia ​ ● Agenesis= canine least ​ ● Sialosis= submandibular (Wharton’s duct) ​ ● How to make tooth looks narrower? I pick to bring the lines together and the ​ ​ embrasure look shallower

Combined remembered Qs 2020

● Least one to get NO2 in the clinic? Patient ​ ● Effect of radiation on the body which molecule? H2O ​ ● Distal rest? To resist vertical force ​ ​ ● Not present in class V=incisal wall ​ (axial/pulpal wall is present) facial wall isn’t) ● Tooth was discolored Q what to expect this tooth had before? Trauma ​ ● =CD4 ● Tooth painful on hot relieved by cold=irreversible pulpitis ​ ● Longer treatment plan less compliance of patient? Both true ​ ● Periodontal disease due to intrinsic causes. Intrinsic or systemic factors play role in ​ progress of disease. First false second true ​ ● Picture of probe 7 mm pocket depth +2 recession total=9 ● Secondary occlusal trauma ● Cherubsim ● Chronic apical abscess = sinus tract ● Airspace in pano right side ● Zygomatic bone in pano left side ● One pano with crest in the middle of centrals I put nose curve ● Two upper and lower no horizontal overlap / reduce buccal to avoid cheack bite ● Increase VOD and decrease intercuspal = patient denture has no retention, chewing ​ while talking

Day 2

1-cryer elivator...ext lower molars roots ​ 2-Tooth#3 has filling it’s not big normal one, tooth is asymptomatic, pt dont complain, u checked and its non-vital what to do? No treatment ​ 3-Pt live in Fluoridated area. His teeth are all clean like manikin teeth. The question what preventive treatment on his lower molar? Options sealant, varnish, composite, no ​ treatment...i pick no tx ​ 4-Pt has 9 teeth upper arch all of them are good. Pt ask to extract them all...what is the least u should do? Complete denture? I choose this ​ ​ 5-If u did what he asked for what code of ethics u break? Non maleficience and autonomy ​

Combined remembered Qs 2020

6-The patient asking for full extraction is related (don’t remember the exact word but means is under which right as the pt think) to which code of ethics? Autonomy and beneficence 7-Pt with poor oral hygiene 100% plaque...has diabetes controlled...asked me what the contraindication for putting implants is? I picked plaque ​ 8-Pt with missing 14-15...there is opposing...and missing 30 and there is opposing...the question which improve the occlusion? One option is to put partial 2 units 14-15 and implant 30 9-ADHD pt Q he start becoming overacting and aggressive during the procedure...talk to his parents privately about what is wrong...talk to the assistant about this issue...cancel appt..talk to the ​ child directly to know what bothers him? I picked D ​ 10-What make his gum looks like this? What i saw was red, dry, little bit like hyperplasia...I picked his medication 11-Does his medication do? Xerostomia 12-Class II Kennedy 13-Fulcrum class II Pt 16 years with pano he has retained uper right 1st primary molar and 1st-2nd left molars ...no permanent premolars clinically or in xray..his upper teeth from canine to canine are all scattered in distal directions and spacing ..diastema...his teeth, gum are healthy 14-when to do frenectomy? I pick before closing the diastema before frenectomy ​ ​ ​ ● You wait until canines erupt, then ortho, then frenectomy

15-what to ask the pt? Caries history, systemic conditions, family missing teeth history...C 16-what angles classification? The picture shows the canines both sides and both were class III 17- What is the pt profile! I pick strait because. The centrals where in class one relationship with lowers normal over jet -over bite in cephalometric x ray It was the upper canines inclined and spaced distally due to missing teeth 18- Why the primary teeth still there? I put ankylosed

Combined remembered Qs 2020

19-Don’t remember the question but they want to pick which medication is not good for the pt Ibuprofen Tynolol 3 Acetaminophen +hydrocodone Oxycodone I picked ibuprofen since i believe all other 3 are related 20-Asthmatic pt become anxious what to do? I put nitros oxide 21-Bipolar...lithium ​ 22-Pt become getting stress and anxiety what preoperative med? 2 MG valium ​ Other was Prozac (we are not physicians) 23-Pt with 9-8 want the incisal edge to be esthetics... the #9 is perfect the #8 has a very small chip in the incisal edge like the size of the probe tip..all options where over treatment and include #9 i pick correct #8 with a bur 24- Is lingual artery entering the mandibular canal? No ​ 25-Class II furcation? GTR ​ 26-Tooth 2 no opposing tooth was in x ray looks super erupted..q. Does it need re arrangements for occlusion if we put lower implant? I pick yes 27-Pt need upper denture there is a big elevation not painful pt even didn’t know it’s there what is this? I pick exostosis ​ 28-What best to diagnose? I pick incisional biopsy ​ 29-U extracts a tooth and under u hyper plastic vascularized ct what is this? Granulated tissue 30-U extract tissue found radicular lesion encircled by epithelial tissue? Cyst 31-Small pink spot under the lower incisor? Parylis, fistula, Fordyce granule fibroma? I pick fibroma Pt with #8 very slightly prepped facially and no cover. It’s the only tooth has edge to edge with lower central...2 questions 32-what was the restoration? Veneer

Combined remembered Qs 2020

33-why it fails? Occlusion 34- pt move to sit upright and he became dizzy what to do? Give oxygen, give glucose, take vital, sit in semi-supine position? D ​ ​ 35-why the pt had these symptoms? orthosthatic hypotension 36- Diabetic pt unconscious? Administer glucose ​ 37-line within cervical area of molar? External oblique ridge ​ 38- Treatment sequence for bad perio pt with problems perio – ortho - prostho ​ 39- Treatment sequence for pt with primary retained teeth with spacing and need...Extraction-ortho-prostho ​ 40- Deep caries? Remove infected, leave affected if two close ​ 41-Class 5 lower premolar what filling? GI 42-What is the difficulty? Isolation ​ 43-Pt drink energy drinks 3-5 times a day. While he was walking to clinic he was drinking also? What is his mouth PH once he sits on your chair? 5.6 - 6.5 - 7.5 - 8? ​ ​ ​ ​ ​ ​ ​ ​ ​ ​ ​ 44-Who build Dentin Bridge? Calcium hydroxide ​ 45- Ferule effect? Prevent root fracture ​ 46-Tooth 7 with big post 47-Tooth 8 with short post 48-Tooth 8 with surgical retreatment 49-What surgical guide don’t decide? Number of implants Location of implants Size Angulation

50-Bridge bulbous 51-Extraction done before the picture show irregular tissue and l...i pick loss of buccal cortical bone during the previous extract

Combined remembered Qs 2020

52-Which tooth need endo retreat? #7 53-Which bridge to cover? All options were for missing teeth except 1 54-Pt on smoke session what not to give? Smokeless tobacco ​ 55-Internal resorption primary EXT ​ ​ 56-What is not count if u decide to extract this primary? Furcation involvement (because tooth has disto caries) 56- Anxiety pt preoperative med? 2 mg valium ​ 57-lithium...For bipolar ​ 58- Lateral tooth picture with mamelons..Question what are these? Normal development ​

Shamili

Sam’s Questions

1. Best area to place an implant ant mand ​ 2. Best area for osseointegration- post mandible 3. What determines the color coefficient of composites hue

4. Pt wants to Extract all his teeth and want a CD, even though all his teeth are good. What ethical principles clashes with this autonomy and non maleficence ​ ​ A. Autonomy & Justice B. Autonomy & beneficence C. Non malfeasance & Beneficence

5. Characteristic of cherubism ● Most cases in mandible ● Bilateral expansion of jaws ● Tumors stop growing after puberty ● Histologically resembles central giant cell granuloma

6. Which is seen least in elders A. Drug abuse is mainly alcohol B. Drug abuse with illegal drugs mostly C. More prone to over use of drugs than misuse ​ D. Least likely to miss use the drugs

7. Radiographs on A. Multiple myeloma B. Pterygomaxillary fissure

Combined remembered Qs 2020

C. Ear lobe D. Air space E. Dentegerous cyst F. Taurodontism G. Ext oblique ridge H. Short crown and roots but pulp is not obliterated and no Periapica radiolucencies also - I chose AI I. Eagles syndrome ( pt is unable to open mouth and pain)

9. Pictures on A. Primary HGS B. Papilloma C. Squamous papilloma D. Recurrent herpes E. Amalgam tattoo (Rct with retrograde filling in 7. What is the pigmentation in the vestibule of tooth 7) F. Pyogenic granuloma or exostosis (confused b/t these two)

10. Dentist writes a prescription to the pharmacy for a 60 year old and mentions not to give child proof lid, this is because A. Social barrier (Options were weird, don’t remember)

11. Pt is irritating the dentist and meanwhile assistant drops an instrument. Dentist shows his anger on her. This is called (the answer given in the file is not in the options)

12 pt has cl 3, what keeps decreasing as he grows A. ANB ​ B. SNA C. SNB D. Point B

13. Point A in ceph is from most concave point of anterior maxilla ​ The “most posterior” point from ANS to supradentale with the head in Natural Head Posture.

14. Pt has anxiety, what do you give him for this in next appointment to keep him calm - Diazepam

15. All of the following frugs MOA is by effecting the ion transport, except A. Proprenolol B. Lido C. Ca blockers

Combined remembered Qs 2020

16. Lingual inclines of buccal cusp of mand molar interferes in lateral movement, which is side interference is this A. Working B. Non working ​ C. Protrusive

17. Fracture of condyle, jaw deviates to A. Anterior, medial.✔ B. Superior, medial C. Posterior, lateral D. Anterior, lateral

18. Purpose of face bow transfer- retain

19. Keratinic lesion in which location most likely to turn into dysplasia A. Buccal mucosa B. Floor of the mouth ( think of carcinoma in situ) ​ C. Tongue

20. Pic of bilateral edentulous jaw (day 2) A. What kennedy classification? Class 1 B. Where to place rest: mesial of last abutment tooth always C. Fulcrum passes between which tooth

21. Pt has wheezing with aspirin, best alternative is Nsaids are contraindicated in asthma. Give tylenol

22. Special characteristic of codon-unambiguity, redundant and universal ​

23. Increased dose of Acetaminophen causesHepatotoxicity ​

24. What bacteria seen at root apex of necrotic tooth: ANAEROBIC A. Facultative anerobes B. Aerobes C. Motile

25. Improperly drained puss from perio pocket leads to A. Abscess ​ B. Cyst C. Granuloma

Combined remembered Qs 2020

26. Most recurrent oral cyst OKC ​ ​

27. Which of the following is assosciated with Basal cell nevus OKC, gorlin syndrome ​ ​

28. Soft tissue swelling in the mucobuccal fold lateral to root of lateral incisor - nasolabial cyst ​

29. Why tetra and penicillin not given together static and cidal. They cancel eacother out ​ ​ 30. Advantages of GIC fluoride, bond ​ 31. Implant distance from tooth 1.5 ​ ​ 32. Implant should be placed where in relation to the CEJ of adjacent tooth 2-4 mm from ​ ​ cej 33. 1ppm of fluoride in 1L water. How much in mg 1 mg ​ ​

34. Handicapped condition associated with brain damage and causes neuromuscular dysfunction A. Trigaminal neuralgia B. Bells palsy C. Cerebral palsy ​

35. Medication of Trigaminal neuralgia Carbamazepine ​ 36. Function of major connector in FPD stability and rigidity ​ ​ 37. How should a Pontiac be designed not blanch ​

38. Vertical incision should be placed in mid facial area of tooth ( true or false kinda question) Vertical incision is used to raise the flap

39. Space for mand molars is created by

40. Post extension of mand denture limited byRetromolar pad is the distal termination of ​ ​ mand denture (which is post to start of rise of ramus)

41. Pendulated maxillary tuberosity, touching the opposing counter part. What do you do A. Osteotomy ​ B. Ostectomy C. Bone file

42. East west cryer used for mand molar roots 43. Flabby anterior maxillary area, dentist uses which imp material zinc oxide eugenol ​ ​ paste 44. Questions of diagnosis and treatment of Reversible and irreversible pulpitis 45. Questions on apical periodontitis 46. Questions on apexification and apexogenesis

Combined remembered Qs 2020

47. Black pigmented area over 7, best diagnosed by (pic related) A. Incisional biopsy ​ B. Excisional biopsy C. Cytology

48. Change in hair texture is associated with which gland disfunction - Thyroid ​ 49. After giving nitroglycerin to stroke pr, how will you you differentiate if it is MI or Angina A. Intensity of pain B. Location of pain C. Characteristic of pain SHOULD BE DURATION OF PAIN

50. Pt comes with pain, tells he is unable to sleep last night as pain was waking him up. Treatment is A. RCT ​ B. Antibiotics C. I and D of root canals

51. Question on calculating attachment loss 52. Question on wavelength in shade selection –HUE ​ 53. Bonding of pit and fissure sealants with tooth by micromechanical retention ​ ​ 54. Dentist conducted a study following a new research. But it gave more false negative. even diseased people are shown as non diseased. Which factor is affected A. Sensitivity ​ B. Reliability Specificity non diseased Sensitivity diseased ​

55. Most common reason to redo anterior composites is A. Stain/ color change ​ B. Chip

56. Which position to place the pt during syncope tredelenburg. Supine with feet up. ​ ​

57. Asthma pt, which position you make him sit during the middle of the treatment A. Straight with slightly leaning forward B. Supine C. Left supine D. Straight with nasal catheter with o2

58. First thing dentist should look for if a pt goes unconscious in the middle of treatment A. Carotid pulse ​ B. BP

Combined remembered Qs 2020

C. Glucose levels D. HR

59. Hyperglycemic pt faints 1 hr after the treatment started, what do you do TXT hypoglycemia: page 143

● Conscious oral carbohydrate OJ, table sugar, cola, candy ​ ● UnconciousEMS should be contacted then 1mg of glucagon injected IM. Should ​ ​ restore pt within 15 min.

60. What age cleft palate occurs

Cleft lip 4-5 weeks Cleft palate 8-9 weeks

61. Pt has depression and is on antidepressants, which of the following drugs causes xerostomia. TCA is the strongest anticolinergic

62. Antagonist to midazolam flumazenil ​ ​

63. Antagonist to meperidine naloxone ​ ​

64. Sequence of treatment planning after perio A. Endo,ortho,prostho ​ B. Prostho, endo, ortho C. Ortho, prostho, endo

EPOSOP

65. Signs of narcotic over dose miosos ​ ​ 66. All of the follow cause bilateral swelling, except (I got 2-3 questions on bilateral and unilateral swelling of PG臘♀) ​ ​​ A. Mumps B. Warthins C. Pleomorphic adenoma D. Sialolithiasis ​

Anorexia causes bilateral swelling

African,5 years boy complaining of bilateral facial swelling, x.ray showed multilocular radiolucenency,cause roots resorption with (Starry Sky appearance): A- Burkitte`s lymphoma ​

Combined remembered Qs 2020

B- cherubism C- fibrous dysplasia

The most common cause of bilateral swelling of the parotid glands in children is A. Mikulicz' disease. B. mumps or acute infectious parotitis. C. mixed salivary tumor (pleomorphic adenoma). D. sialolithiasis.

Bilateral symmetrical swelling of the mandible of a child is likely to be caused by A. Acromegaly B. Paget’s disease C. Giant cell lesion ​ D. Primordial cysts E. Dental cysts

67. Most commonly associated with Sialolith wharton ​

68. Pt has fever, unilateral facial swelling, yellow discharge from parotid on the swollen side, diagnosis A. Sialolithiasis ​ B. Warthins C. Pleomorphic adenoma D. Nectrotizing sialometaplasia

69. What causes turner’s hypoplasia - trauma or infection of primary tooth ​ ​ 70. Grade 3 furcation, best treatment hemisection, root amputation

Grade1:scaling Grade2: gtr Grade 3: hemisection for mandi Root amputation for max Grade 4: extraction

71. Class 2 bone loss, best treatment 72. You can only place one implant in a pt taking warfarin. The INR should be A. 2.5 B. 3.5 ​ C. 7 D. 12

Combined remembered Qs 2020

73. Test to check kidney function - creatinine ​ 74. Chronic alcoholic pt comes for extraction of 22 teeth, which test you suggest A. CBC B. INR ​ C. BT

75. Which of the following is not an NSAID A. Ibuprofen B. Aspirin C. Celecocsib D. Acetaminophen ​

76. Questions on unbundling 77. Questions on downcoding 78. To study effect of specific drug on a disease, what is it called A. Cohot B. Case control C. Randomized trails ​ D. Cross sectional

79. Preauricular surgery or incision which nerve is mostly likely be damaged facial ​ ​

80. In IANB, you inject into a artery that is a A. Lingual artery branch of ICA B. Lingual artery branch of ECA C. Inf alveolar artery branch of ICA D. Inf alveolar artery branch of ECA ​

81. Pterigomandibular raphe consists of what sup constrictor and buccinator ​ ​ 82. What is the only muscle that gets you pierce through while giving IANB buccinator ​ ​ 83. Dentist places pit and fissure sealant on a active decay, what happens to the decay A. Increase B. Decrease C. Stay same D. Arrest ​

Combined remembered Qs 2020

84. How does arrested caries look on a root surface- leathery brown ​ ​ 85. Two different drugs with same dosage binds to the same receptor. One drug has more effect than other. It has more A. Efficacy ​ B. Potency C. Pka

Drug a has higher efficacy than drug b: options: drug a needs lower dose bc its stronger, ​ ​ drug a has higher affinity for receptors Drug A gives maximum effect than drug B drug A has higher efficacy than B more potent smaller dose

When comparing drugs with respect to intensity of response the drugs that produce greatest maximum effects is the one with the highest Affinity Potency Efficacy ​ Therapeutic index

Combined remembered Qs 2020

86. Some question about pka also was there ( don’t remember exactly)

87. Where is the thickest and thinnest attached gingiva (question in the exam was framed differently, but the meaning is same as what I wrote above)

Thinnest mand pm and canine ​ Thickest maxillary centrals ​

88. Mandatory Annual screening test for dentist is TB ​ ​

89. Dentist did a composite class 5 filling, pt came back next day with pain because A. No liner or base ​ B. Too much acid etching C. Improper cavity prep D. Axial wall is deep

90. Characteristic of ectodermal dysplasia 91. What condition it is if it has cleft palate, glososptosis and micrognathea - Perry robin ​

92. Not required in a cavity prep in a class 2 on a primary tooth- need not rounden the internal line angle No gingival bevel ​ ​ ​

93. Cledocranial dysplasia characteristics

94. Which local infiltration you give for maxillary premolar A. Endosseous Local Infiltration ​ B. Paralingamental C. Transligamental D. Supraligamental (Weird option, I know)

95. Which is common between chronic periodontitis and localized aggressve periodontitis A. Number of teeth involved B. Treatment C. Other options I don remains Find for pocket, tooth mobility, bone loss is similarity and difference is aggressive doesn’t have plaque, calculus, inflammation

96. Initial treatment for localized periodontitis srps and abx metronidazole, tetra or amoxi ​ ​ 97. Which of the following condition requires gingivoplasty - ANUG ​ 98. 7 year old, fractured Right central incisor, 3 hrs ago still bleeding. Treatment is A. Pulpectomy and apexification B. Pulpotomy and Cal hydroxide ​ C. Direct pump capping

Combined remembered Qs 2020

D. Pulpotomy with apexogenesis

99. Dentist gave IANBon right side of mandibule, pt feels tongue numb but not lip. Where do you give anesthesia to numb lip A. Inscisive B. Mental ​ C. Local infiltration D. Left IANB

100. Reason for saliva seeping through the rubber dam A. Holes too far B. Holes too close ​

101. Pt is allergic to latex, what is the next step you do A. Do not use rubber dam B. Use rubber dam with napkin C. Use nitrile dam ​

102. Most common problem a dentist faces while restoring a class 5 with composite A. Cavity prep B. Moisture control ​ C. Bond to tooth

103. Medication for status epilepticus diazepam ​ ​ 104. Commonly used porcelain for PFM (weakest) Feldspathic porcelain

st 105. Eruption sequence of primary teeth central lateral 1 ​ molar canine 2 molar ​ ​ ​ 106. Each of the following cause perio ptoblem, except A. Trauma from occlusion ​ B. Nicked soft tissue while tooth preparation C. Rough over hanging restoration

107. In DMF, F is more in - whites ​

108. Which has greatest effect on polishability of composites filler size ​

109. Dentist can start prescribing fluoride supplements since what age A. 6 months ​ B. 3 years ​ C. 12 years

Combined remembered Qs 2020

D. 16 years

1. Pt os 16 year old, which of the following you’ll not do - pit and fissure sealants ​ 110. Termination of local anesthetic articaine - plasmacholinestirase ​

111. MOA of penicillin V oral..cell wall synthesis ​

112. Reason my preferring Pen V over Pen G because, pen G is ● Pen v oral, is less sensitive to acid degradation that’s why its given oral and not Pen ​ g

114. Tetracycline acts by interfering with protein synthesis 30 115. Mental retardation caused by placental transfer of rubella is caused by A. Aquired B. Chromosomal C. Genetic

March 3-4 Rq's DAY 1 1. Drug interaction Epi+LA

2. Drug interaction Epi+Propanolol/Amox ● Propranolol non selective beta blocker, used to treat hypertension ● Drug of choice for adrenergically induced arrythmias ● Beta blocker and epi hypertensive crisis ​ ● Duration of lidocaine can be increased in presence of propranolol (blocks b1 and b2) ​ ● Propranolol + epi will lead to increase in BP without tachy ​ ● Block both b1 and b2 but mostly b1 (mosby) ● Propranolol by itself causes bronchoconstriction, heart block

3. Diagnose Lichen planus(pic from mastery day 2 file)

Combined remembered Qs 2020

4. Benzodiazepine preferred over barbiturates reason? ● Barbitures are more potent ● less addiction ● Benzo doesn’t produce hangovers ● Barbiturate serious drug dependance ● are much safer that benzo ● Less resp depression ● Barbiturates have greater sedative effect and hypotonic Thiopental most common used barbiturate

5. Benzodiazepine MOApotentiate the inhibitory effects of GABA. Enhance the effect of ​ ​ GABA Sulfonamideinhibit gaba ​ ​

6. Naltrexone used to treat addiction. Also methadone ​

7. Flumazenil benzo antagonist ​

8. Prazosin a1 blocker. Ending in “osin” is a1 blocker ​ α1 blockers are used to treat hypertension, heart failure, and benign prostate hypertrophy (α1 blockers cause vasodilation, reduce afterload and preload of the heart, reduce contraction of Qsmooth muscle in the sphincter and trigone muscles of the bladder, and reduce contraction of the prostate).

Combined remembered Qs 2020

Adverse effects of α1 blockers—hypotension (especially first-dose effect), fluid retention, dry mouth, nasal stuffiness.

9. Tetracycline bacteriostatic, inhibit protein synthesis 30s ​

10. Broad spectrum side effect? Opportunistic infections, superinfection commonly candidiasis and resistant bacteria.

11. Drugs causing xerostomia atropine, amphetamines, benzo, anticholinergic, ganglionic ​ blockers, antidepressant ● TCADoxepin, Imipramine,Amitriptyline Inhibit reuptake of serotonin and ne ​ ​ ● Phenothiazine ● Dephinhydramine ● Atropine ● Diazepam ● Midazolam Pilocarpine (head and neck xerostomia) and cevimeline (pt with sjorgen) used to treat xerostomia.

12. Antacidcimetidine h2 blocker ​ ​ H2 ranitidine or cimetidine to decrease acid ​

13. Why Opioid not safe? Resp depression

14. Metoprolol-selective/non selective selective b1 blocker ​

15. Which is both agonist & non agonist drug? Pentazocine

16. Atonolol side effect b1 antagonist ​

Which of the following is not related to a drug toxicity of Atenolol? A. CHF B. Tachycardia ​ C. AV block D. Sedative appearance

Combined remembered Qs 2020

17. Pilocarpine causes cholernergic. its for xerostomia and treats open angle glaucoma too ​ Causes: sweating, nausea, heartburn, diarrhea , miosis

18. Antimicrobial agentchlorhexidine ​ ​

19. Diphenhydramineit’s benadryl - allergic to both lido and ester ​

20. Physostigmine reversible cholinesterase inhibitor. Antidote for atropine ​ Physostigmine inhibits acetylcholinesterase, which metabolizes acetylcholine.

TCA overdose phygo ​ TCA Overdose. drug of choice is: Atropine Phenytoin Physostigmine phenobarbital

21. Amitriptyline TCA. #1 side effect is drowsiness and xero ​

TCA causes ortho hypotension, hyposalivation, drowsiness Epi and TCA cant be used together cause severe hypertension ​

a drug Amitriptyline had adverse effect, Except? a. Hyposalivation b. Ortho Hypotension c. Drowsiness d. weight gain

22. Asthma management-give oxygen, upright position,etc...in options

Combined remembered Qs 2020

Acute asthmatic attack

● Supplementation of oxygen with inhaled beta 2 adrenergic agonist (albuterol) ● If pt resists to beta agonist theophylline

IF SEVERE asthmatic attack that is unresponsive to above txt 0.3mg of 1: 1000 epi ​ subcutaneously

23. Dentigerous cyst pic identification: always associated with a crown of eneruoted tooth rd ​ especially 3 ​ molar. Lined by non kera stratified squamous epithelium, no rete pegs ​

24. Kennedy class 3 modification 1

25. Why opaque is seen above porcelain teeth Opaque showing through means: ● Inadequate tooth reduction dentist fault ​ ● Opaque layer too thick lab fault ​ ● Inadequate thickness of body porcelain lab fault ​

According DD,function of Opaque porcelain are 1.mask dark oxide color 2. gives the restoration its basic shade 3. provide porcelain metal bond

26. Porcelain failure reason: Poor metal framework main cause of fracture

Combined remembered Qs 2020

Surface micro cracks are caused by cooling stresses related to poor thermal conductivity of porcelain. Built in stressed in the porcelain contributes MOST to PFM failure.

27. Gold malleability term-brittle ​ Ductillity metal ability to easily be worked into desired shapes ​ Malleability-a metal’s ability to be hammered (compression) into a thin sheet without rupture. Malleability depends on plasticity, but is not as dependent on tensile strength as ductility. Malleability increases as temperature increases

28. Composite restoration failure in a day reason-debris on filling while curing, condense 2mm in chunks?

29. Pulp floor of composite class 2 perpendicular to buccal & lingual walls-always, only in amalgam, round line angles?

30. White spot in center of healthy max central incisor-hypocalcification, ​ ​ fluorosis,Dentinogenesis imperfect

31. Facial bow in an anterior reason-tip lingually? the facial bow in maxillary retainer used to . A.tipping maxi teeth lingually . B.produce no movement .just retain

It`s used for retain...if some relapse happens you can activate the facial bow to some extent for little lingual tipping of the anterior, but it`s not its primary function (and nobody wants that relapse happens lol)

32. Lateral periodontal cyst R/G

33. Furcation depth for max molar-4mm

34. Furcation depth for mand molar-3mm?

Combined remembered Qs 2020

35. Reason for post placement retain core ​ The post should equal the crown length or 2/3 the root length (whichever is greater).

36. Patient is 21 year old, mother financially responsible. Should you answer her questions or not or with permission of pt. Or only answer ques financially related? Answer with permission of pt, pt is overage

37. Syneresis irreversible hydro. Skrinkage when placed in humidity ​

38. Unbundling

39. Heart+kidney=Steven Johnson. Type 4 hypersensitivity Anticonvulsants/antipsychotics. Ethosuximide and valproic acid

40. Supernumerarycleidocranial ​ ​

41. Oligodontia+hypodontia=ectodermal dysplasia

42. Condensing osteitis: from inflammation. Entire root is seen rather than in cementoblastoma

Combined remembered Qs 2020

43. Gingival recession due to plaque other factors=age? Can be bc of age yes Betel nut cause everything except: Gingival recession Xerostomia ​ SCC Staining

44. Drug causing gingival hyperplasia? Phenytoin, calcium channel blockers, valproic acid, cyclosporine ● Nifidipine ● Amlopidine ● Felodipine ● Verapamil ● Dilitazem

45. Imipramine-antidepressant TCA DIA, Doxepin, amitriptyline, imipramine ​

46. Cheek sweating-Frey syndrome

47. Irreversible pulpitis-"lingering"

48. Pulp necrosis-"no response to tests heat or cold"

Combined remembered Qs 2020

49. Contraindications in cleaning implant-wet pumice ( I think this is false) ​ ​

50. NSAID without COX mechanism cox 2 only is ​ Nsaid does not affect platelets Cox 2 selective ​ ● Celecoxib ● Rofecoxib

51. Pulse oximeter used to measure? measures the saturation of o2 carried in your red blood ​ cells. Percentage of oxygen in red blood cells.

52. Which antibiotic prescribed for children-erythromycin,streptomycin,etc in options? AMOXICILLIN THE BEST

53. Intestinal polypsGardener ​ ​

54. Pt. On warfarin/coumadin, what blood test-CBC,INR,Hb? ​ ​

55. Anesthetizing canine and premolar only which nerve? No lingual in option only Inferior ​ alveolar and lingual ​

56. Facial nerve passes through facial canal, pterygomand. fossa?

57. Implant bone width can be found by which process-CT,MRI,Study models,PA? ​ ​

58. Bells palsy occurs when needle passes too far forward,backward,lateral,medial? ​ ​

59. Electrical shock pain after bitingtrigeminal neuralgia ​ ​

60. "Radiolucency"-always chronic periodontitis

61. "Sinus/Swelling"-always acute periodontitis

62. Parulis-infection/pus/abscess ​

63. Masseterbuccal flange in denture ​ ​

64. Patient management- I can't even explain the questions as my brain was in 'WTF' mode. Day 2 cases:

Combined remembered Qs 2020

● Condensing osteitis ● Angles class 2 div 1 ● class 3 ● exostosis ● melanotic nevus Melanotic macule is flat Melanotic nevus is raised ● which condition causes TMJ crepitus can be osteoarthritis aka degenerative joint ​ disease ● talon cusp

● epulis fissuratum: surgical removal before new dentures. Caused by overextension. Clinically like fibroma but histologically like granular cell tumor. ● lichen planus ● herpes labialis/primary herpetic gingivostomatitis ● increased VDO ● patient's upper lip interfering in the Pantograph ● patient on drug abuse what should you cautious to prescribe ● patient financially down as single mother what would be treatment option for upper(overdenture,rpd,implant,OTC denture liner) ● asymptomatic chronic apical periodontitis ● irreversible pulpitis ● periapical abscess

● amalgam seen on top of porcelain veneer reason(not triturated properly,not condensed properly,creep,corrosion) ● drug causing osteoporosis corticosteroids..prednisone ​

Combined remembered Qs 2020

● patient has history of prostate carcinoma, HIV +ve, HT,hypercholesterolemia, taking Ca channel blocker, warfarin-(kaposis sarcoma, reason for xerostomia glands are dry upon manipulating:medication/sjogren/etc), ● same question in another patient glands secrete very little upon manipulation reason for xerostomia, ● identify teeth with low copper amalgam, ● what do you consider dangerous in this patient before proceeding with dental treatment( bleeding,infection,etc), ● pt with hep A(dentist deny treatment comes under which poilcy-pt.mgmt justice ​ ● pt with knee replacement 6 months ago undergone osteoradionecrosis 60 Gy for some carcinoma 10 years ago do you prescribe antibiotics or send to his specialty ​ physician for prescribing ● patient with ampicillin allergy what other antibiotic do you prescribe ● pt. with generalized periodontitis, very slow improvement with very little bleeding on probing what surgical procedure do you do ● A case on aggressive periodontitis in a single tooth, a case involving furcation and its treatment. For day 2, "mastery app day 2" and "mastery day 2 dental cases file" are good sources and well enough. Make sure to manage time well. As it will for sure take time to think on cases before answering. 1 hour 50 minutes for each section. 7 cases with 49 questions and 2nd session with 7 cases with 51 questions. Kindly look at photographs & radiographs before answering any radiology questions. And same applies for O.path. because you never know where you can find the clue. That's it guys. All the best! Keep me in prayers. May we all pass! Amen.

March 6/7

It is my remembered question 6-7march The exam over all accepted , hope i will pass

Ortho Tri-Cyclen Lo (Norgestimate +Ethinyl Estradiol) which component cause dry socket: ​ ​ ​ estrogen part

Bared alveolar bone after a week of treatment and the patient complain of pain ( alveolar osteitis, acute osteomyelitis) ??!! here it should be alveolar osteitis as there is no pus and ​ ​ ​ occurred in 1 week after extraction

Prefered place for ginigval graft (palate, retromolar pad)??!! ​ ​

Combined remembered Qs 2020

Sulcular brush technique ( modified bass) ​

Stress and fear ,stress relation to pain : Fear inc pain tolerance pain, stress more pain

Endodontic treatment should be some time ,liippstressful , and the patient say he have his own ways to control stressful situation so his pain will (increase , decrease )?? If he face ​ ​ stressful situation

Tranqulizers and drugs u should avoid with alcohol

Extrusion movement 1mm per week , 1 mm per months ...... ??!! ​ ​

Histoplasmosis lesion resemble ( candidiasis , squamous cell carcinoma ) ​ ​

TB and its ulcer in oral cavity Intraoral lesion of TB seen as a)tonsilitis and ulceration b)ulceration on buccal mucosa c) involving tongue Granular cell tumor (psuedoepithetusliomatous hyperplasia)

Epulis is histologically similar to what: Traumatic fibroma Traumatic neuroma granular cell tumor

Epulis is clinically similar to what: Traumatic fibroma Traumatic neuroma granular cell tumor

Agonist ( high intrinsic activity and high affinity) and antagonist ( no intrinsic activity high affinity )

Rapport

Cleft palate and it predisposing factors , and time of surgical treatment 6-9 months( According to ADA it should be between 3-6 months )

Most common seizure type in children febrile ​

Combined remembered Qs 2020

Ttt of grand mal seizure phenytoin ​

Ttt of status epileptics valium diazepam ​

What is the meaning of conjugation of drugs adding molecule to drug ​ ​

Trigeminal neuralgia treated with carbamezepine ​

Naproxen

Epithelial attachment to implant hemidesmosones ​

Percentage of calcific metamorphosis 5-18% ​ ​

Percentage of herpes simplex 85% ​

Dentigerous cyst 1cm treatment ( enucleation , marsupalization ) In decks it says uncover ​ the crown and keep exposed

Panorama (hyoid bone , ear )

Panoramic xray defect if patient move his face for 1 sec during exposure what is the pattern of the defect can be vertical changes border changers what would happen if patient moves during panoramic 1-, one vertical line blurry plus a discontinuation of mandibular border 2-wavy mandibular border

Xray for mesiodent ( occlusal , angulated with shift technique ) ?? ​ ​

Varices of the tongue ( old age ?? , hypertension?? ) hypertension then age ​ ​

Idiopathic osteosclerosis

Bolimia prevelence most in ( male, female,black,white,hispanics,) white female 16 yrs ​ ​ ​

Culture hypersensitivity meaning ?!

Culture sensitivity test? Bacterial resistance Determine the antibiotic tt

Combined remembered Qs 2020

Thyroid storm no epi, high BP, tachy

Mederated calcified upper central at high risk for what ( file separation ? , access perforation ??! ) ​

Success rate of direct pulp capping depend on ( age , good coronal seal )???!!! ​ ​

March 13

1. Which is the best test for long term controlled diabetic patient Hba1c ​ ​

2. When using an occlusal rim what we are transferring to the articulator centric ​ relation..vdo?? - Ans:vdo

3. Why we give atropine before anesthesia - to reduce salivary secretion and bronchial ​ ​ secretion

atropine is given before anesthesia because Decrease incidence of cardiac arrest b. Decrease in resp secretion c. Muscle relaxation d. Rapid recovery e. Smooth induction

General anesthesia with halothane is commonly preceded by administration of atropine to 1. inhibit vagal overactivity commonly caused by halothane. 2. induce muscular relaxation by blocking cholinergic receptors. 3. reduce salivation and bronchial secretions caused by halothane. 4. All of the above

4. Mechanism of action of carbidopaCarbidopa decreases the peripheral metabolism of ​ ​ levodopa

5. Dementia (3 /4 questions about the symptoms)

6. Steps for delivery veneer

Combined remembered Qs 2020

7. Antibiotic against anaerobes and parasite ( metronidazole) ​

8. Pregnant lady.. which side we make her turn and which vessel we relieve ( left side / ​ inf vena cava )

9. INR index value 1 too good to 4 max to perform any

10.Calculate the mean

11.Lot of questions on case study, clinic trial and cross sectional

12.Main Difference between Medicaid: for destitute and Medicare : for elderly

13.Where should the tongue be in relation with occlusal plane on complete dent( above it ​ )

14.Uses of lidocaine as intravenous- arrhythmia for ventricular arrhythmia

15.What we give to convulsant patient (diazepam) ​

16.Hypoglycemic crisis what we give to the patient (something with a sugared drink) OJ ​ ​ juice if conscious

17.Pain control medication for recovered alcoholic options where oxycodone ibuprofen ​ acetaminophen

18.Exceecises comparing drugs A and B .. they were examples of potency and efficacy

19.Where we use cryer elevator mand molar roots ​

20.What we loose last when given local anesthesia ( pain touch pressure temp) ​ ​ lost in following order: pain→cold→warm→touch→deep pressure→motor

21.ASA classification of patient wearing a nitroglycerin patch plus taking it orally 2/3 day ​ asa 4

22.Picture with mucocele

23.Where is most common to find sialoliths Wharton subman ​

24.What type of study emite X radiation

Combined remembered Qs 2020

25.Patient limited opening pain and jaw deviate to the right .. where is the injured side right (deviates to same side) ​

26.Característic of trygeminal neuralgia

27.Best rinse for child with high Caries NaF ​

28.Uses of 10 per cent chx varnish: Articles online say root caries ​

Where do we use 10%chx varnish? 1 P&f caries prevention 2 white smooth surface 3 caries prevention 4 secondary caries prevention ​and root caries ​

29.Stages of gingivitis and microorganisms

30.A picture of a dentinogenesis type 2

31.Garner syndrome

32.Cleidocranial dysplasia panoramic xrays

33.A lot of diagnosis of endo

34.Combined perio-endo lesions EPT ​

35.Causes of xerostomia medication ​

36.What to do when detect abuse of elders: report to HHS ​

37.Pregnant patient 14yo.. should we tell the parents or not don’t need too, shes ​ pregnant

A conscious, mentally competent patient younger than 18 may give consent to his or her own medical treatment, counseling, or testing if he or she is emancipated, married, a parent, pregnant, or in an emer- gency situation. An emancipated minor can also consent to treatment of his or her child.

38.Muscles on denture molding for mandibular dentures masseter ​ ​

Combined remembered Qs 2020

39.Característics of zirconia

40.Where should we stand when taking xrays : 6 feet away

41.What we correct when we change the angulation in the articulator.. something about the occlusal cusps (NO CLUE)

The incline (angulation) of the condylar element on the articulator is anatomically related to the slope of the condylar articular eminences (condylar inclination).

42.How we make sure we obtain informed consent

43.What to do when patient feeling nauseous during nitro at 50/50 rate 100% oxygen ​ ​

44.Allergic reaction what we give .. Epineprine 0.3 ​ ​

45.Ranula picture then what’s the best treatment of recurrent

Decks:The treatment is surgical, either through complete excision or by removing the roof of the cyst. If it persists, excision of the gland may be needed.

46.Prolongated Corticosteroids teraphy consecuentes - osteoporosis and hyperglycemia

47.A picture of side occlusion patient determine the oclusal class

48.Causes of loose crown on implant

49.Causes of failure of gingival free graft Decks: disruption of the vascular supply ​ before engraftment. The second most common reason is infection. ​ ​ ​

50.Digitalis mech of action increasing cardiac contractility ​ ​

51.Best xray to evaluate bone shape bitewings ​ ​

Combined remembered Qs 2020

52.What we change if we want to increase energy on xray inc kvp ​ ​

53.Parts of the biological space and ct ​

54.How far an implant has to be of tooth next to it 1.5mm ​ ​

55.Bell’s palsy facial ​

56.Neurofibromas on tongue

57.Common location of intraorl melanoma max gingiva and palate ​

58.Face bow uses

59.What’s the support on removable

60.Identify Kennedy class picture

61.Some questions on Gracey curettes number

Universal is 2 cutting, 90 Gracey is 1 cutting 60 70 (cutting age offset and curves not like universal) Gracey working angulation more than 45 less than 90 ​

62.Etiologíc factor of warts (sexual) papilloma virus HPV ​

63.When we premedicate

64.Hypertensive patient.. suddenly blurred speech and progressive loose of conciuos what is it.. acv?

65.Angles on preparation for amalgam (round -sharp?)

Decks: Internal line angles in amalgam class 2 are round ​ Internal line angles in inlay are sharp

Combined remembered Qs 2020

66.Gingivectomy healing secondary ​

67.Lesión less than 1cm that doesn’t go away what we do ... excisional biopsy if we ​ ​ suspect malignancy then we to incisional ​

68.They asked me which antibiotic is against anaerobes and some parasites The option where : penicillin , cyprofloxacin, metro, azytro ​ ​

69.Then they asked me mechanism of action of carbidopa and the options where so long ..

70.Why is it that they combine hydroxycodone with acetaminophen: syngertic effect they potentiate eachother (from group) One says that they have diferent effects so they both together are better

March 16/17

1. Which cells are affected in most common leukemia in children? ALL, acute lymphoblastic ​ 2. Dens in dente most common? Max lateral ​ 3. What will happen if the sinus tract is not drained? Decks:cellulitis/bacteremia o osteomyelitis ​ 4. Hypodontia in which syndrome? Ectodermal dysplasia ​ 5. Wavelength associated with what? Hue ​ 6. Where to give retention grooves in class 2? Incisoaxial and gingival axial ​ 7. What is not included in informed consent? Price according to decks…can be written or oral ​ 8. Success of indirect pulp capping depends on? Success of indirect pulp capping depends on 1 age of the patient 2 exposure location 3 coronal seal 4 thickness of CAOH

9. NSAID MOA? Blocks cyclooxygenase function, inhibiting platelet aggregation. ​ 10. Why it is difficult to perform flap surgery near mandibular 1st and 2nd molar? Less attached ​ gingiva 11. Kennedy class 3, modification 1, which RPD design? Single palatal strap ​ 12. Supernumerary teeth present in? Inititation ​ 13. TB oral ulcers most common where? Tongue ​ 14. What can not be advertised by a general dentist? Specialty ​ 15. GTR can not be performed in which class of mobility? Grade 3 ​

Combined remembered Qs 2020

16. Fractured tooth fixation time? 3 months ​ 17. Minimum distance between 2 implants? 3mm ​ 18. What should be the width of the bone if we want to place 4mm diameter implant? 6mm ​ 19. Best root support FPD? Best root support FPD 1 long less bone lose 2 Short less bone lose 3 Conical more bone loss 4 Conical less bone loss

20. Displacement of odontoblastic processes caused by? Dissecation ​ 21. Most common osseous defect? 2 wall aka crater. ​ (remember hemiseptum is 1 wall, crater is 2 wall, trough is 3 wall) 22. Varicosis associated with? Hypertension ​ 23. Down syndrome will have all except? Rampant caries ​ 24. Which muscle is most likely to get pierced in IANB? Buccinator ​ 25. Xray: Hyoid bone, ghost image of earing, tongue, calculate the age of the patient by the xray 26. Which cyst is most likely to cause ameloblastoma? Dentigerous cyst ​ 27. Cocaine side effect CAUSES MYDRIASIS. All other opiods cause miosis 28. Bypass and nutrition study cohort ​ 29. Home internal bleach causes what sensitivity ​ 30. Treatment of internal resorption? RCT ​ 31. Paraphrasing questions 2 times 32. Most allergic material nickel ​ 33. Migrane treatment triptan ​ 34. Relation of teeth when saying Siliant sounds Close together not touching ​ 35. Role of zoe pack after perio surgery decks: doesn’t help in healing process 36. Learn number of rests and location of rests in all Kennedy classifications. Most prostho questions from RPD 37. All drugs cause gingival enlargement except

38. Mouth wash for down syndrome patient CHX 0.12% ​ 39. Pictures: papilloma, varicosis, leukoplakia, lichen planus

‘[p2240. Where to put pin in amalgam restoration? 2mm into dentin 2mm within amalgam Should be 0.5 to 1.00mm of dentin in between pin and DEJ The largest pin that can safely be placed should be placed Places at the linge angles or corners of the tooth (where tooth/root mass is >)

41. How to prepare posterior teeth? 42. Enamel demineralize at which ph? 5.5 ​ 43. How much exposure to the patient who has occupational exposure? Occupational 0.05

Combined remembered Qs 2020

Non occupation is 0.001

44. MRI uses which radiation? radiowaves ​ 45. Benzo MOA potentiates GABA ​

46. Whom to contact in medical emergency? 47. Biopsy for 2mm*4mm lesion excisional excisional biopsy should be done if lesion is less than 1cm (10mm)

48. Biopsy for white lesion on ventral surface of tongue incisional ​ 49. Sinus tract is associated with? Chronic apical abscess ​ 50. Marginal discoloration of crown? microleakage ​ 51. Orange discoloration of teeth due to? Poor OH ​ 52. Luxated tooth gives negative EPT result, why? Nerve endings have been interrupted ​ 53. Cracked tooth with no pulpal involvement extracoronal ​ 54. Turner’s tooth cause infection ​ 55. Cleft palate cause multifactorial failed fusion of palatine shelves ​ 56. Differential diagnosis of epilus fissuratum fibroma ​ 57. What to do when patient has burning sensation under the denture, patient wears dentures for 5 years

Decks: Pressure areas on the impression surface of dentures is checked with PIP (Pressure Indicating Paste). Use digital pressure only, one denture at a time. Special attention should be given to the hard palate and the mylohyoid ridge areas.

58. Dentinogenesis imperfecta associated with? Osteogenesis imperfecta ​ 59. Dry mouth- Sjogrns syndrome cevimeline ​ ​ 60. Most common carcinoma oral SCC aka epidermoid ​ 61. Toughest dental impression material polyether ​ 62. Xray- coronoid process ​

63. Orthostatic hypotension associated with which drug nitroglycerin ? ​ 64. Pt has sickle cell anemia and has thrombolytic crisis, what can precipitate this? COLD ​ 65. Patient has hypotension, reason for unconsciousness in middle of the treatment? syncope ​ 66. Bisphosphanates affect which cells osteoclast ​ 67. Which drug used for mild sedation most commonly used benzodiazepine as a premedication for sedation? Midazolam (confirmed in decks) ​ lorazepam diazepam especially for IV because no thrombophlebitis because lack propylene glycole

Combined remembered Qs 2020

68. Question about meperidine synthetic opioid, causes mydriasis (this is an exception just like cocaine) cant be mixed with MAO, avoided in renal disease 69. Best location to place implant ant mand ​ 70. What to check first if the patient becomes unconscious responsiveness ​ 71. What to check first while placing crown internal/esthetics ​ ​ ​ Decks: When trying crown 1. Check internal 2. Check proximal contacts and Pontic ridge relationship 3. Check marginal integrity 4. Check stability 5. Check axial contours 6. Check occlusion

72. Why the crown wont fit on tooth but will fit on cast?

You notice void on occlusal of cast. Crown will a. Fit on die and not on tooth b. Fit on tooth and not on die c. Fit on both d. Not fit on either

73. How to place patient position with asthma attack upright ​ 74. Steps in formation of plaque pellicle, biofilm, material alba, plaque, calculus ​ 75. Why tetracycline used in perio inc concentrations of GCF 76. Where does the extra fluoride get deposited in the body skeletal ​ 77. Which vaccine for staff every year? Influenza ​ 78. Pharmacology: cardiovascular drugs, sedatives, antibiotics, nsaids, analgesics, antidepressants (so many questions) 79. Xray- mandibular canal

80. Treatment of alveolar osteitis Decks: Flush out debris with slightly warmed saline solution- gently!!! ​ ​ ​ ​ ​ ​ ​ ​ ​ ​ ​ ​ ​ ​ ​ ​ ​ ​ ​ ​ ​ ​ ​ ​ ​ ​ ​ ​ ​ •Place asedativedressingin socket(eugenol).Thedressing shouldberemovedwithin 48 hours and replaced until the patient ​ ​ ​ ​ ​ ​ ​ ​ ​ ​ ​ ​ ​ ​ ​ ​ ​ ​ ​ ​ ​ ​ ​ ​ ​ ​ ​ ​ ​ ​ ​ ​ ​ ​ ​ ​ ​ ​ ​ ​ ​ ​ ​ ​ ​ ​ ​ ​ ​ ​ ​ ​ ​ ​ ​ ​ ​ ​ ​ ​ ​ ​ ​ ​ ​ ​ ​ becomes asymptomatic. Note: ​ ​ ​ ​ ​ ​ ​ ​ ​ ​ ​ ​ (1) The gauze provides an attachment for the obtundentpaste so it stays in the socket ​ ​ ​ ​ ​ ​ ​ ​ ​ ​ ​ ​ ​ ​ ​ ​ ​ ​ ​ ​ ​ ​ ​ ​ ​ ​ ​ ​ ​ ​ ​ ​ ​ ​ ​ ​ ​ ​ ​ ​ (2) Eugenol is the active component in most sedative dressings ​ ​ ​ ​ ​ ​ ​ ​ ​ ​ ​ ​ ​ ​ ​ ​ ​ ​ ​ ​ ​ ​ ​ ​ ​ ​ ​ ​ ​ ​ ​ ​ ​ ​ (3) If gel foam or another resorbable materialis used then, the dressing does not needto be removed ​ ​ ​ ​ ​ ​ ​ ​ ​ ​ ​ ​ ​ ​ ​ ​ ​ ​ ​ ​ ​ ​ ​ ​ ​ ​ ​ ​ ​ ​ ​ ​ ​ ​ ​ ​ ​ ​ ​ ​ ​ ​ ​ ​ ​ ​ ​ ​ ​ ​ (4) The medical term for dry socket is alveolar osteitis ​ ​ ​ ​ ​ ​ ​ ​ ​ ​ ​ ​ ​ ​ ​ ​ ​ ​ ​ ​ ​ ​ ​ • Nonsteroidal antiinflammatory analgesics should be prescribed if necessary. ​ ​ ​ ​ ​ ​ ​ ​ ​ ​ ​ ​ ​ ​ ​ ​ ​ ​ ​ ​ ​ ​ ​ ​ ​ ​ ​ ​ ​ ​ ​ ​ ​ ​ ​ ​ ​ ​ ​

81. Treatment of 7 years old, fractured central incisor 3 hrs ago, pinpoint pulp exposure: pulpotomy 82. GMT and Hachet angle is different ​ 83. Class III patient, What increases as age of the pt.increase- given all angles

Combined remembered Qs 2020

Snb increases with age Anb decrease with age Dye 84. Odontomas location mand molar ramus 85. Fluoride table 86. Cross allergy with penicillin which drug cephalosporins ​ 87. Prazosin acts on? Alpha 1 blocker ​ 88. From which side do you see distal caries more clear in posterior teeth 89. Favorable position of for complete dentures line between hamular notches is 5-13, ​ which is class 1 (House’s classification) ​ 90. Strength of connector in FPD is determined by? Occluso gingival ​ 91. Generalized widening of PDL in? scleroderma or osteosarcoma ​ ​ 92. Why there is space between primary teeth? 93. Drugs used to stop the saliva anticholinergics..atropine ​ 94. Calculation about perio pocket with gingival enlargement 95. Which block to give if you want to anesthesize mandibular anteriors and premolars IAN ​ 96. Which insurance where an employee can not choose the dentist of his choice closed panel ​ 97. Luting cement for porcelain crown resin ​ 98. Which structure of permanent teeth is damaged when a primary teeth gives pressure on it during formation enamel ​ 99. Beaten metal seen in skull radiograph Crouzon syndrome ​ 100. Patient receiving chemotherapy needs extraction, which lab test full cbc to check white blood count and neutrophil count 101. Amalgam failure reason moisture ( fructure mostly cavity preparation?) ​ 102. Patient got punched on right side of the mandible, which side would be fractured Left side ​ 103. Most common site of fracture? condyle ​ 104. Most imp factor that determines the success of implanting avulsed tooth? time ​ 105. In a group of 100 people, 50 has restoration, 20 had caries and 30 had both restoration and caries. What is the probability that if you pick a patient, he would have caries 50 ​ 106. There is caries still left in the middle after you prepared the cavity. How would you excavate the caries with bur large round bur 107. For class III jaw relation, what is done along with BSSO surgery lefort 1 108. Most common malignancy of salivary gland mucoepidermoid ​ 109. Which ion is blocked when we give LA NA sodium ​ 110. All are the characteristics of Kelly syndrome, except? Inc vdo ​ 111. Most common missin teeth, no molars and premolars in option max lateral ​ 112. Blue sclera seen in? osteogenesis imperfecta ​ 113. Ethics questions mostly with cases on 2nd day 114. 2nd day most of the cases were about old patients with CVS diseases 115. Patient is taking ACE inhibitor, patient urines more or less less ​ 116. What does dycal do in pulp capping Decks: Pulp capping is the placing of a sedative and antiseptic dressing on an exposed healthy pulp to allow it to recover and maintain ​ ​ ​ ​ normal function and vitality. The dressing most commonly used is CaOH 2 (Dycal). ​ ​ ​

117. Scarlet fever strep pyogenes causes strawberry tongue (inflamed fungiform) and skin rash

Combined remembered Qs 2020

118. Schedule drugs list schedule 2 is anything finishing in done, cant call in, need prescription 119. Condyles seen best in which view Reverse towne ​ 120. Drug for sialolith 121. What is aversive conditioning positive punishment for example a ortho appliance ​

Singh RQ March

1. Trephination-hole in cortex of bone ​ 2. HBa1c -uncontrolled is above 8% a. 7-7.5 b. 8-8.5 3. HbA1c a. serum b. blood ​ 4. NIOSH-ventilating nitrous oxide in office

5. Noble metal a. palladium ​ b. titanium

Noble metals are gold palladium platinum

6. Arch length is between 1st molar

Arch length is measured at the midline from a point midway between the central ​ incisors to a tangent touching the distal surfaces of the second primary molars or the mesial surfaces of the first permanent molars.

7. Most common oral cancer in

Combined remembered Qs 2020

a. white b. black ​ c. hispanic d. asian 8. Most common periodontal problem -black males ​ 9. Silver diamine fluoride-side effect is staining(read about this a little 3-4 question on ​ ​ ​ this )38% SAF is used 10.Lisinopril-cough as side effect ​ 11.Lisinopril-ace inhibitors ​ 12.Which of following is not competitive blocker - one option was erythromycin

ok so erythro is a macrolide and its competitive so that wouldn't be an answer...in general all bacteriostatic are competitive

13.Propanolol with erythromycin -hypertension and brady 14.Selective beta blocker -atenolol

· Metaprolol is also a selective beta 1 receptor blocker. Atenolol and metaprolol both ​ · Propranolol is beta 1 and beta 2

15.Diazepam-flumazenil 16.Opioid-naloxone 17.A question stating what u dont ask from guardian of asthma patient and one option was does does she need to take both sambutamol and albuterol inhalers 18.Symptomatic Periapical periodontitis-which medication for chief complaint(pain) ​ a. ibuprofen b. ibuprofen and antibiotics

No need for abx

Remember: NSAIDS are contraindicated in asthma, causes bronchospasm

19.Two question on LA dose calculation 20.Patient got mutiple carpules of 2% la 1:100000 epinephrine.got agitated due to vasoconstrictor 21.Steroids avoided in gastric ulcers. ( other option was it causes mental disturbances) ​ ​ 22.For acetaminophen u need to check patient liver status ​ Ibuprofen-->kidney

23.Bleaching question -coronal leaking do not cause rct failure,yes because most rct ​ causes hermetic seal-both statements are incorrect 24.Sickle cell anaemia patient came for extraction what is least desirable a. systemic antibiotic ​

Combined remembered Qs 2020

b. nitrous sedation

NITROUS OXIDE is NOT contraindicated in sickle cell ​ ​

25.Tetracycline bacterioCIDAL for bacteria (false statement). Tetracycline in reality is a bacteriostatic drug

Cidal mnemonic: CAMP FV

Cephalo

Aminogl

Metro

Penicillin

Fluoro

Vancomycin

26.Biotransformation is mostly done in plasma and kidney(true or false statement) ​ ​ 27.Enamel dissolution under 6.5 ph(incorrect) the correct answer is under 5.5 pH ​ ​ Enamel demineralization ph 5.5 or lower

28.Remineralisation of enamel from minerals of saliva not floride ​ ​ 29.Fluoride effect except increase phosphate in tooth ​ ​ 30.Anticonvulsant all except a. phenytoin b. barbiturates c. ✅lasix (diuretic) ​ d. carbamazepine 31.Question on sensitivity.they gave two tables with ppl with diabetes and without diabetes with their true positive and negative results.

Combined remembered Qs 2020

32.Frankfort horizontal plane ​ 33.Anb 5-class 2 ​ 34.Lack of good margin on amalgam restoration but no recurrent caries ​ a. redo full restoration b. prepare and fill only the defect

OBSERVED! 35.Mand molar lack buccolingual retention what's given for resistance a. buccal groove b. lingual groove c. proximal groove

Combined remembered Qs 2020

d. 36.What defines definite patient behavior or something like that all options saying i WILL do it but one says i MUST do it(i picked must) 37.Pvs-latex are gloves ​ 38.Spore testing positive what is done next a. check with physical indicators b. check which method of sterilization is used c. put autoclave out of order 39.How Sensor of digital rxworks- a. detects radiation b. senses radiation(check this one) 40.Scale on cephalogram tells us the magnification 41.No IR in class 1 RPD results in ​ a. tissueward movement of distal extension b. away from tissue ​ Indirect retainers: are placed as far away from the distal extension base as possible to PREVENT VERTICAL DISLODGEMENT of the base from the tissue

42.Class 3 mod 1 what's for stability a. major connector ​ b. rest 43.Major connector connect-rigidity and support 44.Full wax up of crown is done before wax cutout on metal framework of fod for uniform ​ flow of a solder b porcelain c metal alloy d retainer 45.Not in combination syndrome-inc vdo ​ 46.Irritational growth in ant maxilla while making new denture what is considered ​ a surgerb relining ​ c static denture 47.Denture induced candidiasis picture 48.AOT-ant maxilla ​

Combined remembered Qs 2020

Rx showing erupting canine(normal follicle of canine) 49.Lack of attached gingiva on distal to last tooth .what is CI a distal wedge b gingivectomy

BOTH ARE CONTRAINDICATED BUT THAT SITE DESCRIPTION IS FOR DISTAL WEDGE.

50.Microorganism with root canal a obligate anaerobes b obligate aerobes c faculative anaerobe d facultative aerobe 51.Initiation of caries-streptococcus mutans ​ 52.Progression of caries -lactobacilli ​ 53.Pit and fissure caries- a catch with explorer b subsurface opacity 54.Caries progress max at DEJ 55.Xerostomia can result in a leukoplakia b salivary gland duct infection ​ Xerostomia causes all of the following EXCEPT ​ A.Smooth surface caries

B. Candidiasis

C.leukoplakia

D. Salivary gland infection

56.Root caries incidence has increased ​ 57.Least to consider while composite post restoration a temp conductivity b esthetic 58.With time microleakage around amalgam restoration decrease ​ 59.Gum injury due to brush-recession ​ ​ 60.Least like to occur after ant correct a recession of gums at ant mand incisors b stop growth of immature maxillary incisor 61.Seal perforation-caoh ​ ​ 62.Least likely treatment of internal resorption -let it heal on own ​ ​

Combined remembered Qs 2020

63.Fibro osseous dysplasia pic - stating vital tooth 64.Epinephrine acts on a alpha 1 b aloha 2 c beta 1 d beta 2 e dopamine 2 65.Angina don’t need antibiotic prophylaxis ​ 66.Diabetes 2 ,hypertensive patient having bp 172/96 most likely to suffer what during treatment a hyperglycemia b hypertensive crisis c syncope 67.Patient with chest tightness and ache going to left arm first step A. oxygen B. aspirin C. raise chair to supine semi supine position

D. 68.Ranitidine-antacid ​ H2 69.Alginate exhibit syneresis even in 100% humidity 70.Arcin is better over non arcone a transfer of facebow b transfer of cr (don't remember other)

Mosby: Arcon-condylar elements is in lower member Condylar path is in upper member -resembling ​ ​ ​ as tmj-in this condylar guidance and occlusal plane is fixed. Non arcon-condylar elements in upper

member. Condylar path in lower-most widely used for fabrication of complete denture.

Combined remembered Qs 2020

Semiadjustable articulators—there are two types of articulators. a. Arcon, in which the condyles are attached to the lower member of the articulator, and the fossae are attached to the upper member. The mechanical fossae are fixed relative to the occlusal plane of the maxillary cast. This makes them more ​ accurate for fabricating fixed restorations, especially when an interocclusal record is used to mount the mandibular cast. ​ b. Nonarcon, which has the upper and lower members rigidly attached. The occlusal plane is relatively fixed to the occlusal plane of the mandibular cast. These articulators provide easier control in setting teeth for complete and partial dentures.

(1) Semiadjustable articulators generally use an arbitrary facebow record; this orients the cast in the anterior-posterior and mediolateral position in the articulator to anatomic average values (e.g., the use of the external auditory meatus to stabilize the bow).

71.Orbital on facebow is for ​ a. distance bw tmj n mand b. distance bw and plane ​ Axis Orbital Plane is a horizontal plane, that is formed by the transverse horizontal axis of the mandible and a point on the inferior border of the right or left bony orbit

72.Recording in protrusive jaw relation what changes are must in articulator related to ​ guide pin a. increased distance bw guide table and pin b. add material to maje it angle n touch 73.Vdo assessment sounds a S b F c TH 74.Unilateral balanced ​ a. mutual protected b. canine guided c. group 75.Freeway space calculation 76.Biological load rests in a abfraction ​ b attrition 77.Distance between implant and teeth - 1,5 mm ​

Combined remembered Qs 2020

78.4mm implant what the min required space - 6mm, one 1mm on each side ​ (buccolingually) 79.Anti Rotational elemment function in implant are to prevent rotation of: a- implant b- abutment ​ 80.Internal connection in inplant a for abutment and implant ​ b luting abutment c interimplant connection 81.Sebaceous glands in buccal mucosa -pic

82.Not a consideration while placing implant-caries rate 83.Auto Polymerization of acrylic 84.Voids and surface irregularities a-increase stress b released stress

Internal void and surface defects = 1) leads to stress relaxation 2) decreases elastic modulus 3) ​ ​ increases translucency

85.Bull- working lubl -nonworking 86.Kennedy class 3 identification 87.Not a consideration for fpd a pt not willing for ​ a. perio assessment b. b caries risk c. c cost of treatment

Combined remembered Qs 2020

88.Reciprocal arm of clasp- a cross arch stability b do not let the abutment tooth move while placing in rpd ​ The function of the reciprocal clasp arm is to ​ 1. act as an indirect retainer.

2. stabilize the abutment teeth.

3. act as a direct retainer for the distal base.

4. counteract any force transmitted by the retentive arm.

A. (1) (2) (3)

B. (1) and (3)

C. (2) and (4)

D. (4) only

E. All of the above.

Reciprocal arm Should touch tooth before the retentive arm ​

Combined remembered Qs 2020

89.Not a adv of 7/8th crown over full -retention ​ 90.Not advised if crown is not fitting on tooth preparation-altering luting powder liquid ratio 91.Biological width -2.04mm ​ 92.Epi in retraction cord- a rapid pulse ​ b necrosis 93.Supragingival finish line adv- a resistant b anterior c harm gingiva less during impression making 94.Color stability i put hema no udma there 95.Bsso nerve to be taken care of a ian ​ b lingual 96.Alv osteitis no antibiotics 97.Mand fracture-sports injury 98.Hyoid in x ray 99.Cleft lip palate most common congenital 100. DI associated with osteogenesis imperfecta 101. Mri -disc 102. Lingual frenal attachemnt -blue color a venous channels b sublingual acini 103. Bite block a save operator fingers b prevent tmj injury b prevent trismus d keep ​ ​ tongue cheek out of place 104. Incisional biopsy more than 1 cm ​ 105. Excisional less than 1 cm ​ 106. Cytological smear→ candidiasis ​ 107. Transillumination→ sialolith ​ 108. Strands of neutrophil- a. cyst b. abcess ​ c. periodontitis 109. Middle face triangle -cavernous sinus 110. Infection from canine to orbit canine space 111. From premolar to orbit canine soace 112. Malocclusion Class identifications 113. Commonly impacted ant teeth- canine 114. Congenital Missing teeth max li (no pm,molars in option) -max lateral 115. Protraction of maxilla-protraction headgear facemask 116. Occlusal splint not for traumatic occlusion 117. Cross bite correct ASAP 118. Unilateral cross bite in very young a palatal expander b cross elastics finger spring ​

Combined remembered Qs 2020

119. 26 months with 12 upper and lower carious tooth- under GA 120. -early loss of primary mandibular 2nd molar space maintenance a distal shoe b ​ ​ nance c band and loop 121. Polyp -gardner and peutz jegher 122. Problem during apposition of teeth-color changes ​ 123. Anug-blunted papillae 124. Anug -treatment local debridement Not a single question on apexo apexi all the ​ very best guys please pray for us!thanks for your support -Singh & kahlon

Another March RQ

1)Pt took aspirin nd got bronchiospam What is contra indication? all anti inflammatory Ibuprofen Pentazocaine ?

2)Acytylate cyclooxygenase Aspirin or acetaminophen ? ​ 3) dfm calculation 4)Kid diffrent in treatment than adult that kid more probe to 1 dehydrated 2 bleed easily 5)Enamel of primary teeth Most thick in Cusp tip Cusp tip and groove In prox ? 6)Pateint didnt see dentist 10 yrs what x ray indicated

Combined remembered Qs 2020

4 bitewing Full sries Panoramic ? 7)Best antibiotic for gingiva Azithro or augmentin ?!

No doxycycline no tetracycline in options 8) We do hemisection nd root amputation in all except Options Root perforate from mesial Root of mesial perforate and resorp Most of crown lost due to caries ! 9)Strongest Paladium silver Palladium gold metal Type iii gold ??? 10)Radiolucent Looks like stafne Then options were traumatic Abc Or salivary gland something 11)what component of cement contributes to adhesion? Aluminum silicate So2 And two more 12)Frenectomy u do it for all except Speech Esthetic Ortho

Combined remembered Qs 2020

High pull margin ? 13)cellulitis dr want to do drain and antibiotic and pain medication Pencillin or no treatment because of drainage 14 ) match the correct Furocimide lasix , diuretics Verapmil calan calicium canal blocker 15 ) medication no biological effect but use for preclude other medication Placebo 16) alot of competitive and antagonist and physiological please read 17) infection and LA not working why Unstable at low ph or ionized ? LA doesnt work because there is ionized form. La works in free or non ionized form. 18) pic of lichen planus , scc, something in base of sinus is root tip or mucous retention cyst , classes of ortho , post and getta perca length, 19) least cause of early teeth loss Lack of water flouride or incomplete tooth development 20) benzodiaspinase. Antagonist 21) after iv diazepum in 15 days got swell and tender in area of puncture why? 22) null hypothesis 23) after tooth exo there is netrophil Cyst or abcess 24) acute infection has single strand anoerobic or multiple strand anorobic 25) suspecious malignant what type of test I Biopsy pass through buccinator 47 IAN nd lingual for exo of lower 1st pm nd canine 48 u guess the future caries from lower ant fi Alan ala al ​ ​

Combined remembered Qs 2020

Remembered questions may 5th-7th ​

1. Supernumerary teeth: cleidocranial dysplasia ​ 2. Granular tumor most common location : tongue ​ ​

1. Kid (6ys) has unilateral macroglossia, small pink and gray bumps with fluid in them: I put granular tumor (other options) lymphangioma

1. 0.7ppm fluoride community 2. 74% u.s has fluoridated water 3. What if the following causes dry mouth and altered taste? a. Options were HTN drugs (lisinopril, sartan b. Qsymia(topiramate phentermine) I chose this one) the ot was taking it foe obesity and I looked it up afterwards and guessed correctly 4. What fracture is most seen caused by a gunshot in face or high velocity impact accident:

Combined remembered Qs 2020

a. Simple b. Greenstick

c. Comminuted (I chose this one) ​ d. Complex 5. Xray of AOT 6. Inverted Y and what it is composed of 7. X ray of fibrous osseous dysplasia 8. Who most common has an autoimmune disease; middle aged females ​

Google: Almost all autoimmune diseases affect women more often than men. ​ ​

9. Pt taking a medication for Alzheimer's. Question asked what does the medication do? I chose “targets amyloid plaque “

The U.S. Food and Drug Administration (FDA) has approved two types of medications — ​cholinesterase inhibitors​ (Aricept®, Exelon®, Razadyne®) and memantine (Namenda®) — to treat the cognitive symptoms (memory loss, confusion, and problems with thinking and reasoning) of Alzheimer's disease.

Combined remembered Qs 2020

10.Melanoma where is it commonly found? I was between: Gingiva And buccal mucosa or gingiva and hard palate ( I chose that) ​ ​

Decks: It exhibits a definite predilection for the palate and the maxillary gingiva/alveolar ridge.

11.Glass ionomer asked what it does: I said for ionic bond with dentin and enamel- bonds ​ with tooth structure 12.Asked during a checkup of elderly pts perio health and recession what else do you ​ check for…I said cervical caries. 13.It asked what is the cut off for a pedo pt ANC? I didn’t know and guessed >1200. Other optiosn were >1300 >1500 or <1000. ​

a.

Combined remembered Qs 2020

14.Hazard something…I chose OSHA.

15.How often do you go over manual something. I said annually. ​ ​ 16.Which has a high fracture strength? I chose zirconi ​ 17. Which has a high c factor? Class 1 ​

18.Picture of class 4 RPD 19.Kid has excessive Over Jet what class division? Class 2 div 1. ​ ​ 20.Kid pic what ia profile ? Convex ​ 21.If a pt wanted to do ortho to align max incisors? I chose lingual bar. Other options was nance, hawley, RPE ​ ​ 22.Described a ot xray had a beated metal appearance? I said crouzon syndrome

Combined remembered Qs 2020

23.Pt had a oral white lesion that was present for 7 days and then disappeared. I put althous ulcers 24.What medication would best treat perio? ….i put tetracycline other options were ​ ​ amoxicillin, penicillin, cephalexin 25.Pt first visit, 14yrs for checkup and prophy. Before pt informs she is pregnant. What do you say? These were the options. a. Inform the pt that you won’t do txt until she tell her parents. b. Inform the pt she will do txt, go over pregnant and advise her to tell her parents c. Don’t do txt and tell her parents d. Do txt and then tell her parents .

26. Pt had a mo amalgam placed…a few months later restoration fractures. Why?

a. Isthmus was too narrow b. Restoration at isthmus wasn’t deep enough c. Didn’t condense well. d. Didn’t bevel or smooth pulpal/axial line angle.

27. INR to do surgery-I put 2.5 ​

Combined remembered Qs 2020

28. Which of the following would cause blanching? I pur hemangioma ​ ​ ​

Important: How to distinguish between a hemangioma and a hematoma — hemangioma will blanch on diascopy, hematomas do not blanch.

29. Xray of genial tubercle

30. X-ray of lateral periodontal cyst

31. X ray of stafne defec t

32. Xray of calcified parotid( other options were ear, hyoid bone, cervical vertebrae) chose the parotid one because they described it as radiopaque and adjacent to the ramus…it couldn’t be ear.

Combined remembered Qs 2020

· 3mm by 3mm pink little thinf above canine I chose papilloma

· Described vertical root fracture and aside to J shape on x ray what else would u discover; I said isolated deep pocket depth ​

· X ray of a steep smile. I said pt chin too far down ​

· Sialothiasis most common in: wharton duct ( dem submandibular gland) ​ ​

· Pt month ago with bell's palsy: what will happen?

§ Will gradually get better ans restore some function.(I chose this) ​ ​

§ Will get worse

§ Remain the same.

After its sudden onset, the paralysis begins to subside within 2 or 3 weeks, and gradual, complete recovery occurs in over 85% of patients.

· What would cause suppuration? I said actinomycosis.

Combined remembered Qs 2020

· What causes gingival hyperplasia? Verapamil (other options were htn drugs)

· What can you take ti treat xerostomia? Prilocaine

· Firsr sign of multiple myeloma: bone pain

· Non- Vital tooth I said condensing osteitis.

Condensing Osteitis is a diffuse radiopaque lesion representing a localized bony reaction to a low-grade inflammatory stimulus, usually seen at the apex of the tooth. This process is asymptomatic and benign and does not require root canal therapy.

· Best way to take xylitol. I said chewing gum

· Pierre robin syndrome description

· Penicilin: low toxicity

· Description of addisons and most importantly they mentioned pigmentation. ​ ​

· Nitrous ans pt felt uneasy what to do? 100% oxygen

· If treatment of a pregnant patient, why lay them on the left side to avoid what? IVC ​

Combined remembered Qs 2020

· Prevalence definition ​ ​

· What doea nitroglycerin med do? I said directly cause vascular smooth muscle relaxation exert anti-hypertensive effects by causing a direct vasodilator action on the SMOOTH MUSCLE of arterioles, to produce a decrease in peripheral resistance & BP. Compensatory responses may be marked such as salt retention and tachycardia.

Adverse Effects: GI upset, headache, dizziness, and tachycardia.

· How would you decrease value? Add complimentary color ​

• Adding a complimentary color can reduce value. Violet is used on yellow restorations, which has the added effect of mimicking translucency.

• Stains are metallic oxides that fuse to the porcelain during a predetermined firing cycle

• Staining a porcelain restoration will reduce the value (as will using a complementary color). It is almost impossible to increase the value

· What age range can a kid start to brush their teeth on their own? I put range 6-9.otber ​ ​ ​ options were 2-5, 11-14.

Combined remembered Qs 2020

· Xray of pt rct done 8 yrs afo still has a radiolucent small speck. Asymtomatic. I put apical scar

· Best post. I pur serrated tapered ​ ​

Parallel-sided posts are preferred over threaded screw posts

· Description of cerebral palsy pt.

Combined remembered Qs 2020

· Descriptionu of down syndrome pt.

· What is the first thing happens after cleaning teeth? Pellicle formation

· Intra and skin nodules . Pigmentation. Neurofibromatosis.

· Pt 5yr old . avulsed tooth..do not replant

· ASA question on pt was taking nitroglycerin 2 to 3x weekly a yr ago and now only when he does extraneous exercise. What is he now. I said asa 3 I think. ​ ​

· Pt. Taking htn drug. After lidocaine was given blood pressure increased. What adrenwegic recepyoe responsible for such action? Alpha 1 alpha 2. Beta 1 beta 2 ​ ​

Combined remembered Qs 2020

Alpha 1 with ( lidocaine with epinephrine as vasoconstrictor)

· What is normal range for ped pt bp? I chose 110

Ok that is all I can remember at this time.

Good luck!!

May 15

1. All of the following are considered for a demineralized lesion except:

· Texture

· color

· age

· location

2. Associated with rheumatoid heart

· heart mumurs

· Joint effusion

3. Most important detail that would most affect the outcome of a fracture tooth

Combined remembered Qs 2020

· 48 hr delay of txt

· Fracture being far away from apical

· Larger than normal pocket

· Infection

4. Non odontogenic max sinus what antibiotic?

· Amox with clavu

5. #1 chemical or ingredient in weed: 2 options ending in anabol

6. Mumps etiology

7. All are bilateral except: sialolithiasis ​

8. Max molar with divergent roots close to the max sinus what is a method of percaution you should take for extraction?

· odontosection

· Not use elevator

· Use a small elevator that is not curved

· Open the sinus more

9. Max pm infection goes where?

10. Atenololà selective beta blocker ​

11. anthistamine moa:

· blocks histamine at receptor ​ ​

Combined remembered Qs 2020

· physiologic antagonism at histamine receptor

· Prevents histamine to develop (literally exactly same words)

12. What is false for EPT:

· Best test to determine if tooth is necrotic or not

· Helps determine pulp health

13. What is not a competitive antagonist?

· Epi

· Propranolol

· Scopalmine

· Forgot the last option

14. 4 quest of propranolol

15. Nonselective b blocker and selective b blocker..why do we select selective rather than non selective?

16. Epi+ propranolol= Inc BP and bradycardia ​

17. According to ADA What is percent of base metal 40% 60% 80%

18. Sickle anemia mutation: missense ​

Combined remembered Qs 2020

19. Down syndrome: non disjunction ​

20. Hepatitis A route of infection: food and drink ​

21. What does have a dentist have to have in order to not get sued is: competence ​

22. Nitrous oxide ventilation: niosh ​

23. Amalgam what do we not do

· Recycle amalgam

· Put it in hazards

· Something about trap

24. In charge of office materials: 1.something agriculture 2.fda (I put this) 3. Osha

25. Why does the body produce ketone bodies?

Combined remembered Qs 2020

1. Something alcohol was in options

2. Related to gluconeogenesis

3. Related to glucose I forgot

4. Don’t remember last option

26. primary support for max and mand

· Max palatal rugae and mand retromolar pad

· Max Palatal rugae and mand residual ridge

· Max Alveolar ridge and mand retromolar pad

· Alveolar ridges

27. What is the distal border of the denture? (didn’t specify sup or mand)

· Mylohyoid

· Fovea palatini

· Hamular notches (I put this not 100% sure)

· Something else which was wrong about mand

28. Picture of a tooth that was displaced lingually. Question: Tooth is lingually displaced and present with alveolar fracture. The treatment is to reposition and splint. How long do you splint it for? 2 weeks, 3 weeks, 4 weeks and 5 weeks (5 weeks bc you treat alveolar fracture for longer since its of most importance)

Combined remembered Qs 2020

29. Tension side for orthoà osteoblast ​

30. 4 year old first appt great oral hygiene what xrays?

· Bitewings based on findings (I put this)

· two Bitewings

· Pano and bitewings

· Pano only

31. Fremitus is checked in which occlusion

· Laterally displaced

· Mesially displaced

· Centric occlusion

· Centric relation

32. Cardiopulmonary resuscitation most common reason why the patient is not getting oxygen to lungsà

· mask isn’t sealed tightly (this)

· Obstruction

33. Gastric distention whyàtoo much air causing inflation to stomach

st 34. Infection of max 1 ​ pm where too à buccal space ​ ​

35. Chronic perio black male ​

Combined remembered Qs 2020

36. Trismus what spaceàsublingual, submental, masseteric, temporal (no submand or pterygomand in options J )

37. Patient felt tingle in his hands (was having MI) what do you do first?

· Establish responsiveness and call emergency ​

· Call emergency and chest compressions

38. Advantage of indirect over amalgam:

1. better facial contour

2. Better occlusal wear

39. Calculate the amount of epinephrine in 2% lidocaine 1:50,000

A. 34 mg B. 17mg D. 0.17 d.34mcg ​

40. Calculate negative predictive value (with no calculator so much fun J)

41. Calculate sensitivity

42. What is the most important thing to establish so the patient stops smoking?

· Why the patient wants to stop

· Talk about health benefits of stoping

· Tell patient he is going to die if he doesn’t stop lol

· Talk about what happened last time he tried to stop what triggered

43. What is correct about abuse in eldery

Combined remembered Qs 2020

· Patients do not report abuser because they have a sense of loyalty to them

· Most abuse is at healthcare centers

· Oral hygiene in elderly is not good overall

44. Pt had syncope whats the first thing you do:

· Wait and observe

· 100% oxygen

· Inject something saline

· Call 911 (I picked 100% oxygen, there was no tredenburg or position the patient in options)

45. Doesn’t cause loss of furcation:

· enamel pearls

· Developmental groove (I put this) ​ ​

· Endo infection

· Something periodontal related

46. Sealants shouldn’t be placed where: high risk patients, low risk patients ​

47. Topical Antibiotic supragingivally placed to reduce bleeding and plaque (true or false) part 2 of same question: topical supragingival is use as an adjunctive therapy at home with treatment for periodontitis cases. (true or false)

Combined remembered Qs 2020

48. All are signs of corticosteroids use except: Mental psychosis, inc resistance to infection ​

49. You need to know the exact correlation of nutrition and perio. There was 2 true of false question on this.

Pictures from day 1:

1. Similar ​

Combined remembered Qs 2020

2.

Same exact pic. Hemangioma

3.

1. Characteristic amoxicillin which one: low toxicity or broad spectrum ​ ​

Combined remembered Qs 2020

2. Know your cidal and static antibiotic. Easy questions about except that if you know them youll get them right. Super easy.

3. Best test for sympt apical perioà percussion

4. Taking impression inflammationà angioedema

5. Hypochlorite accident all except

1. Pain

2. paresthesia

6. Patient management question of what is a cultural advisor

· Help people feel at home from different cultures

· Advise patients about different cultures

· Establish different txts according to culture

· Don’t remember

7. EPTà a delta

8. What can help you see significance of patient bleeding?

· Pt

· Inr

· Bleeding time

· History

9. Least inflammation associated with?

Combined remembered Qs 2020

· Chronic inflammation

· Atrophic gingiva

· Aggressive periodontitis

· Desquamative periodontitis

10. A question on independent variable and dependent variable

11. Reason why implanted teeth fail?

· Replacement resorption

· Inflammatory resorption

12. Most common reason for early exfoliation of primary canine

· Trauma

· Caries

· Root resorption (In asda its arch length discrepancy so I think its this) ​ ​

· Anterior cross bite

13. 2 questions on calcific metamorphisis

14. Febrile most common in kids

15. 22 years old with fever , malaise, lymphadenopathy, multiple ulcers on tongue, palate with swollen gingiva

1 anug

Combined remembered Qs 2020

2 acute herpetic gingivostomatitis

3 marginal gingivitis

4 herpangina

16. Not in boneà nasolabial

17. Most associated with retention of crown

· Axial Taper (NOOOO)

· Surface area yes J

18. Retention of class 3 comp

19. Retention of class 5 composite

20. What do inlay and amalgam prep have in common?

21. Sjorgen all except: bilateral parotid enlargement

22. 3 years old comes to have what do you see:

· Plaque and pellicle ​

· Nothing

· Bacteria

· Nasthmiyh (idk how to spell it)

23. Serotonin syndrome= ssri+ opiod

Combined remembered Qs 2020

24. same question like ASDA

1. Weird question and options were lichen planus, goid, gus. Didn’t mention acantholysis, basement membrane of anything like that in decks. No clue (read about it)

2. Best to compare internal and root resorptionà pink color, rdx in different angles ​

3. All of these reasons are why you keep mand 2molar in mouth that’s ankylosed as long as possible except:

· Mesialization of per 1m

· Dista of 1pm

· Supraeruption of opposing tooth

· Maintain bone width

4. Pt tmj pain when opens and moves to right the jaw what do u do:

· Occlusal equilibriam

· Ortho

· Occlusal splint

Combined remembered Qs 2020

· Resposition splint

· Inject botu

5. What resembles epiphyseal plate? Synchondrosis ​ ​

6. Patient has came with red erosive like gingiva what do you NOT do as initial txt:

· Biopsy

· Corticosteroids

· OH

· Prophy

7. Secondary hyperparathyroidism: renal failure

8. 80 yr old you inject 6 carpules of anesthesia (didn’t say if with epi or not). She starts talking a lot, figity and tingiling of fingers. What is it? (ALL THE INFO THEY GAVE)

· Hyper (don’t remember if it was tension or ventilation)

· Lido toxicity

· Lido allergy

· Anxious

9. Enamel pearls what stage

10. Cracked tooth most common sign

· Cold

Combined remembered Qs 2020

· Pain on bitting and release

11. You did exfoliative biopsy and came positive with dysplasic cells what do you do next:

· Confirm with another exfoliative biopsy

· Incisional biopsy

· Excisional biopsy

· Wait

12. most important in denture:

· support and rigidity

· retention and rigidity

· rest were wrong

nd 13. Least one that makes you drowsy out of all of these: the awnser was a 2 ​ generation ​ antihistamine: Loratadine

14. Questions of antibiotics all are true EXCEPT. I did not know which one is true but I knew my static and cidal and that was enough to know the exception

15. Xylitol best inà gum

16. internal void and surface defects = 1) leads to stress relaxation 2) decreases elastic modulus (I put this) 3) increases translucency

17. Teeth respond to thermal, rest test are norma. You see mix RL and RO lesions in anteriorà cemento dysplasia

18. Which disease is recently seen most with periodontitis in the past years

Combined remembered Qs 2020

· Hyperthyroidism

· Cardiovascular disease (I put this)

19. Recent tests have shown that out of the following one is the most effective:

· Apf 1.23% for 25 secs (25 secs is wrong)

· Fluoride mouthwash

· Sodium fluoride gel tray for 1 min (I put this)

· Fluoride varnish

20. Patient has asymptomatic white lesion on cervical, not cavitated what do you do?

· Periodic evaluation

· Fluoride placement

· Remove and gic

· Remove and amalgam

21. Nsaid decrease the effect of thiazide, know this fact for a question

22. Most radioresistant

· Striated muscle(I put this)

· Nerve

23. The following when checked orally has one tooth less present in mouth:

· Germination

Combined remembered Qs 2020

· Fusion

24. Same exact question

25. Patient is anxious coming to appointment what is the least invasive treatment that you can do? ​

· Sit the patient next to a window with a nice painting lol

· Prescribe meds

· Nitrous oxide

· Let the patient talk (Picked this)

26. What type of xray is most effectiveà short wavelength and energy

Combined remembered Qs 2020

27. Mandibular denture patient complains of pain by ridge:

· Traumatic neuroma

28. All of the following are advantages of ¾ crown over full crown except:

· Retention

29. Modified ¾ crown can be used forà

· Lingually tipped mandibular molars

30. To make sure casting seats, do the following EXCEPT:

• Increase thermal expansion of investment

• Mix cement thin

• Remove internal nodule with occlude

31. Best area for successful placement of implant: ant mand ​

32. Implant implant distance: 3mm ​

33. Ace inhibitors cause HYPERkalemia, thiazide cause HYPOkalemia

34. Abx+ surgery= LAP ​

35. Drug conjugation what happens: More water soluble, more polar, less lipid soluble. It was choice ​ ​ of 3’s

36. Pregnat needs to go to the bathroom a lotàFetus pressue

37. Pt has hba1c 6.3% and taking Lasix why does the patient go to the bathroom a lot

· Diabetic

Combined remembered Qs 2020

· Medication (this, less than 6.5 to 6.0 is prediabetic, Lasix makes you pee) ​ ​

38. Advantages of screw vs cemented

Screw retained Cement retained

· Retrievability allows for crown · Takes up more space removal, facilitating maintenance · More economical (e.g., soft tissue evaluation, calculus ​ removal). ​ · Allows minor angle corrections to compensate for · Future modification capability. discrepancies between the implant

inclination and the facial crown · Access hole is through the contour. ​ occlusal table of posterior teeth or · Easier to use in small teeth lingual of anterior. ​ than screw-retained ​implant ​ · Main disadvantage is that the crown. ​ screw may loosen during function · Requires more chair time and because of excessive lateral forces, has the same ​propensity to loosen. excessive cantilever force, or ​

improperly screwed crowns. ​

· Screw retained can be shorter and take up less space

39. Bacteria 3 days: gram + ​

40. Psychotics stay how long in bodyà 5-6 days

Combined remembered Qs 2020

41. Sign of successful endoà no dentin regeneration

42. First thing you do when you reline a dentureà try on framework

43. Parkinsonà have mask facies

44. Gracey has one cutting edge, it is NOT triangular

45. Insertion of gracey angle à 0

46. Amalgam pin all except

· The bigger the pin the better the retention

· The bigger the pin the more strength of amalgam

· 0.5 axial wall

· 1 pin per missing line angle

47. They asked me like 3 questions of hybrid, microhybrid, and microfill

· Basically know that hybrid not good with esthetics and that microfill not good with occlusal wear and strength.

48. Another question of particles and what does it mean in terms of strength, the more = what, or the lest =what

49. What is the most common etiology of periodontitis

· Plaque

· Calculus

· Systemic diseases

Combined remembered Qs 2020

50. True or false question. Went something like this: Nutrional and systemic diseases have been seen to be associated with periodontitis because of these local factors …idk I forgot the rest.

51. Bisphosphonates affectà osteoclast

52. Addisonà hypotension

53. Patient opens his mouth all the way and it deviates to the left side what is the problem and what movement:

· Left side: rotation

· Left side translation

· Right side: rotation

· Left side: translation

54. Early sign of syncope: pallor

55. All are true of efficacy except:

· Refers to the attractiveness of the drug to its receptor

rd 56. Sickel cell anemia comes in for extraction of 3 ​ molar that is indicated for extraction (has pain). ​ All are indication for this patient except?

· Nitrous

· Local anesthesia

· Systemic antibiotics (why give systemic antibiotics to a regular extraction? Patient ​ ​ doesn’t need it, abx prophy isn’t indicated here and question didn’t state there was any infection associated with it)

Combined remembered Qs 2020

· Extraction and in histo was neutrophilsà abscess ​

57. Definition of nonmalefecience: do no harm ​

58. Dentinogenesis which stage: bell ​

59. Best for molding masseter: tell pt to bite down ​

60. Old patient comes to your office for maintanence what is something to keep on eye on to catch one time

· Cervical caries

61. Primer all except: I don’t remember options but it was something along these lines

· It said hydrophobic/hydrophilic don’t remember which

· Something about including the smear layer or not (I think I picked this one)

· Rest I don’t remember

62. What is important in single implant tooth replacement: smooth interface, connecting the implant to neighbouring teeth, broad contact with neighbouring teeth, I put antirotational something

63. MOA of ibuprofen

· Extrinsic pathway

· Intrinsic pathway

· Irrversible on platelet

· Reversible on platelet

64. Acron vs non acron= mand movement

Combined remembered Qs 2020

65. Eldery man losing it but sometimes mentally stable, son takes care of him. He can decide for him when he has power of attorney

66. Drug with good first pass effect: low bioavailability ​

67. All are anticholinergics except

68. All are immunosuppressive drugs except

69. What causes the least buccal-lingual resistance to lateral forces? Same exact rq, nothing missing in this rq..its completely correct

A- Two 5mm diameter splinted implants

B- Two 4mm diameter splinted implants

C- One 5mm diameter implant

D- One 4mm diameter implant (I put this)

70. Levadopa mode of action:

· Blocks dopamine

· Replenish dopa

71. Endo was easy, don’t waste time studying it

72. Doing endo txt gutta-percha extruded what do you doà Observe ​

73. Dry socket all exceptà Abx ​

st nd rd 74. 1 ,​ 2 ​ and 3 ​ ​ ​

st · 1 ​ is fl ​

Combined remembered Qs 2020

nd · 2 ​ is md ​

rd · 3 ​ is torque ​

75. Which is a true cyst: dermoid ​

76. why is important to rule out okc

a) due to potential for malignancy b) asso with nevoid basal ​

cell carcinoma c) recurrence rate d) infection ​

77. Green and oral stains: poor oral hygiene ​

78. Does not affect denture à buccinator ​

st 79. A question of ending of calcification of 1 ​ molar ​

80. Most associated with candidiasis:

· Insufficient radiation

· Chemotherapy

81. I got ZERO questions on flaps. Still surprised. ​ ​

nd 82. Most common tooth to get blocked of out mand archà 2 ​ molar ​

83. You do a flap on complete mandible, what nerve is most likely to get damagedà mental ​

84. BSSOà inf alveolar nerve most common to get damaged ​ ​

85. Caries in radiotherapy patients MOSTLY in

· 1 cervical

Combined remembered Qs 2020

· 2 occlusal

· 3 proximal

· 4 All

86. Hardest to maintain:

· Mand 2m in 7 year old

· Mand 2m in 5

87. All are part of infraorbital except: it’s a rq look for it same one

88. I got a question of stages of gingivitis and the cells of each

1. Amalgam breaksà not deep enough ​

Combined remembered Qs 2020

2. Erythroplakiaà dysplasia

3. Initiation of cariesà strep

4. Not initiation but developmentà Lacto (Are these like bible questions that always go?)

5. Hairy tongueà filiform ​

6. What age is commonly affected with Primary herpetic gingivostomatitis?

A- 1 B- 2 C- 6 D- 9 ​ ​

7. Osteoporosis seenà Thin trabecule ​

8. Delay txt of temporalà vision loss ​

9. Verrucous carcinomaà Warty ​

10. Combination syndrome all exceptà inc vdo ​

11. Screened annually forà tb ​

12. Injected didn’t work went to à mylohyoid ​

13. 2 questions on SLOB

14. Gingivectomy mostly forà suprabony pockets ​

15. Minimally attached gingivaà no gingivectomy ​

16. Mean boys and girlsà t test ​

17. Wax try in all exceptà obtain facebow ​

18. Chronic pericoronitis seen in xrayà flame shape distal to third molar ​

Combined remembered Qs 2020

19. Biological widthà 2mm junctional and ct ​

20. Initiator of acrylicà Benz by tertiary amine ​

21. Trephinationà Penetration of bone ​ ​

22. Levoà alpha 1

23. Rest of pharm was more towards what the medication causes so know the side effect s

Day 2: More challenging. You cant see pictures at all.

Case 1:

Patient diabetic, hypertensive, case said he wasn’t controlled since he hadn’t gone to the doctor in several years. He had a removable partial denture in superior from lateral to lateral. He came with

Combined remembered Qs 2020

chief complaint that food was going between his denture and he had bad breath. He had so many cervical caries…I would say about 10 cervical caries…especially in superior. NOTE: missing anterior teeth (lateral to lateral) had significant bone loss and great depression.

Question #1: Pic that they presented was something like this only on anterior though since he was missing anterior teeth…it was a class kennedy 3 division 1. It had some red in the space of 1 of the endetulous portions of the missing teeth. The red outline was less intense than this picture and had the same outline as the partial.

What is the reason why patient presents this?

1. Candida (I chose this, same description as denture stomatitis)

2. Food impaction

3. Allergy to material

4. Some other crazy answer not applicable

Combined remembered Qs 2020

Question #2:

What is NOT of importance when placing implants in anterior missing teeth:

1. Bone loss

2. Systemic condition

3. Caries hygiene

4. Palatal gingiva

Question 3:

They asked me what classification the patient was: Class 3 modification 1

Combined remembered Qs 2020

Question 4:

They asked me what was the RL inside the canal of superior canine. They said: look at both radiographic images taken from 2 different angles.

1. Internal resorption

2. Occlusal caries

3. Smooth surface caries

4. Amalgam

When you see it initially it looked like an internal resorption but when you see the other xray the RL moved places. Internal resorption does NOT move with xray it stays in place. Patient did not present with caries in occlusal on clinical picture nor on dentigram…but did have a huge cervical caries which was seen on dentigram and clinically so I picked that.

Case 3: I don’t remember this one just remember the patient was a COPD patient. What ASA classification is he? 3

Combined remembered Qs 2020

Case 4: Ortho case a girl 10 years old. Her canines were impacted.

All will be included in the ortho informed consent except A. Ortho treatment can bend roots of the ​ ​ teeth B. Caries and gums dieses can happen during ortho treatment ​ ​

Why is her maxillary deviated to the right?

1. Not enough space in anterior (this was answer)

What Is the RO line that crosses her max incisors in cephalon? Palatine process

Ruler? Magnification

They asked me which of these has the highest certainty:

2. Randomized clinical trial

3. Case control

4. Case series

5. Another weird one I have never seen (it was not Meta analysis)

Combined remembered Qs 2020

Her profile according to cephalo? Convex

What occlusion is she? Class 2

Case #5: this was short

She is epileptic. Patient takes Dilatin and a thiazide. Chief complaint: I don’t like my teeth they look short. (How about your gums lady?) The picture looked like this but less severe

All of the following are treatment options except:

· Crowns

· Gingivectomy

· Gingivoplasty

· Tell patient to stop dilatin immediately (you are suppose to contact physician..shes epileptic you cant remove the medication…only speak to physician to change it)

All are symptoms histologically of this patient except:

· Acanthylosis

Combined remembered Qs 2020

Case #6: pt only had 3 teeth left in mouth. Came for a total denture.

Question:

You will place implants on anterior for more retention all are indication except:

· Sinus lift in ant

All of the following is true except:

· A partial denture will have same retention as implant ​

Miscellaneous day 2:

Combined remembered Qs 2020

Endocrine stored? Ant pituitary

All are treatment options for this patient except: full mouth srps (majority of her probing was 2’s why would you do that)

Ant 1/3 of tongue taste which nerve

Post 2/3 of tongue sensation which nerve

Which has the most difficult prognosis long term:

· Mand molar with class 2 furcation

· Other mand molar with probing of 5 and 6

· Tooth with mobility

· Don’t remember last option

There was a case that there was brown pigmentation of the gingiva and it said which of the following medication cause this: no clue.

Combined remembered Qs 2020

May 20 -Mmm I felt it strong in pathology And pharma

1. Why is Atropine good for Pre anesthesia? Vagal reflex inhibition ​ a. Atropine-a powerful anticholinergic that blocks saliva production in the salivary glands. Other anticholinergics have a similar action. 2. Reason for Amalgam fractured at the isthmus 6 week after placement 3. Keratocyst 4. Nevoid basall cell sx

Combined remembered Qs 2020

5. Gingivectomy contraindications = esthetics ​ a. Contraindications: infrabony pockets (defects) and lack of attached tissue. Limitations include compromised esthetics with longer teeth, lack of access to bony defects, and having a broad, open wound post-surgically. 6. Best way to avoid fracture In amalgam MoD or something =Beveling axiopulpal ​ angle 7. Alpha 1 receptor action = vasoconstriction ​ 8. Gingival hyperplasia med And wasn’t there phenytoin calcium Blochers or cyclosporin. I pick carbamazepine ​ 9. Iv drug for ventricular arrythmia= lidocaine ​ 10.Drug for lidocaine overdose = Diazepam ​ 11.Why Acetaminophen and steroids work well together?I pick different MOA a. Steroid control inflammation n pain 12.Best prognosis for perio-endo, endo-perio lesons primary endo or primary perio. I pick perio primary 13.Mand molar after rct with radiolucency in the furcation whst to do? Hemisection, Root amputation, Bone graft. 14.Kid showing signs of abuse what you do. Call local police. Talk to parent. Call to correspondient agency

Combined remembered Qs 2020

15.Patient 14 years old pregnant Tells you to not tell to their parents What you do? 16.Best treatment for root sensitivity. Plaque control ​ 17.Days to suspend warfarin in a patien for extractions. 2 days in Mastery 18.Carbidopa moa = Peripheral inhibitory

a. Carbidopa-a drug used to treat PARKINSON'S DISEASE, but only works when combined with LEVODOPA (treats Parkinson's Disease to replenish the brain's supply of dopamine, which is the deficient neurotransmitter in Parkinson's. Levodopa (L-dopa, Dopar)-in combination with carbidopa, is the precursor of dopamine, and MAIN TREATMENT for Parkinson's Disease. Administered with carbidopa to increase its effectiveness and reduce adverse effects.

19.Non inhibition of wall synthesis = azithromycin 20.Best flap for visualization of apex = trapezoidal

Combined remembered Qs 2020

Combined remembered Qs 2020

21.Srp indication =I pick >4 pockets 22.Who Invented the universal standard care something like that = Cdc ​ 23.Opoid most common side effect = Constipation ​ a. drowsiness, cns depression, miosis, euphoria, dysphoria, dry mouth…

Combined remembered Qs 2020

MAY 21

Had around 5 questions about diagnosing pulpal and PDL health on day one. Almost all cases on day two gave you information on diagnosing correctly a tooth. So my advice is know the differences between every type of pulpitis and periodontitis. It’s not difficult to know the answer. CROSS OUT the answers you know aren’t true and you will usually only be left with one. It’s not about having all the characteristics of the disease it’s about being the closest to it. So please relax, read the questions well and a simple process of elimination goes a long way. They throw in a few occlusion questions that are somewhat confusing but don’t mind them, it is just about 4 questions out of 500 so mark it and come back to it and give it your best guess if anything. Lol The files I recommend to read is caterpillar, Dan man (make sure it’s a corrected version, the first one I got was full of mistakes) and patient management file (there is two of them one has more information). But definitely pick your source carefully, that is the base of your prep and you need it if you want to understand the questions given and correlate the answers. PS: these questions are going to be worded either the same or differently but they are all going to ask the same thing! GOOD LUCK GUYS! 1. Bacteria that secretes collagenase, hemolysin for periodontal breakdown? - Porymorpha other choices: Streptococus, lactobacillus. 2. Virus that causes Oral Hairy Leukoplakia: Answer? - EBV; other choices included HPV candida and something else. 3. All are side effects of nitroglycerin? - Headaches, lightheadedness, flushing…. Etc. 4. What immunoglobin is dominant in anaphylaxis? - IgE (allerGE) 5. Antihistamines effects against histamine? - Competitive antagonist

Combined remembered Qs 2020

6. All these drugs can cause gingival hyperplasia except? - Benzodiazepine. (cyclosporine, CCB and phenytoin) 7. A question was about different types of bone lesions, which one wasn’t caused by inflammation, trauma or infection: - Options included osteomyelitis and condensing osteitis. Don’t remember what I had put or the last two options. (Danman file has similar question #17, it says idiopathic osteosclerosis) 8. What bone lesion is most susceptible to malignant transformation? - Pagets disease. 9. Mechanism of NSAIDS: - Inhibit COX. 10. Oligodontia/anadontia is common in: (list of syndromes) answer: - Ectodermal dysplasia. 11. Had 2 questions asking about what an antagonists are characterized for: - High affinity, no intrinsic activity. 12. What type of gold is used for a RPD? type IV ​ 13. Advantage of penicillin: a) low toxicity b) high price c) low spectrum 14. What lesion is not found in bone? a. Nasolabial cyst b. Others I don’t remember lol 15. Going from an 8” pid to 16” what happens to the intensity of the xray? a. Stays the same b. Its ½th strength c. Its ¼th strength d. 1/8th the strength. 16. How to reduce radiation? a. Cone collimation

Combined remembered Qs 2020

b. Cylinder collimation c. Rectangular collimation 17. Function of the collimation in the radiograph apparatus? - Restricts the dimensions of the x-ray beams. 18. What is the spenoccipital syndochondrosis? a. Cartilage b. Fibrous suture (I put this one I wasn’t sure tbh) c. Intramembranous bone 19. Common location of supernumerary teeth: A. Max posterior B. Man anterior C. Man anterior D. Max anterior 20. Developmental anomaly of supernumerary teeth occurs in which stage of tooth development : A. Initiation B. Apposition C. Bell stage 21. Sialolithiasis is common in = Wharton's duct 22. First step in preparing an abutment tooth for an RPD? - Preparing guide planes. 23. Most danger for an operator during xrays? a. Scatter from xray head b. 2a scatter from patient 24. Intrusion of primary central of 5mm in a 5 year old boy how to treat? a. Adjust the tooth and splint b. Extract to prevent damage to permanent ​ c. Ortho 25. The most important factor for reducing sensitivity after periodontal treatment?

Combined remembered Qs 2020

- Plaque control 26. How long to splint an apical root fracture? - 7-10 days 27. Where you cannot place an apical positioning graft? - palate 28. Ectopic canines in a young patient you most commonly see? - Gingival recession 29. During closing of the mandible, what is least important? - Relaxation of lateral pterygoid. 30. What reflex is the most important to maintain during conscious sedation? - Verbal 31. Percussion test tells you what? - Pdl inflammation. 32. Class III composite with brown borders what to do? - Replace composite 33. 1 week old composite with staining on the margins but with good internal seal? - Remove 1 mm of composite and place more composite. 34. Primary central intruded 5 mm in a 5 year old patient what to do? a. Reposition and splint. b. Leave alone and allow for it to reerupt. c. Extraction to avoid damage to permanent tooth. (I picked this one) 35. Avulsed primary tooth what to do? a. Replant b. No need to replant. 36. Whats the most important factor of an avulsed permanent tooth? a. Storage medium b. Debridement and rinse of socket c. Time 37. Common feature of Cleidocranial dysplasia?

Combined remembered Qs 2020

- Delayed eruption and supernumerary teeth. 38. Horizontal mid root fracture? - Spline and recall. 39. Mandibular root is fractured during extraction what to use? - East west (cryer) elevator. 40. When preparing a cast post what is the function of the key way? - Anti-rotation. 41. Which teeth can be successfully extracted with and IAN and lingual N block? - 1 pm and canine on the same side. 42. How to treat a fearful child? - Modeling using sibling as an example. 43. Dentist ignoring unacceptable behavior? - Extinction. 44. Telling a patient to replace amalgams due to toxicity of mercury is breaking what code of ethics? - Veracity. 45. Informed consent is a type of? - Autonomy 46. What medicine to avoid with… (here they can place any of the natural anticoagulants) ginger, gensing, ginkgo biloba, turmeric etc. ? they asked 2 different questions about the same thing with different answer choices provided below. - Aspirin or clopidrogel 47. Who’s most affected by chronic periodontitis? - African American males 48. Modified Widman flap is a? - Full mucoperiosteal flap coronal to the mucogingival junction. 49. Periodontitis least helped with antibiotic therapy? - Chronic periodontitis. 50. Systemic or local antibiotic and SRP are combined treatments for?

Combined remembered Qs 2020

- Aggressive periodontitis. 51. Medication of choice in status epilepticus in the dental chair? - Any benzodiazepine especially midazolam, diazepam or lorazepam would be the answer choice. 52. Osteomas, unerupted teeth, retained primary teeth (x ray) are signs of? - Gardener’s syndrome. 53. What bones are underdeveloped or missing in cleidocranial dysplasia? - Clavicle 54. Pigmentation of the lips and oral cavity plus intestinal polyps are signs of? - Peutz Jeghers syndrome. 55. Dementia is characterized by? - Lack of short term memory retention (short term memory loss same thing) 56. Longest acting local anesthetic in IAN block? - Bupivacaine. 57. Which of the following dental conditions is often found in Down's syndrome patients? A. Mulberry molars B. Supernumeraries C. Short, conical roots D. Hutchinson's incisors 58. To avoid porcelain fractures in PFM the junction to metal should be? - Right angle (90°) 59. Maxillary Premolar extraction forceps? - 150 60. Mandibular Premolar extraction forceps? - 151 61. Method of action of doxycycline 20 mg? - Inhibit host response or collagenase. 62. Amalgam margins over time? - Improves seal

Combined remembered Qs 2020

63. What preparation step in amalgam offers resistance to fracture during compression? - Rounded or beveled axial pulpal line angle. 64. With no barrier the person taking the xrays should be where? - 6 feet away and 90-135° from the original beam. 65. Increase in the Kvp (voltage) of the xray will produce? - Short wavelength and high energy beam. 66. Some question about a patient moving for a second during the panoramic imaging? - It was talking about the focal trough and how it would be distorted. 67. Best radiograph image to view condyle fractures? - Submentonvertix 68. Waters are used to view all these sinuses except? - Sphenoidal sinuses - The other choices were: maxillary, ethmoidal and frontal sinuses. 69. Radiographs to confirm a horizontal root fracture? - 3 PA’s and 1 occlusal. 70. Ahh the scarring from the electrical burn victim question always asked. Scarring from an electrical burn in a developing child will? - Decrease VDO 71. A pregnant woman faints what do you do? - Left lateral decubitus (turn on left side) 72. Local anesthetic safe to use in pregnant patients? - Lidocaine 73. Best trimester to treat a pregnant patient? - 2nd trimester (only do simple things like prophylaxis, debridement and emergencies, elective treatments should be postponed) 74. Cleft lip occurs in what weeks during intrauterine? - 6-9 weeks (I know I know the study material says other dates but that’s how its worded in all the exams other choices were before 5 months or after 10 months) 75. Cleft lip more common in?

Combined remembered Qs 2020

- Males - Cleft palate alone is more common in females. 76. Internal bleaching causes? - None of the answers said cervical resorption so the closest to it was internal resorption. (didn’t specify root resorption). 77. A dentist did a study of caries in patients and found that the study was faulty at predicting the disease so the study is considered? - Low sensitivity (something like that, they are trying to confuse you by making it seem like they are talking about sensitivity but read the questions well) 78. When receiving a pfm crown from the lab whats the first step? - Check the color of the crown! 79. First step in seating a crown? - Check proximal contacts. 80. Following up on a patient with SRPs and still has pockets of 5 mm what to do next? - Open debridement. 81. Patient presents with grade 3 furcation involvement in a tooth with no additional apparent bone loss what to do? - Hemisection. 82. Discoloration 1 week after placing a veneer, what could be the cause? - Microleakage. 83. Green discoloration of porcelain is caused by? - Silver 84. All are effects of opioid except? - Diuresis 85. Gustatory sweating? - Frey’s syndrome 86. LD50 and ED 50 is the? - Therapeutic index. 87. Slight anterior crowding and canines are erupting what to do?

Combined remembered Qs 2020

- Stripping (enamel disking) 88. Best antibiotic to give a patient who is allergic to penicillin for endocarditis? - Clindamycin 600 mg (child 20 mg/kg) - If not there then azithromycin 500 mg (child 15 mg/kg) 89. Diazepam antagonist? - Flumazenil. 90. Morphine overdose antidote? - Naloxone. 91. New patient taking amantadine, what class of drug is it? - Antiviral. 92. 7 year old boy, new patient walks into your office and is taking amphetamine, what is the most likely diagnosis of the disease? - ADHD (attention deficit hyperactive disorder) 93. Best analgesic to give a pregnant patient? - Acetaminophen. 94. Picture of a sinus tract and a radiograph with a periapical radiolucency, what’s the lesion on the gingiva? - Parulis (know this term, it’s an old term for sinus tract) 95. Not a question but rather know the reason they use epinephrine/vasoconstrictors in local anesthetics, they had a question ask all are desirable effects of the vasoconstrictor in lidocaine except? 96. Compression rate in an adult? - 120 x min 97. Gingival index is considered? - Ordinal 98. Which interleukin has been found recently associated with periodontal loss? - IL 1 99. Minimum amount of tissue needed for a 4 mm width implant? - 7 mm (no answers said 6)

Combined remembered Qs 2020

100. Critical pH for demineralization of enamel? - 5.5 101. Critical pH for demineralization of flour remineralized enamel? - 4.5 102. White spot on enamel, fades when moist? - Incipient carious lesion. 103. Cheek biting in denture when? - Inadequate horizontal overlap in molars. 104. How to correct cheek biting in a denture? - Increase horizontal overlap in molars. 105. Kennedy class III gets support from? - Occlusal rests (tooth borne device) 106. Thumb sucking patient has a fixed oral appliance placed, this is considered? - Positive punishment. 107. Axial walls in MOD cavity for in cast onlay? - Convergence towards pulpal floor. 108. Best type of allograft material? - FDBA 109. Maximum concentration of nitrous oxide in children? - 50 is absolute max; 30 was the answer given. 110. Cariogenic bacteria are found in? - Non shedding surface. 111. Most common dental emergency? - Vasovagal syncope. 112. Maximum amount of carpules for a cardiac patient with lidocaine 2% 1:50 000 - 1 (1:50 000 = .036 mg) max dose is .04 113. Oxygen is administered in all dental situations except? - Hyperventilation syndrome. 114. Opioid overdose:

Combined remembered Qs 2020

- Miosis! 115. Wavelength of color? - Hue. 116. Prevalence of cleft lip in USA? - 1/700 117. Patient presents with flabby and mobile anterior ridge in maxilla, how to take impression for new dentures? - Passive 118. Vertical angulation in radiograph produces? - Elongation or foreshortening - Memorize one of them and the other is the opposite. 119. Chemical disintegration of enamel? - Erosion. 120. Reason for taking shade of tooth before placing rubber dam? - Dehydration of tooth. 121. Irreversible index? - DMFT 122. Most common site and cause of mucocele? - Lip and trauma. (separate questions) 123. The mandibular division of the trigeminal nerve passes through what orifice in the skull? - Foramen ovale. 124. Dens in dente is most commonly found in what tooth? - Maxillary lateral incisor. 125. Property of a drug to be able to increases tendency to cross membranes? - Non ionized/ high lipid solubility. 126. Most common cyst of the oral cavity? - Radicular cyst 127. Strawberry tongue, fungiform papillae involvement? - Scarlett fever

Combined remembered Qs 2020

128. Fluoride to use in a patient with implants and porcelain? - Sodium fluoride. 129. Treatment of choice of posterior crossbite in mix dentition patients? - Fixed maxillary expansion devices. 130. Lingual flange determinant of the mandibular denture? - Mylohyoid muscle 131. Distal extension of a mandibular denture is determined by the action of what muscle? - Masseter. 132. Most common donor site for a gingival/ CT graft? - Palate 133. Patient has endo lesion and periodontal involvement what is the correct order of treatment? - Endo first perio second. 134. The use of lateral positioning grafts (pedicle graft) is limited by? - The amount of attached gingiva from adjacent tissues. 135. Gingival attachment of implants are given done by? - Hemidesmosomes 136. Type of osteointegration for an implant? - Osseous 137. The acts as a semi permeable membrane through which? - Fluids from the gingiva seeps into the sulcus. 138. Retention for a cast crown is given primarily by? - Axial taper 139. Calcification of permanent 1st molar occurs? - At birth 140. MOA of carious detection dyes? - Stains infected dentin. 141. The function of occlusal rest? - Support

Combined remembered Qs 2020

142. The depth of an occlusal rest in posterior teeth is? - 1.5 At the marginal ridge and deeper into the fossa. 143. Treatment of choice for trigeminal neuralgia? - Carbamazepine. 144. Pulp test for a tooth with a crown? - Thermal testing 145. Premolar most likely with 3 canals? - Maxillary 1 PM 146. Patient denies having an addiction to smoking, is considered to be in what phase? - Precontemplation. 147. A graft from another species (pig) is considered a? - Xenograft 148. Patient taking anticoagulants needs an extraction, how to control dosage? - Consult with primary physician about altering patient dosage. 149. Occupational max dose of radiation per year? - 0.05 SV or 50 mSV 150. Blue sclera, osteogenesis imperfecta and obliterated pulp canals associated with what disease? - Dentinogenesis imperfecta. 151. Best pulp test for newly erupted tooth? - Cold test 152. Caries zone of enamel appears on xray is what zone? - Body 153. Concentration of acidulated fluoride? - 1.23% 154. What type of material is the receptor of the anode in the xray machine made of? - Tungsten 155. Edentulous patient with candida treatment? - Nystatin

Combined remembered Qs 2020

156. What property of color increases with age? - Chroma 157. Space for 2 and 3rd mandibular molars is done by? - Bone resorption of anterior ramus. 158. Primate spaces in maxilla? - Canine and lateral incisor (mesial of canine) - Mandibular (distal of canine) 159. 2 patients with same amount of bone loss from periodontal disease, who has a better prognosis? - Older patient. 160. Treatment of alveolar osteitis? - Saline rinse and Medicated dressing 161. Distance between 2 adjacent implants? - 3mm 162. Hands foot and mouth disease (and herpangina) caused by what virus? - Coxsackie virus 163. Pka of a local anesthetic affects? - Onset of action. 164. Analgesic for an alcoholic? - Ibuprofen 165. Stress and illness are often related. The best description of their relationship is which of the following? Jk a. Stress is primary cause of illness b. Illness is adaptation to stress c. Stress is a psychological reaction d. They often occur together but are casually unrelated e. Stress is contributory to illness and illness is usually stressful 166. In pursuit of what the dentist believes is best for the patient, the dentist attempts to control patient behavior. This is known as?

Combined remembered Qs 2020

a. Autonomy b. Competence c. Maleficence d. Paternalism 167. HIPAA was designed to a. Ensure the security and privacy of health information ​ b. Provide insurance coverage for providers c. Increase hospital testing ability d. Encourage employees to stay in their jobs to retain insurance coverage 168. MOST APPROPRIATE TEST TO DETERMINE THE DIFFERENCES BETWEEN TWO MEANS? a. CHI SQUARE TEST b. T TEST ​ 169. Information about subjects in a study included their ethnicity. What level of measurement is ethnicity? a. Ordinal b. Nominal ​ c. Ratio d. Interval

170. A moderately mentally challenged 5-year-old child becomes physically combative. The parents are unable to calm the child. Which action should the dentist take? a. Force the nitrous oxide nosepiece over the child’s mouth and nose. b. Hand over mouth exercise (HOME).

Combined remembered Qs 2020

c. Discuss the situation with the parents. ​ ​ d. Firm voice control. 171. Which of the following is the principal nonverbal cue that two or more people can use to regulate verbal communication? a. Eye contact b. Verbal communication c. Gentle touch d. Facial expressions 172. Which is the most important feature of systematic desensitization? a. Muscle relaxation b. Cognitive restructuring c. Exposure d. Education 173. If a dentist is stuck with a needle white treating an HIV infected patient, which should he perform? - Antiretroviral therapy 174. Untreated caries in permanent dentition? - African Americans 175. Children with most prevalence of caries? - Hispanics 176. All are ways of sterilizing in the autoclave except one which one is the exception? - Solid metal box. (other answers included plastic pouches, bags) 177. Which term listed below measures the proportion of those without disease who are correctly identified by a negative test? - Specificity 178. I had a question ask about identifying the independent variable in a study. Not the same from the files. (read the patient management files and look out for a question about a question asking the same thing and know how to determine the variable)

Combined remembered Qs 2020

General notes rubber dam gives a black background appearance, affecting the Shade selection Everything will llook whiter than it actually is The attachment of the JE to the tooth surface can occur on enamel, cementum, or dentin. The position of the EA on the tooth surface is initially on the cervical half of the anatomical crown when the tooth first becomes functional after tooth eruption. ferrule means "a metal ring or band around a slender shaft that prevents splitting". In it basically means a ring of solid tooth structure going around your crown prep. I believe most prosthodontists recommend 1 mm minimum of good tooth structure in order to put a crown on a tooth in order to have a decent prognosis (this doesn't include the build-up). I may be wrong about the 1 mm minimum- you definitely want as much tooth structure as possible going around the tooth. This creates what is referred to as a "ferrule effect" which improves retention and resistance, exactly and the longer the ferrule the more the resistance to fracture! welcome...check mosby p 27 as well to get it.. In Davis Crowns a Post + Core is Casted where core covers 1-1.5 mm on to prepared coronal tooth stump to prevent vertical splitting of root due to wedging effect of cast post= Ferrule effect *Forceps # 89# Maxillary right molar 90# Maxillary left molar 17# Mandibular molar 23# Mandibular molar roots 210# Maxillary 3rd molar 222# Mandibular 3rd molar 150A# Maxillary premolar and molars 151A# Mandibular premolars

Combined remembered Qs 2020

150S#Maxillary primary teeth 151S#Mandibular primary teeth 65#Max premolars 286#Max premolars,incisors and root tips Whats penumbra??? Smaller the focal spot area sharper the image appears Fuzzy unclear area that surrounds a radiographic image is called as penumbra So ↑ in source to object distance ↓ in film to object distance. Smoking is not a contraindication for the placement of dental implants, however, failure rates are higher in smokers. The failure rate is related to the amount of smoking on a daily basis and the pack/years history for the patient. Based onon angle formed by cutting edge with edge of the blade. Away from the handle distal GMT Towards the handle mesial GMT. BURS-: 330- pear shaped, 34 inverted cone, 6-round, 558 straight fissure cross cut. round- 1/4, 1/2, 1, 2,3,4,5,6,7,8,10 Inv cone- 33 1/2, 34,35,36,37,38,39, 36L, 37 L Straight plain- 56,57,58,59,60, 57L,58L Straight cross cut- 556,557,558,559,560,557L, 558L Tapered fissure plain- 169, 170,171,169L,170L, 171L taperd cross cut - 699(l)700 (l),701,702,703, End cutting-957,958 Wheel-14 Pear- 329,330,331 331L, 332 It's dried blood. Someone mentioned on the earlier posts only dripping wet blood comes under regulated waste; went by that explanation y-shaped in antral. I remember someone had got this in their exam.

Combined remembered Qs 2020

At low doses, nitroglycerin will dilate veins more than arteries (reduces preload, primary mechanism of action), but at higher doses it also dilates arteries (afterload reduction) and is a potent antihypertensive agent. In cardiac treatment, the lowering of pressure in the arteries reduces the pressure against which the heart must pump, thereby decreasing afterload.[18] Dilating the veins decreases cardiac preload and leads to the following therapeutic effects during episodes of angina pectoris: subsiding of chest pain, decrease of blood pressure, increase of heart rate, and orthostatic hypotension. Ectopic eruption sequences: Its max first molar>max canine>mand canine>mand second premolar>max lateral.... Most common to least Hep C. Also associated with chronic liver disease, hepatocellular carcinoma and the no 1 cause of liver transplantation in US Other names for alarm clock headache: Cluster headache, Horton's headache, red neuralgia, histamine headache, and Sphenopalatine neuralgia best post= parallel and best pin= self threading Biostatistics in Pt management * Validity....> should be compared to gold standard and should be high sensitive,specific and unbiased *Realibility....> should be reproducible and repeatable with same value means produce similar results * Sensitivity....> % of persons having the disease TP/TP+FN X 100% * Specificity ....> % of persons not having the disease TN/TN+FP X 100% * predictive value positive....> TP/TP+FP X 100% * predictive value negative....> TN/TN+FN X 100% Highest prevalence of caries = Hispanics Highest DMFT = White (caucasian) (highest amount of restored teeth) Highest untreated primary teeth = Hispanic Highest untreated perm teeth = Black (African American) Moderate periodontitis = Black males ( African American) Class II caries = Whites (caucasian)

Combined remembered Qs 2020

Class III caries = Blacks (African American) Cleft lip/palate w/ Class III occlusion = Native American Cleft lip alone = Asian Cleft lip in USA = 1:700 to 1:800 class 2 malocclusion : whites of northern European descent class 3 malocclusion : Asian Caucasians have more lip cancer while African american have more oropharyngeal carcinoma. Anterior open bite: African American(blacks) Deep bite: cuacasian( whites) Cemento osseous dysplasia - black middle aged wome so caries is generally low in down sy and periodontal disease are high in these patients. At the same time in case of cystic fibrosis due to medication (antibacterial specially tetracycline) and composition of saliva caries is very lowe(lower than Down sy). If you had both, then choose CF ,I red this today in oral patho il-8 attracting inflammatory cells, TNF similar to il-1 but less potent, mmp 8 is collagenase so they involve in periodontal tissue destruction. Minimum thickness of a connector is 3 mm Mesiodistal area is shared by pontics and connectors ..In Class 2 -->Amalgam (internal line angles are rounded) but in case of Class 2 -->Inlay (internal line angles are sharp) .In working side interferences:BULL rule (inner inclines) In Balancing side interferences:Mandibular Buccal Cusp (inner inclines)------>although they are Secondary Centric Holding Cusp we never grind Maxillary lingual Cusp (Primary Centric holding Cusp) Supracrestal fibrotomy B is particularly for rotations..while D keep the contacts wide is for lower anteriors so as to make sure to prevent slippage of contacts...as given by Raleigh Williams and I think also in Garber...I ll go with D... For Cusp Protection via GOLD

Combined remembered Qs 2020

a.Capping is done in Functional cusp b.Shoeing (Veneer) is done in Non functional cusp except Buccal cusp of MAXILLARY PREMOLARs and MOLARs for minikin ..its 3 mm minim ...5 mm cross bite:----Dentoalveolar: Inclined plane, Reverse SS Crown, Tongue Blade, Maxillary Hawley with Z-Springs, and Posterior Bite Block Combination to discclude the anterior dentition Functional... check for the premature contacts.... Most likely they are primary canines which are the culprits as they haven't attritioned with rest of the primary dentition osseous resective surgery includes ostectomy and osteoplasty...... in which aim is to reduce and removve supporting and non supporting bone ...... in this first of osteoplasty includes technique of fesooning which is vertical grooving or bone festooning to reduce buccal and lingual thickness of bone interdentally...... Aldosterone release and presence of insulin causes uptake of potassium from blood stream causing hypokalemia Both true,etching time for primary teeth is 30-60 seconds. patient is aware of problem but there are some barriers like time, expenses.. so it comes under contemplating Overextended max denture __ result in sore throat Condylomata acuminata (6-11) HPV while 16 18 , 31 33 HPV for verrucous carcinoma 1 nd 2 High copper(no gamma 2 phase no loss of marginal integrity) Spherical(less hg content 40-45%) Acromegaly :Enlarged Tongue Pieer robins:Posterior tongue displacement(glosoptosis) Down:fissured Does anyone have a reference for 3?

Combined remembered Qs 2020

A crown's color is selected by looking at the adjacent teeth. And if it has gotten 'lighter' means the adjacent teeth have stained. And since a crown is selected by looking at surrounding teeth only, will we not change the crown rather then messing with all these other teeth? Flouride fatal doses Fatal dose 16 mg/kg in children 2g in adults while toxic dose 5-10mg/kg in children Agranulocytosis, you see ragged ulcer in gingiva and palat @Yeah Swiss cheese pattern in cribriform is seen in adeno ....and honey comb in acinic 1.5 mm between tooth and implant and 3 mm between two implants This was discussed here a few days ago , someone wrote that serial extraction may cause deep bite , coz of lack of support , so in case there is already deep bite present then its better not to do it it's written in decks that OSHA responsible for all the employees to get hep b vaccination and it tells cdc to take care of those vaccination..I don't remember Bonding agent contains a hydrophilic n hydrophobic strata . Hydrophilic part bonds to dental tissues n composite should bond to the hydrophobic part... since dentin contains more organics n H2o than enamel, the agent bonds better to dentin than enamel GINGIVAL CYST of newborn or adult Not seen radiologically. Not to be mistaken with Epstein’s Pearl (on median palatal raphe) and Bhon’s nodule - seen on alveolar ridge of the newborn. Operant conditioning (sometimes referred to as instrumental conditioning) is a method of learning that occurs through rewards and punishments for behavior. b, you dont remove caries when they are half way in enamel.. they are treated by flouride tx... but when it touched DEJ you have to remove it 16 Weeks between the 1st n 3rd dose n 8 weeks between dose 2 n 3. If only dose 3 is left it should be given immediately

Combined remembered Qs 2020

For ADHD patient ask patient to have the medicine 1/2 hr before appointment, rest is all same , keep short appointment and morning one, Yes most is chlorphenarmine maleate then phenargan then least is diphenhydramine Aluminum is the weakest crown material for implant abutment pneumbra, causes indiscrete margins of x-ray's film 245- cavity prep Prospective-->foward in time (starts with the risk factor and ends with the disease) Retro(back)pective--->Backward in time(starts with the Disease and asking about the risk factor in the past) iagnostic Clues Craze lines are frequently confused with cracks, but can be differentiated by transillumination. If the tooth is cracked, the light will be blocked, allowing only a segment of the tooth structure to light up; if the tooth only has a craze line, the entire tooth structure will light up,shouldn't the answer be Periapical abscess then as the tooth is intact?What is the answer? Ok,Periapical abscess seems appropriate as the whole tooth is intact n can be illuminated. low grade mucoepidermoid carcinoma has best prognosis Iopa of max canine in periapical region lateral wall of nasal fossa and anterior border of maxillary sinus form an inverted Y known as inverted Y OF ENNIS B, this is what I found Most Sensitive: Blood-forming organs Reproductive organs Skin Bone and teeth Muscle Least sensitive: Nervous system Physical restraints for moderate retarded child at emergency ... Cos voice control and HOME are for pts with normal mental activity.. GA not possible as it is an emergency visit... what's the sequence of hue value chroma when we have to match a shade? First Hue then Chroma then Value

Combined remembered Qs 2020

Wavelength dependence ------> hue Concentration dependent------>chroma Reflection of amount of light dependent-----> value For years, people with asthma and allergies have been told to avoid antihistamines because they dry sinuses and cause constriction of the airways and sedation. Newer antihistamines, including fexofenadine (Allegra), loratadine (Claritin), and cetirizine (Zyrtec) may be beneficial for mild allergic asthma, but antihistamines are still problematic and are usually not beneficial for moderate to severe asthma. People should not use antihistamines if a sinus or respiratory infection is present. Used routinely, antihistamines thicken mucous secretions and can worsen respiratory infections. It is particularly important to treat any co-existing sinus infection in people with asthma because they might not respond to asthma treatments unless the infection is first cleared up. Contemplation reason: Contemplation . pt is thinking about quitting but not enough motivated yet.. pre-Contemplation .. pt is not even thinking of quitting i think that denial and preContemplation are the same.. action. pt is actually working to quit. correct me if i am wrong ... yep... I was reading some random article and according to tht., if fluconazole isn't the given choice then ketoconazole is the answer...for systemic anti fungal According to A.D.A., the acceptable dimensions of a tooth brush are (a) 1-11/4 inch long, 5/16-3/8 inch wide, 2-4 rows, 5-12 tufts per row caries is seen on which aspect of 1st primary molar?? distal surface, occlusal, Ans is occlusal. Full ext—how long do you wait for max denture o 1 day o Same day o 1 week o T- 8 week o 16 week

Combined remembered Qs 2020

6 weeks for preliminary impression and 8 weeks for final impression The pain from the loss of pulpal vitality is the most common presenting complaint ofpatients with combined lesions. The symptoms reported are those most often found with pulpal disease. Thermal pulp testing provides information relative to the status ofthe pulp, and dental radiographs can confirm the presence of apical changes and the extent ofbone loss. Careful probing confirms the presence and morphology ofany periodontal pocket and permits location of the conmunication with the apical lesion. In combined endodontic-periodontic lesions, it is generally wise to treat the endodontic component first, because in many cases this will lead to complete resolution of the problem. After successful endodontic treatment, the residual periodontal pocket that remains can be more predictably heated. The periodontal therapeutic objeclives vary with the extent Permissive way....wid special child decisive is when its a normal child and u know he has ro capabilities to understand the reason ans is pemphigoid where there is seperation of membrane and epethilium from connective tissue whie in pemphigus there is blistering of skin due to antibodies being direted against desmosomal adhesion molecules ..meaning only the epidermis is affacted Pemphigoid -sub basilar cleft Pemphigus- intra epidermal cleft intenal: RCT, for external: if not self limitting than in most cases extraction chi-square is the sum of the squared difference between observed (o) and the expected (e) data (or the deviation, d), divided by the expected data in all possible categories. From best to worst for implantation ... D1 Ant Mand, D2 Post Mand, D3 Ant Max, D4 Post Max:)

Combined remembered Qs 2020

ECC locations start with the most affected 1. Ant max 2. Post max 3. Post mand 4. Ant mand --> least because tongue cover them unbundling is done by dentists and downcoding is done by insurance bundling : all pricedures into one -by insurance upcoding : dentist playing with codes to get more $$ If there was any other problem the margins wouldnt have fit....whenever we are fitting a crown the first thing u check is proximal contacts...then margins and then occlusion...so if the margins are ok it is understood tht the proximal contacts are ok too Its iv bisphosphonates its effect will not reduce on stopping the medication just 3 months before that too when patient is taking from 2 years, I guess second option will be best , we will definitely choose non invasive procedure coz its IV bisphosphonates Normal salivation rate is : It's 1.5 L/day = 1 ml/min a pier abutment is a natural tooth located between 2 terminal abutments for example..if u have a missing 1st premolar and missing 1st molar,the 2nd premolar would be the pier abutment that is located between the canine and 2nd molar. condyle for both...only difference is second most common...symphysis is 2nd most common in children and angle is nd most ommn in adults

To make the crown narrower , Move line angles more facially and increase interproximal embrasure mental the most difficult one,platal the easiest but painful,The buccal block is a successful injection because the buccal nerve is readily located on the surface of the tissue and not within bone In self cure: initiator- benzoyl peroxide, activator: toluidine. In heat cure: initiator -benzoyl perxide , activator -heat. Working cusp for amalgam is 2.5 to 3

Combined remembered Qs 2020

Non working for amalgam is 2.o W cusp for cast gold is 1.5 Non w cusp for cast gold is 1.0 W cusp for metal ceramic is 1.5 - 2.0 Non w cusp for metal ceramic is 1.5 -2.0 I read that people with CP have a 30-50% or 35-50% of seizure development and mental retardation/learning disability. Working side interference: reduce de lingual incline of the buccal cusps of maxilary molars and inner incline of lingual cusps of mandibular molars. Non working side interference: grind only the inner incline of the buccal cusps of madibular molars. In reality u should never reduce the primary centric holding cusps (lingual of max.) But in nbde u can reduce it if is high in centric, protusive and lateral excursion movement. Stieglitz pliers -- use for removing silver points Implanted opened apex tooth regain blood, Ans is 20 days...it will be regained within 20 days after replantation but nerve supply lags behind. Cavicide disinfectant for dental chair Is phenargan contraindicated in pregnancy Animal reproduction studies have shown an adverse effect on the fetus and there are no adequate and well-controlled studies in humans, but potential benefits may warrant use of the drug in pregnant women despite potential risks. CATEGORY C-pregnancy b . cheek bite with edge to edge relationship A dentrifices have 20-40% abrasive particles. And a abrasive paste have about 80% of abrasive particles. Also most dentrifices (tooth pastes) have fluoride incorporated... most tooth pastes have 1450 ppm of fluoride. 300-200/1000 = 0.1, incidence define no. Of new cases

JUNE 3

Combined remembered Qs 2020

Exam was okay okay. Lets hope I PASS this exam. 1-composite monomer-options were polymethy methacrylate,UDMA,methacrylate 2-common duct for sialolithiasis-stenson,bartholin,wharton,minor salivary duct 3-Radiograph showing large radioopacity in maxi anterior tooth region-options-AOT,Fibrous dysplasia,Ameloblastoma,condensing ostitis 4-pregnant patient in left to prevent pressure on-IVC 5-Case question in which pt has habit of smoking 30 pack cigrates per year.Options were- precontemplatory,contemplatory,denial,action,Maintenance 6-cryer forcep used for-extraction of mandi molar root piece. 7-what doesnt get killed in steam sterilization-options were all virus and one bacillus st. 8-minimum buccolingual width needed for 4 mm implant- 6mm 9-occlusal plane of denture is limited by-fox plane registration,1/2 or 2/3 of retromolar pad,something about tuberosity.,and other option was distal flange something like that not sure 10-best graft for osseous defect- Autograft,allograft,alloplast,dried frozen 11-premedication for endocarditis patient.- 12-what is not recommended to place the instruments in steam sterilization-paper packets,paper plastic packets,cloth pouches,solid metal containers,plastic bags. 13- broken file in one of the canal of molar what to do - obturate and follow up. 14-infection from which of the following space goes to mediastinum-pharyngeal pouches,submandibular,Infratemporal,Retropharngeal. 15-Angular chelitis caused by all expect- decreased VDO,lingual inclination of mandi Anteriors,vitamin deficiency,smoking habit 16-cells present in initial stage of periodontitis 17-question on combination syndrome but it was not straight forward to understand. 18-case and radiolucent lesion in periapical region of mandi anterior with no pain on percussion and normal response to thermal stimulus-Periapical cemnto dysplasia. 19-muscle that is in contact with disto buccal flange of mandi denture-lateral pterygoid,medial pterygoid,masseter,temporalis

Combined remembered Qs 2020

20-radio opaque structure seen in front of C3/C4 in cephal 21-how to reduce value-add blue cyan,complementory color,adjacent color,redo 22-mean question- 23-property of GIC over composite 24-warfarin -Vit K 25-identify kennedy classification picture 26-radiograph-dentigerous cyst 27-radiograph -odontoma 28-differential diagnosis of crusted ulcer on lip-phemphigus,phemphigoid,verucous vulgaris,HSV 29-xray projected @ 20 angularion from distal where is distal /mesial root located-something like that -SLOB rule questions I got 2 but they were NOT easy to answer. 30-inverted Y line radiograph and how it is formed 31-identify occlusion- 32-most commonly seen occlusion in paediatric patient 33-position to treat 2year old clild 34- 2 year old child has fear from -light,sound,instrument or eugenol smell. 35-case question slurred speech cause -ischemic attack 36-bells palsy- facial nerve 37-fibroma -hyperplasia 38-what to avoid in minimum keratinised tissue- apical graft 39-least important during closure of mandible-relaxation of lateral pterygoid 40-numbness in lower lip without any anesthesia- Malignancy 41-avulsed primary tooth- do not reimplant 42-all of the following factor contributes to reduce sensitivity of root after perio treatment except-options were 6 months maintence visit to dentist,desensitizing toothpaste, plaque control, and use of fluoride varnish 43-appilance use for orthodontic correction of 4 maxi anteriors-hawley retainer,nance button,lingual arch

Combined remembered Qs 2020

And something else 44-for maxi anterior tooth-which block should be given along with nasopalatine block. 45-identify radiograph showing linear radiolucency above the root of 1st maxi molar.-I choose PSA canal. 46-initial step in biofilm formation. 47-Y we avoid xray for a routine follow up appointments within 6 months 48-better prognosis-1 wall defect,2 wall defect,3 wall defect,4 wall defect 49-emergency treatment for endo infection. 50-indirect pulp capping when? 51-reasons for fracture of amalgam restoration except- 52-MO amalgam restoration Y did it fracture- narrow dovetail, rounded axiopulpal floor,wide intercuspal distance,etc 53-feature if gracey’s scaler. 54-bells palsy cAused due to needle placed too backward,forward,superior,inferior 55-Procedure for apical closure in permanent immature non vital tooth-Apexification. 56-one more question for apexifixation but asked in different way. 57-alginante impression material placed in water -Imbition 58-Y acrylic denture is cured after 24 hours 59-if you want to work on infectious site.how do you numb? Block or infiltration 60-pt needs to take break in between procedure- what medicine-Laxis 61-fluoride in community water-75% 62-medication of status epilepticus- diazepam. 63-quetion on drug,potency,efficency , 64-cafe e lu spots-Neurofibromatosis 65-all are feature of dementia expect- Preservation of short term memory,lose of function,reduce intelligency,and one more. 66-evaluation of 2 groups A and B with 2 drugs for same period what type of study. 67-fetal alcohol syndrome 68-percussion test is used to check-

Combined remembered Qs 2020

69-polycrystalline ceramic- Zirconia(not sure exactly what question was) 70-gun shot wound is similar to which type of fracture-simple,cominutted,complex and one more option 71-high strength /low strength ceramic 72-HPV which is more specific in oral lesions.-6,11,18,20 73-mandible follows under which tissue growth.? Somatic,neural,lymphoid etc 74-treatment for cervical caries no GIC in option 75- flowable composite- A1,A3,B2,C

June 5-6’ 2020 RQS.

Kindly read the questions very carefully guys. What we all provide each other as RQS are not the same what we get in REAL EXAM.. and now I understand why people can only provide topic names in their rqs cuz its difficult to remember all with all the options.

In my exam first 2 sections were very hard to understand the questions, difficult questions with wired options. Whatever I have provided in my rqs are no doubt the repeated question too, I realized when I start typing them, but these are still the few I remember, EXAM is of 400 questions for day 1 so many more questions are there which make u madd. Stay strong and pray for each other.

So please keep me in your prayers please please.

Just an advice for COVID-19 candidate. ​ ​

we have to wear our mask all the time, what I did--- remove the mask from ur nose when u sit Infront of ur exam screen. I feel suffocated so just do that if u feel the same.

DAY 1

Combined remembered Qs 2020

1. Major connector- support and rigidity ​ 2. Associated with rheumatoid heart

· heart mumurs

· Joint effusion

3. Most important detail that would most affect the outcome of a fracture tooth

· 48 hr delay of txt

· Fracture being far away from apical

· Larger than normal pocket

· Infection

1. Non odontogenic max sinus what antibiotic? Amox with clavu ​ 2. ingredient in maurijuna ending in cannibal or THC

3. antihistamine moa: blocks histamine at receptor ​ ​ 4. Epi+ propranolol= Inc BP and bradycardia ​ 5. Sickle anemia mutation- something really diff in language not glut to valine ….. don’t know what.. missense mutation (Point mutation) ​ 6. Amalgam what do we not do- Put it in hazards ​ 7. Othro tension side (entirely diff French language ☹ ) but ans OESTEOBLAST. ​ ​ 8. CPR I put lose airmask not sealed properly as when we doing CPR we have already clean the throat and then start CPR.. so CPR include chest compression and breathing. I marked this.. (pray)

9. There was a question on smoking who will leave and why? precontemplation ​ 10. Cause of furcation except – I put endo.. other options were pearls , grooves ​ ​ ​ ​ 11. Heamingioma pic ​ ​

12. Fibrous dyaplsai pic ​ ​

Combined remembered Qs 2020

13. Angina pain which fibers – A delta ​ ​ 14. All of the following are considered for a demineralized lesion except: ​ ​ · Texture

· color

· age

· location

15. MI treatment—diff language ​ ​ Supportive care Defibrillation and Oxygen therapy Medications Anticoagulant, Antianginal, Narcotic, Beta blocker, Statin, and ACE inhibitor Medical procedure Coronary stent and Coronary angioplasty Surgery Coronary artery bypass surgery Ace inhibitors and Glycosides like Digitalis and Digoxin (ace inhibitors captorpil)

16. Most common periodontal disease – chronic perio,, option were aggressive, marginal ​ ​ ​ ​

Papillonma and fibroma diff – The fibroma is a smooth surfaced, pink, mushroom shaped growth with a stalk that appears on the affected surface. A papilloma is also mushroom shaped but has an irregular, white, pebbly surface. A papilloma is a growth that ears in its own, not as a result of irritation. Rx: conservative exc

17. Synchrondosis – epiphyseal plate ,b/w vertebrae immovable. ​ ​ There were many many questions on FLOURIDE.. put of the world ☹☹ Cariostatic effect of fluoride ​ ​ is at calcification stage of tooth development. ■ Fluoride converts hydroxyapatite to fluorapatite. ■ Fluoride ↓ solubility of enamel. ■ Toothpaste contain 1100 ppm of fluoride.

18. LAP treatment – SRP, Abx ​ ​ ​ 19. Endo bacteria – obligate anaerobes ​ ​ ​ Before rct it is strict anaerobic But after rct it is gram + facultative anaerobes

Combined remembered Qs 2020

20. Addiosns disease- hypotension ​ ​ ​ 21. Veryyyy diff 3 questions on antogginist – competitive: atropine and propantheline, ​ ​ physiologic: epi and nitro. Naloxone : non-selective and competitive opioid receptor antagonist in ​ case of opioid overdose

22. Potency ​ ​ Response to a drug over a given range of concentrations. Depend on dose of drug; less mg for same efficacy has more potency

23. Mand denture closure on molding ​ ​ Distobucc: masseter, distoling: sup pharyngeal constrictor, buccal shelf: buccinator

24. Radio patient cervical caries - chemotherapy ​ ​ 25. Many questions on dry socket- what to do what not to do.. please read the question options ​ ​ carefully again and again.. I got excited whenever I read some topic of known question but when I read options that are entrilyyyyy different --- hahahhahah. Dry socket treatment: no abx, no curettage, warm saline and sedative dressing alveogel Dry socket can occur three to four days after an adult tooth is removed. The blood clot that should form after removal is dislodged or dissolved before the wound heals, exposing underlying bone and nerves

26. What is not a true cyst. stafne ​ ​ 27. What is a true cyst. dermoid ​ ​ 28. Nevoid basal cell carcinoma symptoms ​ ​ Minor Features:

· Medulloblastoma.

· Increased head size and large forehead.

· Cleft lip or palate, extra fingers or toes.

· Abnormal shape of the ribs or spinal bones.

· Eye problems such as cataracts, small eyes, or tumors in the iris.

Combined remembered Qs 2020

· Fibromas, meaning benign fibrous tumors, of the ovaries or heart.

· Abdominal cysts.

Gorlin-Goltz syndrome is an autosomal dominant disorder with a high degree of penetrance[1] and variable expressivity. [2] It is characterized by basal cell carcinomas, odontogenic keratocysts, palmar and/or plantar pits, and ectopic calcifications of the falx cerebri.

29. BSSO nerve damage IAN.. ​ ​ ​ ​ 30. Anterior cross bite treat immediately ​ ​ ​ 31. Cross bite. manyyy diff questions on it. ​ ​

A: applianceS to correct anterior cross bite: 1) tongue blade 2) palatal spring 3) Z-spring 4) finger spring

applianceS to correct posterior cross bite: 1) jack screw 2)hyrax 3)haas 4) hawley 5)quad helix 6) transpalatal arch

32. Amalgam fracture why? Inadequate prep ​ ​ ​ 33. Amalgam low modulus of elasticity ​ ​ 34. Gingivectomy – supra bony , adequate gingiva ​ ​

Combined remembered Qs 2020

35. What not to do where there is less keratinized gingiva specifically on distal surface of last ​ ​ tooth.. option apical position flap, and distal wedge. I picked distal wedge as distal to last ​ molar is distal wedge not apical reposition on distal.

36. No perio questions on flapsssssss ​ ​ 37. No opertiveeeeeee amalgam composite, ​ ​ 38. No classs cavity prep questions ​ ​ 39. Lots of patient management – trust me that’s ur gut feeling when u pick that ​ ​ 40. Whats the things you will do when a patient visited to u and worried about his/her ​ ​ condiotin.. show empathy and concern- It seems like you r worried about your condition, but we will look into it (something similar question in running in our old rqs)

41. Maleficence and non-male definitions with diff wordings. – understanding is important (no ​ ​ similar explains examples like we did)

42. There was a question on dopamine and Parkinson’s. ​ ​ 43. Biological width – JE + CT ​ ​ 44. Propranolol is the Prophylactic medication to angina not nitroglycerin. ​ ​ 45. C4 is the hyoid bone not C3 pleaseee keep this in mind (these 2 questions have confused ​ ​ ​ options for us in our previous discussions.

46. Question of VRF SYMPOTOMS—SYMTPOM IS COLD SENSITIVITY , sSIGN is ​ ​ ​ ​ ​ ​ ​ pain on biting .. I choice cold

47. QUESTION: What does percussion test? ​ ​ a. Presence of inflammation in PDL or not b. Spread of inflammation to periodontium from PDL or not c. Responsiveness d. Pulp vitality

48. avulsion and splint ? how many days – flexible splint, 7-10 days ​ ​ ​ 49. horizontal fracture and splint- just the duration of it, no type of forizaintal root fracture was mentioned – ​ ​ Horizontal root fractures—-> Rigid Splint, 3 months (12 weeks) . Dx: take multiple vertical X-rays. 3 vertical and 1 occlusal Coronal is 3 months Middle is 3 weeks

Combined remembered Qs 2020

Apical 2 weeks or leave

50. mastoid process red spot and periorbital echymosis, which typoe of fracture? Lefore 1 , 2 , base of skull ​ ​ ​ fracture,( so its lefort 3 base of skull fracture and mastoid process red spot means bleeding there)

51. nutrient canals xray ​ ​

52. worst solution for avulsed tooth to dip--- water ​ ​ 53. QUESTION: Advantage of a direct composite vs. a veneer? Direct composite is only 1 appointment vs. ​ ​ ​ veneer is at least 2

54. supra gingival antibiotics effective for removal of plaque? T/F – False ​ ​ ​ Gracey curette shape and details . every option is mixing read it slowly please----Semi circular, more then 45 and less than ​ 90 degree angle. semicircular.. enter at 0 then perpendicular 45 but less than 90 the blade of the Gracey ​ ​ curette is only offset by 70 degrees, giving the blade a lower cutting edge and an upper non-cutting edge

55. extraction of deflected divergent roots of upp molar, how to extract? --- section the tooth , others options ​ ​ ​ ​ were flaps and more aggressive ..

56. ortho treatment with some veener placement on anteriors EXCEPT ​ ​ -move some teeth after the veneer -move some teeth before veener ( I put this considering the EPOSOP as ortho before prostho) ​ -just veener

57. one question on pins EXCEPT , which I was discussing last days whats wrong in pin statement and ans was ​ ​ pins provide strength to amalgam as an EXCEPT question, it provides retention does not strengthen the ​ amalgam) ​ 58. leakage from rubber dam—holes too close. ​ ​ ​ 59. some questions on radio therapy and its symptoms after that.. erythema was not in option :D and I don’t ​ ​ remember too please read this dryness, itching, blistering, or peeling. Fatigue

60. QUESTION: Three factors that affect caries initiation: Substrate, bacteria, host susceptibility but options ​ ​ ​ ​ were not these .. im sorry I forget the options.

61. something about the PH of enamel which is not correct.. and some long statement was given was 6.5 and I ​ ​ chose that .. enamel: 5.5, dentin: 6.5

Combined remembered Qs 2020

62. non working side cusp was related to this and best I think was this-- mand buccal cusp lingual inclines, max ​ ​ ​ lingual cusp buccal inclines

63. what movement dis occludes full posterior teeth on movements---- mutually protected (canine guidance) ​ ​ ​ 64. QUESTION: Two things that account for a successful posterior composite restoration? The type of resin and ​ ​ ​ the type of preparation

65. few questions on ENDO 2 I think. ​ ​ ​ 66. I had many short cases on day 1 too and those are time consuming too. Please manage accordingly. ​ ​ 67. inflamtory resportion which condition it happens? ​ ​ 68. Bowl Shaped Resorption (inflammatory resorption) – involves dentin and cementum; tx – ​ ​ - immediate RCT; CaOH every 3 mo and after 1 yr, obturate with CaOH sealer; necrotic pulp

69. ankylosis replacement resportion. ​ ​ Replacement Resorption – resorption of root surface and bone causing ankylosis; often seen in replant cases; accompanies dento alveolar ankylosis, characterized by progressive replacement - of root by bone (no pdl); signs: no mobility, metallic percussion sound, and infraocclusion.

DAY 2: Case 1: Tonsillitis case: GIRL having halitosis and caries in mouth, hr class mate make fun of her due to bad breath.. Q1-common question what the cause of bad breath and I marked-- chronic tonsillitis (there were not much caries like rampant caries in mouth of more than half of her teeth are involved as carious lessions- just 5 – 6 I think carious teeth.. so I don’t think this much caries can cause bad breath.. so ​ I picked CHRONIC tonsillitis the case of bad breath Q2- what is he best oral treatment u will advise – maintaining good oral hygiene. ​ CASE 2- asthma patient with inhaler presented in clinic ​ Q1- patient had diff in breathing in start as he enters the clinic, what will you do for treatment management, - -STOP THE TREATMENT UNTIL HE IS RECOVERED with the asthma ( I marked this, as he is asthmatic patient and attack might be due to anxiety and follow this SPORT) - Continue treatment

Combined remembered Qs 2020

- delay treatment Q3- atenolol side effects- staining in teeth , xerostomia( THIS) ​ ​ Case 3: male 45 years patient has lots of missing teeth and not happy with smile, and what to ​ replace all his teeth (didn’t mention he want CD or fixed as implants after removal ) he’s just not happy with his teeth, need removal of all and Q 1- what would u advise him for treatment plan EXCEPT.

- CD (THIS)

- PD with fixed in posteriors

- IMPLANT WITH PD in posteriors of lower Q2- considering the patients demand (removal and replacement ) what is included in treatment option for upper arch EXCEPT

- Removal of 2, 5 (as these 2 teeth were only present in the upper arch)

- Sinus lift ------sinus lifting will be recommended if patient wants fixed implants as the presented Pano had left sinus and bone level almost sinus was so deep towards bone)

- Alveoloplasty (THIS I choice not sure) ​ - One more I forget Case 4 , 5 2 CASES ON ORTHO and CORSS BITE . Q-1WHATS not included in ortho treatment plans Q2-If we do the treatment and doesn’t included patients view which principal u r breaking.

- Beneficence

- Non male

- Informed consent (I choice this as informed consent is in principal of autonomy and here, we ​ are breaking autonomy of patient if not taking informed consent)

- Paternalism ALL THE BEST AND JUST A SMALL PRAYERS FOR MY PASS NEEDED. Jazakallah

It’s a PASS guys! Thank you so much for your help! My exam was on June 5 and 6!

Combined remembered Qs 2020

Rqs Paola Alvarez. Good luck everybody! Part 2 is longer than part 1, I would advice you to get some rest the night before of your exam and the night between day 1 and day 2, I felt day 2 was way longer and it took me half of the time so be ready for that!! Overall the exam is doable, most of my questions came from recent files (Strawberry, DanMan, Canine, El Maestro). For day 2 there is a file called masterday2 and unicorn. Those are important, the questions wont be exactly the same but you will be asked about certain pathologies that are mentioned in those files. 1) Question about behavior shaping. Which one is not a behavior shaping tecnique.1 of the options was “Carrot-Stick tecnique” thats the one I marked but after the exam I found out that the carrot-stick tecnique actually exists!! 2) Multiple questions about endo. They will tell you the patients symptoms and you have to diagnose if the pulp is vital or not and what procedure you would perform, I didnt find them too complicated. 3) I had one picture on day 1, it was a Lichen. 4) What does the "W" mean for clamps-- Wingless. ​ ​ 5) What forceps for a max premolar. 150 ​ 6) How long you have to wait after a bleaching to place a composite. 1 week 7) Mechanism of action of fluoride. Make enamel stronger.  What does fluoride do. Replaces hydroxil 9) A patient comes back to the office for a first follow up, he tells you the bottom denture you gave him keeps falling out when he talks. Probable cause? --Overextended borders-- 10) How many pins to replace cusps with a onlay amalgam? 1 per cusp? 11) Mechanism of action of Sulfonamides? PABA 12) Implant placement: High torque, Low speed ​ 13) Opioids antagonist: Naloxone ​ 14) They asked me 3 times this about antagonists: High affinity, no intrinsic activity. ​ 15) Axillary nodules: Neurofibromatosis 16) Amantadine: medication for parkinson

Combined remembered Qs 2020

17) What is not a sign of opioid overdose: Mydriasis. I feel most of the questions were about the exception.. 18) Angles classification: like 3 or 4 questions on that with Xrays and or Pictures and then, whats the patients profile. 19) Ear Lobe, max sinus and zygoma on Xrays 20) Medication for status epilepticus: Diazepam was not in the options so I put Midazolam 21) How many permanent teeth does a 9year old have in the mouth. 9-12? 22) Only 1 question about how much anesthetic for a 20kilos boy. 2 was not in the options so I put 1 23) There was a picture of a bridge and the gingiva surrounding one of the retainers was very swollen.. what its not the probable cause? Shade selection 24) I got like 6 questions on Hue, Value and Chroma so make sure you know those well. 25) Most important thing when placing an implant? Silly options so I put stabilization when placing it 26) Patient that has been smoking for 30 years.- Precontemplation, Action, Contemplation, Maintenance? I put precontemplation 27) Questions on Perio.. Whats the recommended time for maintenance appointments? Every 6 months?? 28) What is true about Codeine? 29) Treatment for Osteitis: Dressing and gentle wash. 30) Multiple questions about Cleidocranial dysplasia: Supernumerary and Ectodermal dyplasia: Oligodontia 31) You want to report a drug? FDA 32) Whats the first thing a dentist have to do? Stablish rapport 33) Who is more prone to Chronic Periodontitis: Black males 34) Which one is malignant? Pagets disease 35) First thing that will happen after high dose of radiation? Erythema 36) Multiple Osteomas? Gardners 37) Leukoedema (Picture) It was bilateral, Blue/grey lesion

Combined remembered Qs 2020

38) Epulis hystologically? Granuloma... Clinically: Fibroma 39) Disadvantage of oral sedation: No tritation 40) 1 month after placement of amalgam and its pigmented.. microleakage?? 41) You were about to put anesthesia and pt starts wheezing (Pt was asthmatic)? Asthma attack 42) What is not a sign of Parkinson? Good short term memory 43) What drugs cause dry mouth... Weird options 44) % of HPV? 85% 45) Distance between 2 implans? 3 mm 46) Pt taking baby aspirin(81mg)? We dont have to stop the medication 47) Whats the O in PICO questions? Outcome 48) What nerve does not give sensation to the tongue? Hypoglossal 49) What causes Osteonecrosis 50) Inform consent: When to get it? After explaining treatment plan 51) Prevalence/Incidence questions. Prevalence: Existing cases. Incidence: New cases 52) Biopsy after 2 weeks. 53) A picture of a big white lesion on the lateral border of the tongue, what biopsy: Incisional 54) Pt with CHF, what not to do.. Position him horizontally 55) Pregnant patient: IVC 56) Most potent LA? Bupivacaine 57) Patient is very anxious, you want to help him, what to do? Explain the treatment plan carefully.. prescribe him a Midazolam 56) 5mm intrusion, primary tooth? let it re-erupt 57) Epi and histamine. Physiologic antagonism 58) pKa: Onset 59) What lesion is radiolucent? OKC 60) Drug is more efficient when its free in blood 61) Ludwigs Angina? Not in retropharyngeal space 62) Pt with pigmentation on the face, heart and kidney problems? Lupus

Combined remembered Qs 2020

63) What antifungal can be given orally? Clothrimazole 64) How long to splint a tooth with a mid root fracture? 7-10 days 65) Photo initiator for composite? Camphoroquinone 66) Bone resorption IL? IL1 67) What fibers are stimulated when using Electrical test? A-Delta fibers 68) What type of radition on MRI? 69) Posterior border max denture? Coronoid process 70) Posterior border mand denture? Masseter muscle 71) Muscle that wont allow the pt lift his upper lip? Orbicularis oris 72) 1996 children with no healthcare.. something like that? I put medicaid 73) Pt with Down syndrome: No rampant caries 74) Best prognosis for a GTR? 3 walls defect 75) Dentinogenesis imperfecta and Amelogenesis Imperfecta are not related 76) Daughter bringing her dad to the dentist, she was his legal guardian. How to handle that situation 77) Picture of a metal frame work, the question said "the lab rejected the design" what to do?-- tell the lab to send the cast back to re-design 78) What not to give to a pt taking Gingseng? Aspirin 79) Whats not an adverse effect of lisinopril? Cough, high pottassium levels, interaction with Nsaids or orthostatic hypotension 80) Antihistamine MOA: Block receptors Physiological antagonist are drugs which are not effect directly on the same receptors although they have different effects. They do not attach to the same receptors. But antihistamine drugs attach to the receptors. 81) Collimation: Reduces exposure 82) Highest absorption of radiation? Scattered from pt 83) Picture of a pt with cervical caries on central and lateral incisors? biggest concern? possibility of isolation 84) Picture of abrasion and how to restore.. I chose composite

Combined remembered Qs 2020

85) Pt who is 25 years old, no history of extractions and no 3rd molars.. cause? Agenesia 86) Xray of a 9 year old.. 2 radiolucencies on areas of 2nd molars.. what are they? lesions? I put 2nd molars formation 87) Picture of an amalgam and a grey shade surroinding it.. what is it? I put caries.. how to restore? I put crown and possible post and core! It was big and near the pulp 88) Picture of a pt with atrophic candidiasis. 89) Basal membrane lesion? Pemphigus 90) Pt complaining that she shows too much gum when she smiles.. how to fix that? Lefort I 91) What includes a Lefort I? Max sinus 92) Califlower lesion? Papilloma 93) Picture of a pt missing some teeth. What is the concern if you are planning on giving him an RPD? He didnt have space on the right side for RPD 94) Pt with no attached gingiva on 2nd molar. What not to do.. Distal wedge 95) Xray of pneumatization of max sinus and if its true that it happens when the pt has chronic sinusitis 96) Sealants? Mechanical retention 97) Systemic fungal infection? I put histoplasmosis 98) Pt with pulp necrosis and sinus tract, what to do after RCT? no further tx needed. 99) Apexification apexogenesis questions 100) Not true about Sodium Hypochlorite? Chelating agent 101) Trauma to PDL? Intrusion 102) Least chance of creating a ledge? Short root, long root, curved root. I put short root. 103) Critial PH of enamel? 5.5 104) Preparation for an RPD.. Guide planes first 105) What pontic design is not recommended for the anterior area? Hygienic 106) What gives composite the radiopacity? I put the matrix 107) About 3 questions on trigeminal neuralgia 108) 3 questions on racial pigmentation of the gums 109) Pontic should? Gently rest on tissue

Combined remembered Qs 2020

110) There was a picture of a resin base partial picture and the question was, whats the difference between this partial and a cast metal with acrylic partial. I put they dont act the same during vertical forces 111) You tell a pt he has to replace his amalgams for composites.. what principle? Veracity 112) Cohort study 113) Unbundling 114) High blood pressure.. Make sure you know when its ok to treat and when its not 115) Dentist ignores pt behavior? Extinction 116) Tell show do tecnique 117) Epiphyseal plates? Synchondrosis 118) Cleft lip how many weeks in utero? 6-9 119) Hyoid bone twice on Xrays

JUNE 15

1. Collimation- reduces xray beam size and reduces low energy radiation we’re both in the options ​ ​ ○ Filtration obstruct low energy wavelength 2. Why do you Bead a max denture? Options were Support, strengthen base, retention, form a finish line, improve tissue contact

Combined remembered Qs 2020

So for rpd tissue contact and for max denture retention

3. What is in sealer that prevents resin polymerization? Eugenol ​ 4. You have a Retruded tongue. How does this affect dentures? Mastication, speaking, not being able to ​ stabilize mandibular as well 5. Root amputatation how to do it exactly 6. Root amputation - max 1 m, max 2 m, mand molars which ? If the tooth has Tongue blade - know anterior versus posterior and if it’s for one tooth versus bilateral ans: one tooth ant cross bite 7. Emphysema: is it proximal or distal airspace and is it constriction or dilation ​ ​ ​ ​ 8. Histamine - know it’s role bronchiole constriction, precursor is histidine, high amount in skin? It’s all except question and they all seemed correct so just look into what histamine does

Combined remembered Qs 2020

9. Intrapulpal injection for the pulp? Subperiosteal, Supraperiosteal, intraosseous ​

10. Tramadol was answer to opioid abuser question 11. What is unique about opioids? a. Diarrhea b. I put can inc CNS depression when mixed with other drugs ​ ​

12. Trigeminal neuralgia? a. Flushing, b. certain trigger points

Combined remembered Qs 2020

13. pen and tetra? 14. High amount in GCF? Doxy or mino wasn’t in options, I put tetra 15. Most soluble anesthetic? a. lido b. articaine c. prilocaine 16. Match the drug to what it does EXCEPT: zidovudine and varicella (Zido is for HIV) 17. Kaposi or syphilis associated w both transmitted through sex . saliva and blood, Kaposi HHV8 18. What anesthetic for pregnant lady? I was between meperidine and lido ​ I think it’s lido 19. Hypercementosis? Pagets 20. Amellogensis imperfecta is in the histodifferntiation phase - but some sources say apposition so look it up 21. Ectodermal dysplasia - oligodontia BUT options were tricky. They all sounded correct. I even think congenitally absent teeth was an option. 22. Hypodontia affects what ? Maxilla, mandible, midfacial, alveolus 23. What affects hair thickness? Thyroid ​ 24. What is not associated w cleft palate? All seemed correct: microdontia, ectopic erupted teeth, congenitally missing teeth. Idk what right answer was look into it 25. Same thing that causes herpes? a. Measles, b. mumps, c. chicken pox, ​ d. herpangina

Combined remembered Qs 2020

26. If you have HIV you get ? Viral, bacterial, infection 27. Some weird about rubella (it had a second name like encephalo) and if it was chromosomal, genetic, viral, congenital (? Idk if that was an option) 28. What’s an advantage of resin? 29. Humulin r question about how much meds and food to give a. only fluid and half dose of insulin 30. Why not put a child lock on old person medication? Physical barrier ? ​ ​ 31. Affected and infected which is true? Very confusing answers but I put that there is bacteria present in infected. Another option was that infected has no remineralization potential 32. Most likely to cause ear pain? a. Max 1 molar b. mand molar c. max pre molar

I also had the one where if IANB doesn’t work like the lip isn’t numb but everything else is ? I put mental Scaling and root planing? To remove calculus and clean root or to remove cementum and expose collagen Incidence Lingual Groove that commonly has caries common in? Incisors, canine, PM, molar

Combined remembered Qs 2020

Which of the following is not for shock? Hypertension? IV Dextrose is patient is unconscious You gave oxygen to patient having heart attack. They start to get better. What do you do? Add more, continue treatment, send them to the doc 3rd molar disappears ? Caldwell or take a radiograph to visualize Endo bacteria? Strict anaerobes Arrested caries? Brown and soft or brown and shiny ? PSA is branch of maxillary from Infratemporal You have a first max molar coming Down (extruded) and a missing opposing molar you’re going to do a bridge on. Why would you first fix the max molar before bridge? Level occlusal plane or properly distribute occlusal forces Why do you do a suture? I put to reduce granulation tissue 3 qs on hemisection and root amputation very specific

11.Ingredients of marijuana? Tetrahydrocannabinol ​ 12.Indirect retainers prevent? Portions were towards the tissue ward and other way. 13.Pupal and periapical differences( they use different words and ask many questions on it) 14.Mandi nerve enter cranium? infratemporal fossa 15.Mandi nerve exit cranium?foramen ovale 16.Apex locator what not do? Perforation? 17.2 Implant distance to implant is 3 … to tooth is 1.5mm 18.Sterilization controls? FDA 19.Which does not Monitor sterilization unit?options are Chemical, electrical, biological, mechanical. 20.Bisphosphonates acts on which cell?options osteclast, osteoblasts. 21.Trisomy 21 infrequently seen? Options Rampant caries, others all symptoms of trisomy 21. 22.distal border extension of denture(didn’t mention which one upper or lower)? Options mylohyoid line, Hamilcar botch, mental foramen, incisive foramen. 23.benzodiazapene reversal? 24.hydrocodone reversal? 25.pregnant pt have acute abscess event to treat? Options 1,2,3trimeter, immediately when she have. 26.cantilever bridge what not true.options difficult to clean and others I don’t remember. 27.recent study show relation of periodontitis with what? Options cardiovascular, hypothyroidism, hepatitis c, cancer. 28.differential wbcs count diagnosis of? Eosinophilia, anemia, spherocytosis, thrombocytopenic purpura.

Combined remembered Qs 2020

29.which two factor or variable helps to diagnosis?options clinical diagnosis and history, radiographs, biopsy, lab results. 30.Lot of prostho questions related to occlusion. No questions on materials was asked in my exam. That’s all I remember for day1. But Day 1 was little hard and had lot many tricky questions. Day2 was simple and good. Cases were doable. Make sure to check time. All the best to all and hope we all pass